2005 Book ProblemsInAlgebraicNumberTheor
2005 Book ProblemsInAlgebraicNumberTheor
Ram Murty
Jody Esmonde
Problems in
Algebraic Number Theory
Second Edition
M. Ram Murty Jody Esmonde
Department of Mathematics and Statistics Graduate School of Education
Queen’s University University of California at Berkeley
Kingston, Ontario K7L 3N6 Berkeley, CA 94720
Canada USA
[email protected] [email protected]
Editorial Board
S. Axler F.W. Gehring K.A. Ribet
Mathematics Department Mathematics Department Mathematics Department
San Francisco State East Hall University of California,
University University of Michigan Berkeley
San Francisco, CA 94132 Ann Arbor, MI 48109 Berkeley, CA 94720-3840
USA USA USA
[email protected] [email protected]. [email protected]
umich.edu
9 8 7 6 5 4 3 2 1 SPIN 10950647
springeronline.com
It is practice first and knowledge afterwards.
Vivekananda
Preface to the Second
Edition
Since arts are more easily learnt by examples than precepts, I have
thought fit to adjoin the solutions of the following problems.
Isaac Newton, in Universal Arithmetick
Learning is a mysterious process. No one can say what the precise rules
of learning are. However, it is an agreed upon fact that the study of good
examples plays a fundamental role in learning. With respect to mathemat-
ics, it is well-known that problem-solving helps one acquire routine skills in
how and when to apply a theorem. It also allows one to discover nuances of
the theory and leads one to ask further questions that suggest new avenues
of research. This principle resonates with the famous aphorism of Lichten-
berg, “What you have been obliged to discover by yourself leaves a path in
your mind which you can use again when the need arises.”
This book grew out of various courses given at Queen’s University be-
tween 1996 and 2004. In the short span of a semester, it is difficult to cover
enough material to give students the confidence that they have mastered
some portion of the subject. Consequently, I have found that a problem-
solving format is the best way to deal with this challenge. The salient
features of the theory are presented in class along with a few examples, and
then the students are expected to teach themselves the finer aspects of the
theory through worked examples.
This is a revised and expanded version of “Problems in Algebraic Num-
ber Theory” originally published by Springer-Verlag as GTM 190. The
new edition has an extra chapter on density theorems. It introduces the
reader to the magnificent interplay between algebraic methods and analytic
methods that has come to be a dominant theme of number theory.
I would like to thank Alina Cojocaru, Wentang Kuo, Yu-Ru Liu, Stephen
Miller, Kumar Murty, Yiannis Petridis and Mike Roth for their corrections
and comments on the first edition as well as their feedback on the new
material.
vii
Preface to the First
Edition
by students to prepare themselves for the Cambridge Tripos. Ramanujan made it famous
by using it as a problem book.
ix
x Preface
independent solving of challenging problems will aid the reader far more
than aphorisms.”
Asking how one does mathematical research is like asking how a com-
poser creates a masterpiece. No one really knows. However, it is clear
that some preparation, some form of training, is essential for it. Jacques
Hadamard, in his book The Mathematician’s Mind, proposes four stages
in the process of creation: preparation, incubation, illumination, and ver-
ification. The preparation is the conscious attention and hard work on a
problem. This conscious attention sets in motion an unconscious mecha-
nism that searches for a solution. Henri Poincaré compared ideas to atoms
that are set in motion by continued thought. The dance of these ideas in the
crucible of the mind leads to certain “stable combinations” that give rise
to flashes of illumination, which is the third stage. Finally, one must verify
the flash of insight, formulate it precisely, and subject it to the standards
of mathematical rigor.
This book arose when a student approached me for a reading course on
algebraic number theory. I had been thinking of writing a problem book on
algebraic number theory and I took the occasion to carry out an experiment.
I told the student to round up more students who may be interested and so
she recruited eight more. Each student would be responsible for one chapter
of the book. I lectured for about an hour a week stating and sketching the
solution of each problem. The student was then to fill in the details, add
ten more problems and solutions, and then typeset it into TEX. Chapters 1
to 8 arose in this fashion. Chapters 9 and 10 as well as the supplementary
problems were added afterward by the instructor.
Some of these problems are easy and straightforward. Some of them
are difficult. However, they have been arranged with a didactic purpose.
It is hoped that the book is suitable for independent study. From this
perspective, the book can be described as a first course in algebraic number
theory and can be completed in one semester.
Our approach in this book is based on the principle that questions focus
the mind. Indeed, quest and question are cognates. In our quest for truth,
for understanding, we seem to have only one method. That is the Socratic
method of asking questions and then refining them. Grappling with such
problems and questions, the mind is strengthened. It is this exercise of the
mind that is the goal of this book, its raison d’être. If even one individual
benefits from our endeavor, we will feel amply rewarded.
We would like to thank the students who helped us in the writing of this
book: Kayo Shichino, Ian Stewart, Bridget Gilbride, Albert Chau, Sindi
Sabourin, Tai Huy Ha, Adam Van Tuyl and Satya Mohit.
We would also like to thank NSERC for financial support of this project
as well as Queen’s University for providing a congenial atmosphere for this
task.
J.E.
M.R.M.
August 1998
xi
Contents
Acknowledgments xi
I Problems
1 Elementary Number Theory 3
1.1 Integers . . . . . . . . . . . . . . . . . . . . . . . . . . . . . 3
1.2 Applications of Unique Factorization . . . . . . . . . . . . . 8
1.3 The ABC Conjecture . . . . . . . . . . . . . . . . . . . . . 9
1.4 Supplementary Problems . . . . . . . . . . . . . . . . . . . . 10
2 Euclidean Rings 13
2.1 Preliminaries . . . . . . . . . . . . . . . . . . . . . . . . . . 13
2.2 Gaussian Integers . . . . . . . . . . . . . . . . . . . . . . . . 17
2.3 Eisenstein Integers . . . . . . . . . . . . . . . . . . . . . . . 17
2.4 Some Further Examples . . . . . . . . . . . . . . . . . . . . 21
2.5 Supplementary Problems . . . . . . . . . . . . . . . . . . . . 25
4 Integral Bases 41
4.1 The Norm and the Trace . . . . . . . . . . . . . . . . . . . . 41
4.2 Existence of an Integral Basis . . . . . . . . . . . . . . . . . 43
4.3 Examples . . . . . . . . . . . . . . . . . . . . . . . . . . . . 46
4.4 Ideals in OK . . . . . . . . . . . . . . . . . . . . . . . . . . . 49
4.5 Supplementary Problems . . . . . . . . . . . . . . . . . . . . 50
xiii
xiv CONTENTS
5 Dedekind Domains 53
5.1 Integral Closure . . . . . . . . . . . . . . . . . . . . . . . . . 53
5.2 Characterizing Dedekind Domains . . . . . . . . . . . . . . 55
5.3 Fractional Ideals and Unique Factorization . . . . . . . . . . 57
5.4 Dedekind’s Theorem . . . . . . . . . . . . . . . . . . . . . . 63
5.5 Factorization in OK . . . . . . . . . . . . . . . . . . . . . . 65
5.6 Supplementary Problems . . . . . . . . . . . . . . . . . . . . 66
7 Quadratic Reciprocity 81
7.1 Preliminaries . . . . . . . . . . . . . . . . . . . . . . . . . . 81
7.2 Gauss Sums . . . . . . . . . . . . . . . . . . . . . . . . . . . 84
7.3 The Law of Quadratic Reciprocity . . . . . . . . . . . . . . 86
7.4 Quadratic Fields . . . . . . . . . . . . . . . . . . . . . . . . 88
7.5 Primes in Special Progressions . . . . . . . . . . . . . . . . 91
7.6 Supplementary Problems . . . . . . . . . . . . . . . . . . . . 94
II Solutions
Bibliography 347
Index 349
Part I
Problems
Chapter 1
Elementary Number
Theory
1.1 Integers
The nineteenth century mathematician Leopold Kronecker wrote that “all
results of the profoundest mathematical investigation must ultimately be
expressible in the simple form of properties of integers.” It is perhaps this
feeling that made him say “God made the integers, all the rest is the work
of humanity” [B, pp. 466 and 477].
In this section, we will state some properties of integers. Primes, which
are integers with exactly two positive divisors, are very important in number
theory. Let Z represent the set of integers.
3
4 CHAPTER 1. ELEMENTARY NUMBER THEORY
(say). Notice det A = ±1 and A−1 has integer entries whose bottom row
gives x, y ∈ Z such that ax + by = 1.
Theorem 1.1.2 Every positive integer greater than 1 has a prime divisor.
Proof. Suppose that there is a positive integer having no prime divisors.
Since the set of positive integers with no prime divisors is nonempty, there
is a least positive integer n with no prime divisors. Since n divides itself,
n is not prime. Hence we can write n = ab with 1 < a < n and 1 < b < n.
Since a < n, a must have a prime divisor. But any divisor of a is also
a divisor of n, so n must have a prime divisor. This is a contradiction.
Therefore every positive integer has at least one prime divisor. !
We can use the same method to prove the following more general result.
Exercise 1.1.9 Prove that if n is a composite integer, then n has a prime factor
√
not exceeding n.
Exercise 1.1.10 Show that if the smallest prime factor p of the positive integer
√
n exceeds 3 n, then n/p must be prime or 1.
6 CHAPTER 1. ELEMENTARY NUMBER THEORY
!Exercise
" 1.1.11 Let p be prime. Show that each of the binomial coefficients
p
k
, 1 ≤ k ≤ p − 1, is divisible by p.
Exercise 1.1.12 Prove that if p is an odd prime, then 2p−1 ≡ 1 (mod p).
by interpreting the left-hand side as the probability that a random number chosen
from 1 ≤ a ≤ n is coprime to n.
Let π(x) be the number of primes less than or equal to x. The prime
number theorem asserts that
x
π(x) ∼
log x
as x → ∞. This was first proved in 1896, independently by J. Hadamard
and Ch. de la Vallée Poussin.
We will not prove the prime number theorem here, but derive various
estimates for π(x) by elementary methods.
1.1. INTEGERS 7
pk+1 ≤ p1 p2 · · · pk + 1.
Exercise 1.1.24 By observing that any natural number can be written as sr2
with s squarefree, show that
√
x ≤ 2π(x) .
Deduce that
log x
π(x) ≥ .
2 log 2
&
Exercise 1.1.25 Let ψ(x) = pα ≤x log p where the summation is over prime
powers pα ≤ x.
(i) For 0 ≤ x ≤ 1, show that x(1 − x) ≤ 41 . Deduce that
' 1
1
xn (1 − x)n dx ≤
0 4n
ψ(2n + 1) ≥ 2n log 2.
x log 2
π(x) ≥
2 log x
for x ≥ 6.
show that
9x log 2
π(x) ≤
log x
for every integer x ≥ 2.
8 CHAPTER 1. ELEMENTARY NUMBER THEORY
Exercise 1.2.5 Prove that if f (x) ∈ Z[x], then f (x) ≡ 0 (mod p) is solvable for
infinitely many primes p.
Exercise 1.2.6 Let q be prime. Show that there are infinitely many primes p so
that p ≡ 1 (mod q).
the Fermat numbers. Fermat made the conjecture that these integers are
all primes. Indeed, F0 = 3, F1 = 5, F2 = 17, F3 = 257, and F4 = 65537
5
are primes but unfortunately, F5 = 22 + 1 is divisible by 641, and so F5 is
composite. It is unknown if Fn represents infinitely many primes. It is also
unknown if Fn is infinitely often composite.
Exercise 1.2.7 Show that Fn divides Fm − 2 if n is less than m, and from this
deduce that Fn and Fm are relatively prime if m #= n.
n
Exercise 1.2.8 Consider the nth Fermat number Fn = 22 +1. Prove that every
prime divisor of Fn is of the form 2n+1 k + 1.
α
Exercise 1.2.9 Given a natural number n, let n = pα 1
1 · · · pk
k
be its unique
factorization as a product of prime powers. We define the squarefree part of n,
denoted S(n), to be the product of the primes pi for which αi = 1. Let f (x) ∈ Z[x]
be nonconstant and monic. Show that lim inf S(f (n)) is unbounded as n ranges
over the integers.
1.3. THE ABC CONJECTURE 9
Exercise 1.3.1 Assuming the ABC Conjecture, show that if xyz #= 0 and xn +
y n = z n for three mutually coprime integers x, y, and z, then n is bounded.
Exercise 1.3.3 Assuming the ABC Conjecture, show that there are infinitely
many primes p such that 2p−1 #≡ 1 (mod p2 ).
2p−1 #≡ 1 (mod p2 )
2p−1 !≡ 1 (mod p2 ),
Exercise 1.3.5 Show that if the Erdös conjecture above is true, then there are
infinitely many primes p such that 2p−1 #≡ 1 (mod p2 ).
10 CHAPTER 1. ELEMENTARY NUMBER THEORY
Exercise 1.3.6 Assuming the ABC Conjecture, prove that there are only finitely
many n such that n − 1, n, n + 1 are squarefull.
Exercise 1.3.7 Suppose that a and b are odd positive integers satisfying
rad(an − 2) = rad(bn − 2)
for every natural number n. Assuming ABC, prove that a = b. (This problem is
due to H. Kisilevsky.)
Exercise 1.4.3 Prove that if the number of prime Fermat numbers is finite, then
the number of primes of the form 2n + 1 is finite.
Exercise 1.4.5 An integer is called perfect if it is the sum of its divisors. Show
that if 2n − 1 is prime, then 2n−1 (2n − 1) is perfect.
Exercise 1.4.7 Show that there are no integer solutions to the equation x4 −y 4 =
2z 2 .
Exercise 1.4.8 Let p be an odd prime number. Show that the numerator of
1 1 1
1+ + + ··· +
2 3 p−1
is divisible by p.
Exercise 1.4.9 Let p be an odd prime number greater than 3. Show that the
numerator of
1 1 1
1 + + + ··· +
2 3 p−1
is divisible by p2 .
Exercise 1.4.10 (Wilson’s Theorem) Show that n > 1 is prime if and only
if n divides (n − 1)! + 1.
1.4. SUPPLEMENTARY PROBLEMS 11
Exercise 1.4.11 For each n > 1, let Q be the product of all numbers a < n
which are coprime to n. Show that Q ≡ ±1 (mod n).
Exercise 1.4.13 Use Exercises 1.2.7 and 1.2.8 to show that there are infinitely
many primes ≡ 1 (mod 2r ) for any given r.
Exercise 1.4.17 Assuming ABC, show that there are only finitely many con-
secutive cubefull numbers.
Euclidean Rings
2.1 Preliminaries
We can discuss the concept of divisibility for any commutative ring R with
identity. Indeed, if a, b ∈ R, we will write a | b (a divides b) if there exists
some c ∈ R such that ac = b. Any divisor of 1 is called a unit. We will
say that a and b are associates and write a ∼ b if there exists a unit u ∈ R
such that a = bu. It is easy to verify that ∼ is an equivalence relation.
Further, if R is an integral domain and we have a, b != 0 with a | b and
b | a, then a and b must be associates, for then ∃c, d ∈ R such that ac = b
and bd = a, which implies that bdc = b. Since we are in an integral domain,
dc = 1, and d, c are units.
We will say that a ∈ R is irreducible if for any factorization a = bc, one
of b or c is a unit.
13
14 CHAPTER 2. EUCLIDEAN RINGS
Exercise 2.1.4 Let R be a domain satisfying (i) above. Show that (ii) is equiv-
alent to (ii$ ): if π is irreducible and π divides ab, then π | a or π | b.
ax + πy = 1,
⇒ abx + πby = b.
Exercise 2.1.8 If F is a field, prove that F [x], the ring of polynomials in x with
coefficients in F , is Euclidean.
and
g(x) = db0 + db1 x + · · · + dbm xm ,
16 CHAPTER 2. EUCLIDEAN RINGS
Exercise 2.2.5 A positive integer a is the sum of two squares if and only if
a = b2 c where c is not divisible by any positive prime p ≡ 3 (mod 4).
(x − ρ)(x − ρ) = (x − ρ)(x − ρ2 ) = x2 + x + 1
so that
±(θ3 ∓ 1) = ξ3 − 1
= (ξ − 1)(ξ − ρ)(ξ − ρ2 )
= (dλ)(dλ + 1 − ρ)(1 + dλ − ρ2 )
= dλ(dλ + λ)(dλ − λρ2 )
= λ3 d(d + 1)(d − ρ2 ).
±1 ± 1 ± 1 ≡ 0 (mod λ4 )
could be satisfied. The left side of this congruence gives ±1 or ±3; certainly
±1 is not congruent to 0 (mod λ4 ) since λ4 is not a unit. Also, ±3 is not
2.3. EISENSTEIN INTEGERS 19
We notice that these Bi are pairwise coprime, since if for some prime p, we
had p | B1 and p | B2 , then necessarily we would have
p | B1 − B2 = β
and
p | λB1 + B2 − B1 = α.
This is only possible for a unit p since gcd(α, β) = 1. Similarly, we can
verify that the remaining pairs of Bi are coprime. Since λ3n−2 | A1 , we
have λ3n−3 | B1 . So we may rewrite (2.1) as
−ελ3n−3 δ 3 = B1 B2 B3 .
From this equation we can see that each of the Bi is an associate of a cube,
since they are relatively prime, and we write
B1 = e1 λ3n−3 C13 ,
B2 = e2 C23 ,
B3 = e3 C33 ,
A1 = α + β,
A2 = α + ρβ,
A3 = α + ρ2 β.
so we have that
0 = ρ2 λB3 + ρλB2 + λB1
and
0 = ρ2 B3 + ρB2 + B1 .
We can then deduce that
Solution. We first consider the case where y is even. It follows that x must
also be even, which implies that x3 ≡ 0 (mod 8). Now, y is congruent to
0 or 2 (mod 4). If y ≡ 0 (mod 4), then y 2 + 4 ≡ 4 (mod 8), so we can
rule out this case. However, if y ≡ 2 (mod 4), then y 2 + 4 ≡ 0 (mod 8).
Writing y = 2Y with Y odd, and x = 2X, we have 4Y 2 + 4 = 8X 3 , so that
Y 2 + 1 = 2X 3
and
(Y + i)(Y − i) = 2X 3 = (1 + i)(1 − i)X 3 .
(Y + i)(Y − i)
X3 =
(1 + i)(1 − i)
! "! "
1+Y 1−Y 1+Y 1−Y
= + i − i
2 2 2 2
! "2 ! "2
1+Y 1−Y
= + .
2 2
We shall write this last sum as a2 + b2 . Since Y is odd, a and b are integers.
Notice also that a + b = 1 so that gcd(a, b) = 1. We now have that
X 3 = (a + bi)(a − bi).
We would like to establish that (a+bi) and (a−bi) are relatively prime. We
assume there exists some nonunit d such that d | (a + bi) and d | (a − bi).
But then d | [(a + bi) + (a − bi)] = 2a and d | (a + bi) − (a − bi) = 2bi. Since
gcd(a, b) = 1, then d | 2, and thus d must have even norm. But then it is
impossible that d | (a + bi) since the norm of (a + bi) is a2 + b2 = X 3 which
is odd. Thus (a + bi) and (a − bi) are relatively prime, and each is therefore
a cube, since Z[i] is a unique factorization domain. We write
a = s3 − 3st2 ,
b = 3s2 t − t3 .
Adding these two equations yields a+b = s3 −3st2 +3s2 t−t3 . But a+b = 1,
so we have
1 = s3 − 3st2 + 3s2 t − t3
= (s − t)(s2 + 4st + t2 ).
(y + i)(y − i) = xp .
2.4. SOME FURTHER EXAMPLES 23
(y + i) = ik (a + bi)p .
Now,
(y − i) = (y + i) = (−i)k (a − bi)p .
Thus
(y + i) − (y − i) = 2i
= ik (a + bi)p − (−i)k (a − bi)p .
Thus ! "
* p
(k−1)/2
1 = (−1) p−j
a (b) (−1)
j j/2
.
even j,
j
0≤j<p
24 CHAPTER 2. EUCLIDEAN RINGS
Since a divides every term on the right-hand side of this equation, then
a | 1 and a = ±1. We observed previously that only one of a, b is odd; thus
b is even. We now substitute a = ±1 into the last equation above to get
* ! "
p
±1 = (b)j (−1)j/2
even j,
j
0≤j<p
! " ! " ! "
p p p
= 1 − b2 + b4 − · · · ± bp−1 .
2 4 p−1
If the sign of 1 on the left-hand side of this equality were negative, we would
have that b2 | 2; b is even and in particular b != ±1, so this is impossible.
Thus
! " ! " ! "
p p p
0 = −b2 + b4 − · · · ± bp−1
2 4 p−1
! " ! " ! "
p p p
= − + b2 − · · · ± bp−3 .
2 4 p−1
# $
Now we notice that #2 $| b, so 2 | p2 . If p ≡ 3 (mod 4), then we are
finished because
#p$ 2 ! 2 . Suppose in fact
p
that 2q is the largest power of
2 #dividing
$ 2 . We # shall
$ show that 2
q+1
will then divide every term in
2 p p−3 p
b 4 − ··· ± b p−1 , and this will establish that no b #will $
satisfy our
equation. We consider one of these terms given by (b)j−2 pj , for an even
#p$
value of j; we rewrite this as b2m−2 2m (we are not concerned with the
sign of the term). We see that
! " ! "
p p−2 (p)(p − 1)
=
2m 2m − 2 2m(2m − 1)
! "! "
p−2 p 1
= ,
2m − 2 2 m(2m − 1)
so we are considering a term
! "! "
p−2 p b2m−2
.
2m − 2 2 m(2m − 1)
# $
Now, 2q | p2 by assumption. Recall that b is even; thus 22m−2 | b2m−2 .
Now m ≥ 2; it is easy to see then that 2m − 2 ≥ m, so 22m−2 does not
divide m. Thus when we reduce the fraction
b2m−2
m(2m − 1)
to lowest terms, the numerator is even and the denominator is odd. There-
fore, ! "! "
p−2 p b2m−2
2(2 ) |
q
.
2m − 2 2 m(2m − 1)
2.5. SUPPLEMENTARY PROBLEMS 25
# $ # p $ p−3
Thus 2q+1 divides every term in b2 p4 − · · · ± p−1 b and we deduce that
no value of b can satisfy our equation.
Case 2. k is even.
This case is almost identical to the first case; we mention only the
relevant differences. When we expand
√
Exercise 2.5.6 Show that Z[(1 + −19)/2] is not Euclidean for the norm map.
√
Exercise 2.5.7 Prove that Z[ −10] is not a unique factorization domain.
√
Exercise 2.5.8 Show that there are only finitely many rings Z[ d] with d ≡ 2
or 3 (mod 4) which are norm Euclidean.
and * √ i
bi 2 3n−i = 0
i even
27
28 CHAPTER 3. ALGEBRAIC NUMBERS AND INTEGERS
The degree of p(x) is called the degree of α and is denoted deg(α); p(x)
is called the minimal polynomial of α.
Complex numbers which are not algebraic are called transcendental .
Well before an example of a transcendental number was known, mathe-
maticians were assured of their existence.
Example 3.1.5 Show that the set of algebraic numbers is countable (and
hence the set of transcendental numbers is uncountable).
3.2. LIOUVILLE’S THEOREM AND GENERALIZATIONS 29
Solution. All polynomials in Q[x] have a finite number of roots. The set
of rational numbers, Q, is countable and so the set Q[x] is also countable.
The set of algebraic numbers is the set of all roots of a countable number of
polynomials, each with a finite number of roots. Hence the set of algebraic
numbers is countable.
Since algebraic numbers and transcendental numbers partition the set
of complex numbers, C, which is uncountable, it follows that the set of
transcendental numbers is uncountable.
√
Exercise 3.1.6 Find the minimal polynomial of n where n is a squarefree
integer.
√
Exercise 3.1.7 Find the minimal polynomial of 2/3.
holds.
If |p/q| ≤ M , then 1 1
1 p 11
1αi − ≤ 3M
1 q1
so that
1 1
1
1α − p 11 1 1
1 1 ≥ 2n ≥ .
q |an |q n
j=2 |αj − p/q| |an |(3M )n−1 q n
Using this theorem, Liouville was able to give specific examples of tran-
scendental numbers.
is transcendental.
Solution. Suppose not, and call the sum α. Look at the partial sum
*k
1 pk
= ,
n=0
10 n! qk
5∞It is−n!
easy to see that this argument can be generalized to show that
n=0 a is transcendental for all positive integers a. We will prove this
fact in the Supplementary Exercises for this chapter.
Hence,
1 1 1 1
1x 1 1x 1
|f (x, y)| = |y | 1 − α1 1 · · · 1 − αn 11 ,
1 n1 1
y y
1 1
1 x 1
|m| ≥ k|y|n 11 − α1 11 ,
y
1 1
|m| 1x 1
n
≥ 1 − α1 11 ,
1
k|y| y
where
n
6
1
k= |αi − α1 |.
2n−1 i=2
c m 1 m(ck)−1
≤ ⇔ ≤ .
y n/2+1 ky n y n/2+1 yn
However, for n ≥ 3, this holds for only finitely many (x, y), contradicting
our assumption. Thus f (x, y) has only finitely many solutions.
for any ε > 0. This improved inequality gives us a new family of transcen-
dental numbers.
&∞ n
Exercise 3.2.4 Show that n=1 2−3 is transcendental.
&∞
Exercise 3.2.5 Show that, in fact, n=1 2−f (n) is transcendental whenever
f (n + 1)
lim > 2.
n→∞ f (n)
Notice that
n
* m
*
φ(g) + φ(h) = ai αi + bi αi = φ(g + h)
i=0 i=0
and
/ n 0 m
* * *
φ(g)φ(h) = ai αi bj α j = ai bj αi+j = φ(gh).
i=0 j=0 0≤i+j≤n+m
Exercise 3.3.3 Let α be an algebraic number and let p(x) be its minimal poly-
nomial. Show that p(x) has no repeated roots.
The roots of the minimal polynomial p(x) of α are called the conjugate
roots or conjugates of α. Thus, if n is the degree of p(x), then α has n
conjugates.
If θ = θ(1) and θ(2) , . . . , θ(n) are the conjugates of θ, then Q(θ(i) ), for
i = 2, . . . , n, is called a conjugate field to Q(θ). Further, the maps θ → θ(i)
are monomorphisms of K = Q(θ) → Q(θ(i) ) (referred to as embeddings of
K into C).
We can partition the conjugates of θ into real roots and nonreal roots
(called complex roots).
K is called a normal extension (or Galois extension) of Q if all the
conjugate fields of K are identical √ to K. For example, any quadratic exten-
sion√ of Q is normal. √ However, Q( 3
2) is not
√ since the two conjugate fields
2 3
Q(ρ 2) and Q(ρ 2) are distinct from Q( 3 2). (Here ρ is a primitive cube
3
root of unity.)
We also define the normal closure of any field K as the extension K̃
of smallest degree containing all the conjugate fields of K. Clearly this is
well-defined for if there were two such fields, K̃1 and K̃2 , then K̃1 ∩ K̃2
would have the same property and have smaller √ degree if K̃
√1 != K̃2 . In the
above example, the normal closure of Q( 3 2) is clearly Q( 3 2, ρ).
Exercise 3.3.8 Show that Z[x] is not (a) Euclidean or (b) a PID.
36 CHAPTER 3. ALGEBRAIC NUMBERS AND INTEGERS
αN = αN −n αn
= αN −n [−(a0 + a1 α + · · · + an−1 αn−1 )]
= (−αN −n a0 )1 + (−αN −n a1 )α + · · · + (−αN −n an−1 )αn−1 .
is transcendental for a ∈ Z, a ≥ 2.
is transcendental for a ∈ Z, a ≥ 2.
Exercise 3.4.5 Using Thue’s theorem, show that f (x, y) = x6 +7x5 y −12x3 y 3 +
6xy 5 + 8y 6 = m has only a finite number of solutions for m ∈ Z∗ .
Exercise 3.4.10 Let I be a subset of the positive integers ≤ m which are coprime
to m. Set # i
f (x) = (x − ζm ).
i∈I
Exercise 3.4.13 Let ζm denote a primitive mth root of unity. Show that Q(ζm )
is normal over Q.
Exercise 3.4.14 Let a be squarefree and greater than 1, and let p be prime.
Show that the normal closure of Q(a1/p ) is Q(a1/p , ζp ).
Chapter 4
Integral Bases
so TrK (α) = TrA and NK (α) = det A where A is the matrix (aij ).
41
42 CHAPTER 4. INTEGRAL BASES
5n
Proof. We begin by writing αωi = j=1 aij ωj ∀i. Then we have
n
*
(k) (k)
α(k) ωi = aij ωj ∀i, k,
j=1
;
0 if i != j,
where δij = Now, if we define the matrices
1 if i = j.
(j)
A0 = (α(i) δij ), Ω = (ωi ), A = (aij ),
Exercise 4.1.2 Let K = Q(i). Show that i ∈ OK and verify that TrK (i) and
NK (i) are integers.
√
Exercise 4.1.3 Determine the algebraic integers of K = Q( −5).
Then
*
B(v, u) = B(ai ei , u)
i
*
= ai B(ei , u)
i
*
= ai bj B(ei , ej )
i,j
(B(ωi , ωj )) = ΩΩT ,
with 0 ≤ ci < pii , and there are p11 p22 · · · pnn of them. So [M : N ] =
p11 p22 · · · pnn . !
Exercise 4.2.5 Show that the discriminant is well-defined. In other words, show
that given ω1 , ω2 , . . . , ωn and θ1 , θ2 , . . . , θn , two integral bases for K, we get the
same discriminant for K.
dK/Q (a1 , a2 , . . . , an ) = m2 dK .
46 CHAPTER 4. INTEGRAL BASES
4.3 Examples
Example 4.3.1 Suppose that the minimal polynomial of α is Eisensteinian
with respect to a prime p, i.e., α is a root of the polynomial
xn + an−1 xn−1 + · · · + a1 x + a0 ,
αn + an−1 αn−1 + · · · + a1 α + a0 = 0,
pξ = b0 + b1 α + · · · + bn−1 αn−1 ,
where not all the bi are divisible by p, for otherwise ξ ∈ M . Let j be the
least index such that p ! bj . Then
! "
b0 b1 bj−1 j−1
η = ξ− + α + ··· + α
p p p
bj j bj+1 j+1 bn
= α + α + · · · + αn
p p p
is in OK , since both ξ and
b0 b1 bn
+ α + · · · + αj−1
p p p
are in OK .
If η ∈ OK , then of course ηαn−j−1 is also in OK , and
bj n−1 αn
ηαn−j−1 = α + (bj+1 + bj+2 α + · · · + bn αn−j−2 ).
p p
Since both αn /p and (bj+1 +bj+2 α+· · ·+bn αn−j−2 ) are in OK , we conclude
that (bj αn−1 )/p ∈ OK .
We know from Lemma 4.1.1 that the norm of an algebraic integer is
always a rational integer, so
! "
bj n−1 bnj NK (α)n−1
NK α =
p pn
bnj an−1
0
=
pn
4.3. EXAMPLES 47
must be an integer. But p does not divide bj , and p2 does not divide a0 , so
this is impossible. This proves that we do not have an element of order p,
and thus p ! [OK : M ].
Exercise 4.3.3 Let α be an algebraic integer, and let f (x) be the minimal poly-
n /
nomial of α. If f has degree n, show that dK/Q (α) = (−1)( 2 ) n (i)
i=1 f (α ).
(
√
Example 4.3.4 Let K = Q( D) with D a squarefree integer. Find an
integral basis for OK .
√
Solution. An arbitrary element α of K is of the form α = r1 + r√ 2 D with
r1 , r2 ∈ Q. Since [K : Q] = 2, α has only one conjugate: r1 − r2 D. From
Lemma 4.1.1 we know that if α is an algebraic integer, then TrK (α) = 2r1
and
√ √
NK (α) = (r1 + r2 D)(r1 − r2 D)
= r12 − Dr22
are both integers. We note also that since α satisfies the monic polynomial
x2 − 2r1 x + r12 − Dr22 , if TrK (α) and NK (α) are integers, then α is an
algebraic integer. If 2r1 ∈ Z where r1 ∈ Q, then the denominator of r1 can
be at most 2. We also need r12 −Dr22 to be an integer, so the denominator of
r2 can be no more than 2. Then let r1 = g1 /2, r2 = g2 /2, where g1 , g2 ∈ Z.
The second condition amounts to
g12 − Dg22
∈ Z,
4
which means that g12 − Dg22 ≡ 0 (mod 4), or g12 ≡ Dg22 (mod 4).
We will discuss two cases:
Case 1. D ≡ 1 (mod 4).
If D ≡ 1 (mod 4), and g12 ≡ Dg22 (mod√4), then g1 and g2 are either
both
√ even or both odd. So if α = r1 + r2 D is an algebraic integer of
Q( D), then either r1 and r2 are both integers, or they are both fractions
with denominator 2. √
We recall from Chapter 3 that if 4 | (−D + 1), then
√ (1 + D)/2 is an
algebraic integer. This suggests that we use 1, (1 + D)/2 as a basis; it is
clear from the discussion above that this is in fact an integral basis.
Case 2. D ≡ 2, 3 (mod 4).
If g12 ≡ Dg 2
√ 2 (mod 4), then both g1 and g2 must be even. Then a basis
for OK is 1, D; again it is clear that this is an integral basis.
√
Exercise 4.3.5 √ If D ≡ 1 (mod 4), show that every integer of Q( D) can be
written as (a + b D)/2 where a ≡ b (mod 2).
48 CHAPTER 4. INTEGRAL BASES
α2
OK = Z + Zα + Z .
b
NK (1 + α + α2 ) NK (α3 − 1) (r − 1)2
NK (c) = = =
27 27NK (α − 1) 27
α2
1 = 3c − α − b ,
b
α = 0 + α + 0,
α2
α2 = 0+0+b ,
b
4.4. IDEALS IN OK 49
then Theorem 4.2.2 tells us that the index of Z+Zα+Zα2 in Zα+Zα2 /b+Zc
is 3b. Therefore
α2 1 + α + α2
OK = Zα + Z +Z .
b 3
Case 3. If r ≡ 8 (mod 9), consider d = (1 − α + α2 )/3.
This is an algebraic integer. TrK (d) = 1, NK (d) = (1 + r)2 /27 ∈ Z, and
the remaining coefficient for the minimal polynomial of d is (1 + r)/3 ∈ Z.
By the same reasoning as above, we conclude that 3 | m and so m = 3b.
We choose α, α2 /b, d as an integral basis, noting that
α2
1 = 3d + α − b ,
b
α = 0 + α + 0,
α2
α2 = 0+0+b ,
b
so that the index of Z + Zα + Zα2 in Zα + Zα2 /b + Zd is 3b. We conclude
that
α2 1 − α + α2
OK = Zα + Z +Z .
b b
Exercise 4.3.7 Let ζ be any primitive pth root of unity, and K = Q(ζ). Show
that 1, ζ, . . . , ζ p−2 form an integral basis of K.
4.4 Ideals in OK
At this point, we have shown that OK is indeed much like Z in its algebraic
structure. It turns out that we are only halfway to the final step in our
generalization of an integer in a number field. We may think of the ideals
in OK as the most general integers in K, and we remark that when this
set of ideals is endowed with the usual operations of ideal addition and
multiplication, we recover an arithmetic most like that of Z. We prove now
several properties of the ideals in OK .
Exercise 4.4.3 Show that if a is a nonzero ideal in OK , then a has finite index
in OK .
Exercise 4.4.4 Show that every nonzero prime ideal in OK contains exactly one
integer prime.
Exercise 4.5.2 Let K/Q be an algebraic number field of degree n. Show that
dK ≡ 0 or 1 (mod 4). This is known as Stickelberger’s criterion.
(1 − ζm )ϕ(m) = pOK .
(−1)ϕ(m)/2 mϕ(m)
dK/Q (ζm ) = .
pm/p
ϕ(m)−1
Exercise 4.5.9 Let m = pa , with p prime. Show that {1, ζm , . . . , ζm } is
an integral basis for the ring of integers of K = Q(ζm ).
(−1)ϕ(m)/2 mϕ(m)
dK = .
pm/p
4.5. SUPPLEMENTARY PROBLEMS 51
Exercise 4.5.11 Show that Z[ζn + ζn−1 ] is the ring of integers of Q(ζn + ζn−1 ),
where ζn denotes a primitive nth root of unity, and n = pα .
√
Exercise 4.5.15 Let p and q be distinct primes ≡ 1 (mod 4). Let K = Q( p),
√ √ √
L = Q( q). Find a Z-basis for Q( p, q).
∆ = dK/Q (a1 , . . . , an ).
∆ = dK/Q (a1 , . . . , an ).
For each i, choose the least natural number dii so that for some dij ∈ Z, the
number
,i
wi = ∆−1 dij aj ∈ OK .
j=1
aj = cj1 w1 + · · · + cjj wj ,
cij ∈ Z, j = 1, . . . , n.
Exercise 4.5.21 Show that only finitely many imaginary quadratic fields K are
Euclidean.
√
Exercise 4.5.22 Show that Z[(1 + −19)/2] is not Euclidean. (Recall that in
Exercise 2.5.6 we showed this ring is not Euclidean for the norm map.)
If we set
log |dM |
δ(M ) = ,
[M : Q]
deduce that δ(KL) = δ(K) + δ(L) whenever gcd(dK , dL ) = 1.
Exercise 4.5.25 Let ζm denote a primitive mth root of unity and let K =
Q(ζm ). Show that OK = Z[ζm ] and
(−1)φ(m)/2 mϕ(m)
dK = / φ(m)/(p−1)
.
p|m p
Dedekind Domains
Exercise 5.1.1 Show that a nonzero commutative ring R with identity is a field
if and only if it has no nontrivial ideals.
m is maximal,
⇔ R/m has no nontrivial ideals,
⇔ R/m is a field.
53
54 CHAPTER 5. DEDEKIND DOMAINS
⇔ ab ∈ ℘ ⇒ a ∈ ℘ or b ∈ ℘,
⇔ ab + ℘ = 0 + ℘ in R/℘ ⇒ a + ℘ = 0 + ℘ or b + ℘ = 0 + ℘ in R/℘,
⇔ R/℘ has no zero-divisors,
⇔ R/℘ is an integral domain.
Exercise 5.1.5 Show that every unique factorization domain is integrally closed.
OK [α] = OK + OK α + OK α2 + · · · + OK αn−1
5.2. CHARACTERIZING DEDEKIND DOMAINS 55
Note that this theorem, and its proof, were exactly the same as Theo-
rem 3.3.9, with OK replacing Z.
M = OK u1 + · · · + OK un , αM ⊆ M.
Let OK5= Zv1 +· · ·+Zvm , where {v1 , . . . , vm } is a basis for K over Q. Then
m 5n
M = i=1 j=1 Zvi uj is a finitely generated Z-module with αM ⊆ M , so
α is integral over Z. By definition, α ∈ OK . !
Exercise 5.2.1 If a " b are ideals of OK , show that N (a) > N (b).
56 CHAPTER 5. DEDEKIND DOMAINS
Theorem 5.2.3 For any commutative ring R with identity, the following
are equivalent:
(1) R is Noetherian;
(3) OK is Noetherian.
Exercise 5.2.4 Show that any principal ideal domain is a Dedekind domain.
√
Exercise 5.2.5 Show that Z[ −5] is a Dedekind domain, but not a principal
ideal domain.
5.3. FRACTIONAL IDEALS AND UNIQUE FACTORIZATION 57
Exercise 5.3.2 Show that the sum and product of two fractional ideals are again
fractional ideals.
Proof. Let S be the set of all proper ideals of OK that do not contain a
product of prime ideals. We need to show that S is empty. If not, then
since OK is Noetherian, S has a maximal element, say a. Then, a is not
prime since a ∈ S, so there exist a, b ∈ OK , with ab ∈ a, a ∈ / a, b ∈/ a.
Then, (a, a) $ a, (a, b) $ a. Thus, (a, a) ∈ / S, (a, b) ∈ / S, by the maximality
of a.
Thus, (a, a) ⊇ ℘1 · · · ℘r and (a, b) ⊇ ℘$1 · · · ℘$s , with the ℘i and ℘$j non-
zero prime ideals. But ab ∈ a, so (a, ab) = a.
Thus, a = (a, ab) ⊇ (a, a)(a, b) ⊇ ℘1 · · · ℘r ℘$1 · · · ℘$s . Therefore a contains
a product of prime ideals. This contradicts a being in S, so S must actually
be empty.
Thus, any proper ideal of OK contains a product of nonzero prime ideals.
!
℘−1 = {x ∈ K : x℘ ⊆ OK }.
Proof.
Existence. Let S be the set of ideals of OK that cannot be written as
a product of prime ideals. If S is nonempty, then S has a maximal element,
since OK is Noetherian. Let a be a maximal element of S. Then a ⊆ ℘ for
some maximal ideal ℘, since OK is Noetherian. Recall that every maximal
ideal of OK is prime. Since a ∈ S, a != ℘ and therefore a is not prime.
Consider ℘−1 a. ℘−1 a ⊂ ℘−1 ℘ = OK . Since a # ℘,
℘−1 a # ℘−1 ℘ = OK ,
5.3. FRACTIONAL IDEALS AND UNIQUE FACTORIZATION 59
℘$2 · · · ℘$s = ℘2 · · · ℘r .
Thus, continuing in this way, we see that r = s and the primes are unique
up to reordering. !
When ℘ and ℘$ are prime ideals, we will write ℘/℘$ for (℘$ )−1 ℘. We
will also write
℘1 ℘2 · · · ℘r
℘$1 ℘$2 · · · ℘$s
to mean (℘$1 )−1 (℘$2 )−1 · · · (℘$s )−1 ℘1 ℘2 · · · ℘r .
Exercise 5.3.7 Show that any fractional ideal A can be written uniquely in the
form
℘1 . . . ℘ r
,
℘(1 . . . ℘(s
where the ℘i and ℘(j may be repeated, but no ℘i = ℘(j .
Exercise 5.3.9 Show that if a and b are ideals of OK , then b | a if and only if
there is an ideal c of OK with a = bc.
x ≡ a (mod a),
x ≡ b (mod b).
e
Then x − ai ∈ ℘e11 · · · ℘r−1
r−1
, x ≡ ar (mod ℘err ). Thus, x − ai ∈ ℘ei i ∀i, i.e.,
ei
x ≡ ai (mod ℘i ) ∀i. !
Exercise 5.3.14 Show that ord℘ (ab) = ord℘ (a) + ord℘ (b), where ℘ is a prime
ideal.
N (℘e ) = (N (℘))e
r
/r−1 0
? ?
℘ei i = lcm ℘ei i , ℘err
i=1 i=1
/r−1 0
6
= lcm ℘ei i , ℘err
i=1
r
6
= ℘ei i .
i=1
>r 2r
Thus, ker(φ) = i=1 ℘ei i = i=1 ℘ei i , which implies that
OK /a 1 ⊕(OK /℘ei i ).
2r
Hence, N (a) = i=1 N (℘ei i ).
(b) Since ℘e # ℘e−1 , we can find an element α ∈ ℘e−1 /℘e , so that
ord℘ (α) = e − 1. Then ℘e ⊆ (α) + ℘e ⊆ ℘e−1 . So ℘e−1 | (α) + ℘e . But
62 CHAPTER 5. DEDEKIND DOMAINS
γ ∈ ker(φ) ⇔ γα ∈ ℘e
⇔ ord℘ (γα) ≥ e
⇔ ord℘ (γ) + ord℘ (α) ≥ e
⇔ ord℘ (γ) + e − 1 ≥ e
⇔ ord℘ (γ) ≥ 1
⇔ γ ∈ ℘.
N (a)
N (ab−1 ) = .
N (b)
Let
n
*
mωi∗ = aij ωj ,
j=1
so
n
* aij
ωi∗ = ωj ,
j=1
m
and
n
*
ωi = bij ωj∗ .
j=1
Thus, det(bij ) = dK .
64 CHAPTER 5. DEDEKIND DOMAINS
and thus
aij
|det( )| = N (D−1 ) = N (D)−1 .
m
Hence, |dK | = |det(bij )| = | det(aij /m)|−1 = N (D). !
Proof. We may assume that e is the highest power 5nof ℘ dividing (p). So let
(p) = ℘e a, gcd(a, ℘) = 1. Let x ∈ ℘a. Then x = i=1 pi ai , pi ∈ ℘, ai ∈ a.
Hence,
*n
p
x ≡ ppi api (mod p),
i=1
and
n
* m m
pm
x ≡ ppi api (mod p).
i=1
m m m
For sufficiently large m, ppi ∈ ℘e , so xp ∈ ℘e and thus, xp ∈ ℘e a = (p).
m
Therefore, Tr(xp ) ∈ pZ, which implies that
m
(Tr(x))p ∈ pZ
⇒ Tr(x) ∈ pZ
⇒ Tr(p−1 ℘a) ⊆ Z
⇒ p−1 ℘a ⊆ D−1
⇒ Dp−1 ℘a ⊆ DD−1 = OK
⇒ D ⊆ p℘−1 a−1 = ℘e a℘−1 a−1 = ℘e−1
⇒ ℘e−1 | D. !
5.5 Factorization in OK
The following theorem gives an important connection between factoring
polynomials mod p and factoring ideals in number fields:
Exercise 5.5.3 Suppose that f (x) in the previous exercise is Eisensteinian with
respect to the prime p. Show that p ramifies totally in K. That is, pOK = (θ)n
where n = [K : Q].
Exercise 5.6.3 Find a prime ideal factorization of (2), (5), (11) in Z[i].
√
Exercise 5.6.4 Compute the different D of K = Q( −2).
√
Exercise 5.6.5 Compute the different D of K = Q( −3).
Deduce that
1
D −1 = (Zb0 + · · · + Zbn−1 ).
f ( (α)
Exercise 5.6.12 Let p be prime and let a be squarefree and coprime to p. Set
θ = a1/p and consider K = Q(θ). Show that OK = Z[θ] if and only if ap−1 #≡ 1
(mod p2 ).
Exercise 5.6.14 Let K = Q(θ) and suppose that p | dK/Q (θ), p2 ! dK/Q (θ).
Show that p | dK and p ramifies in K.
Exercise 5.6.18
√ Determine the prime ideal factorization of (7), (29), and (31)
in K = Q( 3 2).
Exercise 5.6.21 Let L/K be a finite extension of algebraic number fields. Show
that the map
T rL/K : L × L → K
is non-degenerate.
68 CHAPTER 5. DEDEKIND DOMAINS
Exercise 5.6.22 Let L/K be a finite extension of algebraic number fields. Let
a be a finitely generated OK -module contained in L. The set
−1
DL/K (a) = {x ∈ L : T rL/K (xa) ⊆ OK }
Exercise 5.6.26 Let L/K be a finite extension of algebraic number fields. Sup-
pose that OL = OK [α] for some α ∈ L. If f (x) is the minimal polynomial of α
over OK , show that DL/K = (f ( (α)).
This chapter mainly discusses the concept of the ideal class group, and some
of its applications to Diophantine equations. We will prove that the ideal
class group of an algebraic number field is finite, and establish some related
results.
As in all other chapters, we shall let K be an algebraic number field
with degree n over Q, and let OK be the ring of algebraic integers in K.
|N (α − β)| < 1.
69
70 CHAPTER 6. THE IDEAL CLASS GROUP
Let t = t1 − t2 , then
1 / 01
1 *n 1
1 1
|N (tα − β)| = 1N ({t1 ci } − {t2 ci })ωi 1 .
1 1
i=1
(j) 2n 5n (j)
where ωi is the jth conjugate of ωi . If we take Ln > j=1 ( i=1 |ωi |) =
HK (say), then
|N (tα − β)| < 1.
Furthermore, since 0 ≤ t1 , t2 ≤ Ln , we have |t| ≤ Ln . Thus, if we choose
1/n
L = HK , we are done. !
Let us call HK as defined above the Hurwitz constant, since the lemma
is due to A. Hurwitz.
Exercise 6.2.1 Show that the relation ∼ defined above is an equivalence rela-
tion.
and we have M a ⊆ (β). This means that (β) divides (M )a, and so
(M )a = (β)b,
for some ideal b ⊆ OK .
Observe that β ∈ a, so M β ∈ (β)b, and hence (M ) ⊆ b. This implies
that |N (b)| ≤ N ((M )) = CK . Hence, a ∼ b, and CK = N ((M )) satisfies
the requirements. !
72 CHAPTER 6. THE IDEAL CLASS GROUP
Exercise 6.2.3 Show that each equivalence class of ideals has an integral ideal
representative.
Exercise 6.2.4 Prove that for any integer x > 0, the number of integral ideals
a ⊆ OK for which N (a) ≤ x is finite.
Exercise 6.2.6 Show that the product defined above is well defined, and that H
together with this product form a group, of which the equivalence class containing
the principal ideals is the identity element.
Theorem 6.2.5 and Exercise 6.2.6 give rise to the notion of class number.
Given an algebraic number field K, we denote by h(K) the cardinality of
the group of equivalence classes of ideals (h(K) = |H|), and call it the class
number of the field K. The group of equivalence classes of ideals is called
the ideal class group.
With the establishment of the ideal class group, the result in Theorem
6.2.2 can be improved as follows:
Exercise 6.2.7 Show that the constant CK in Theorem 6.2.2 could be taken to
be the greatest integer less than or equal to HK , the Hurwitz constant.
and √
(5) = ( −5)2 .
√ √ √
Thus,
√ ℘1 can√ only be (2,√1 + −5), √(2, 1 − −5), (3, 1 + −5), (3, 1 −
−5), (7, 3 + −5), (7, 3 − −5), or ( −5). The same conclusion
√ holds for
any ℘i for i = 2, . . . , m. Moreover, it can be seen that ( −5) is principal,
and all the others are not principal (by taking the norms), but are pairwise
equivalent by the following relations:
√ √
(2, 1 + −5) = (2, 1 − −5),
√ √ √
(3, 1 + −5)(1 − −5) = (3)(2, 1 − −5),
√ √ √
(3, 1 − −5)(1 + −5) = (3)(2, 1 + −5),
√ √ √
(7, 3 + −5)(3 − −5) = (7)(2, 1 − −5),
√ √ √
(7, 3 − −5)(3 + −5) = (7)(2, 1 + −5).
x2 + k = y 3 , (6.2)
74 CHAPTER 6. THE IDEAL CLASS GROUP
which was first introduced by Bachet in 1621, and has played a fundamental
role in the development of number theory. When k = 2, the only integral
solutions to this equation are given by y = 3 (see Exercise 2.4.3); and this
result is due to Fermat. It is known that the equation has no integral
solution for many different values of k. There are various methods for
discussing integral solutions of equation (6.2). We √
shall present, here, the
one that uses applications of the quadratic field Q( −k), and the concept
of ideal class group. This method is usually referred to as Minkowski’s
method. We start with a simple case, when k = 5.
1 = b(3a2 − 5b2 ).
The discussion for many, but by no means all, values of k goes through
without any great change. For instance, one can show that when k = 13
and k = 17, the only integral solutions to equation (6.2) are given by y = 17
and y = 5234, respectively.
We now turn to a more general result.
x2 + k = y 3 . (6.4)
√
Show that if 3 does not divide the class number of Q( −k), then this equation
has no integral solution.
A similar analysis for the real quadratic fields is more difficult and is post-
poned to Exercise 8.3.17 in Chapter 8.
Exercise 6.4.1 Fix a positive integer g > 1. Suppose that n is odd, greater
√ than
1, and ng − 1 = d is squarefree. Show that the ideal class group of Q( −d) has
an element of order g.
Exercise 6.5.4 Show in the previous question if the volume ≥ 2n , the result is
still valid, if C is closed.
Exercise 6.5.5 Show that there exist bounded, symmetric convex domains with
volume < 2n that do not contain a lattice point.
6.5. SUPPLEMENTARY PROBLEMS 77
|Li (x)| ≤ λi , 1 ≤ i ≤ n.
Exercise 6.5.7 Suppose that among the n linear forms above, Li (x), 1 ≤ i ≤ r1
are real (i.e., aij ∈ R), and 2r2 are not real (i.e., some aij may be nonreal).
Further assume that
That is,
n
,
Lr1 +r2 +j (x) = ar1 +j,k xk , 1 ≤ j ≤ r2 .
k=1
|Li (x)| ≤ λi , 1 ≤ i ≤ n,
Exercise 6.5.8 Using the previous result, deduce that if K is an algebraic num-
ber field
8with discriminant dK , then every ideal class contains an ideal b satisfying
N b ≤ |dK |.
(x1 , . . . , xr , y1 , z1 , . . . , ys , zs )
2r−s π s tn
,
n!
where n = r + 2s.
78 CHAPTER 6. THE IDEAL CLASS GROUP
show that there exist rational integers x1 , . . . , xn (not all zero) such that
x1 a1 + · · · + xn an ∈ C.
Exercise 6.5.14 If K and L are algebraic number fields such that dK and dL
are coprime, show that K ∩ L = Q. Deduce that
Exercise 6.5.23 (Hermite) Show that there are only finitely many algebraic
number fields with a given discriminant.
Exercise 6.5.24 Let p be a prime ≡ 11 (mod 12). If p > 3n , show that the
√
ideal class group of Q( −p) has an element of order greater than n.
Quadratic Reciprocity
The equation x2 ≡ a (mod p), where p is some prime, provides the starting
point for our discussion on quadratic reciprocity. We can ask whether there
exist solutions to the above equation. If yes, how do these solutions depend
upon a? upon p? Gauss developed the theory of quadratic reciprocity to
answer these questions. His solution is today called the Law of Quadratic
Reciprocity. Gauss, however, christened his result Theorema Auruem, the
Golden Theorem.
In this chapter, we will be examining this interesting facet of number
theory. We will begin with some of the basic properties of reciprocity. We
will then take a brief trip into the realm of Gauss sums, which will provide
us with the necessary tools to prove the Law of Quadratic Reciprocity.
Finally, once we have developed this Golden Theorem, we will show its
usefulness in the study of quadratic fields, as well as primes in certain
arithmetic progressions.
7.1 Preliminaries
In this section, we would like to search for solutions to equations of the
form x2 ≡ a (mod p), where p is prime. We will discover that quadratic
reciprocity gives us a means to determine if any solution exists.
In order to appreciate the usefulness of quadratic reciprocity, let us
consider how we would tackle the congruence
x2 ≡ −1 (mod 5).
The naive method would be to take all the residue classes in (Z/5Z) and
square them. We would get 02 ≡ 0, 12 ≡ 1, 22 ≡ 4, 32 ≡ 4, and 42 ≡ 1.
Since 4 ≡ −1 (mod 5), we have found two solutions to the above equation,
namely 2 and 3. This brute force method works well for small primes but
becomes impractical once the size of the numbers gets too large. Thus
81
82 CHAPTER 7. QUADRATIC RECIPROCITY
Remark. One of the interesting results of this exercise is that we can now
determine which finite fields Fp , for p prime, have an element that acts like
√
−1. For example, if p = 5, then p ≡ 1 (mod 4), and so, (−1/p) = 1.
So there exists an element a ∈ F5 such that a2 = −1. However, 7 ≡ 3
(mod 4), so F7 can have no element that is the square root of −1.
Before going any further, we will determine some properties of the Leg-
endre symbol.
Exercises 7.1.3 to 7.1.5 give some of the basic properties of the Legen-
dre symbol that we will exploit throughout the remainder of this chapter.
Notice that Exercise 7.1.4 shows us that the product of two residues mod p
is again a residue mod p. As well, the product of two quadratic nonresidues
mod p is a quadratic residue mod p. However, a residue mod p multiplied
by a nonresidue mod p is a nonresidue mod p.
(1 + i)2 = 1 + 2i + i2 = 2i.
So,
Exercise 7.1.7 Show that the number of quadratic residues mod p is equal to
the number of quadratic nonresidues mod p.
&p−1
Exercise 7.1.8 Show that a=1 (a/p) = 0 for any fixed prime p.
* * ! a2 c "
2
S = ζpa(1+c) .
p
(a,p)=1 (c,p)=1
Thus
* !c" *
S2 = ζpa(1+c)
p
(c,p)=1 (a,p)=1
* !c" !
p−1
"
= (−1) + (p − 1)
p p
1≤c≤p−2
* ! c " ! −1 "
= (−1) + (p − 1).
p p
1≤c≤p−2
From Exercise 7.1.8, we know that the first term on the right-hand side
must be equal to 0. So,
* ! c " ! −1 "
2
S = (−1) + (p − 1)
p p
1≤c≤p−2
3 ! "4 ! "
−1 −1
= (−1) − + (p − 1)
p p
! " ! "
−1 −1
= + (p − 1)
p p
! "
−1
= p.
p
E F
But now we have shown the desired result, namely, S 2 = −1
p p. !
But both sides only take on the value ±1, and since q ≥ 3, the congruence
can be replaced by an equals sign. This gives us
! " ! "
p q p−1 q−1
= (−1) 2 · 2 .
q p
!
With this result, we can answer the question we asked at the beginning
of this chapter. That is, if we fix some a, for what primes p will x2 ≡ a
(mod p) have a solution? Expressed in terms of the Legendre symbol, we
want to know for which p will (a/p) = 1. We know from Exercise 7.1.4 that
88 CHAPTER 7. QUADRATIC RECIPROCITY
The next exercise will demonstrate how to use Exercise 7.3.2 to compute
(q/p) in the special cases q = 5, 7.
- . - .
5 7
Exercise 7.3.3 Compute p
and p
.
(3) pOK = ℘.
If (1) is true, we say that p splits. When case (2) occurs, we say that
p ramifies. Finally, if (3) occurs, we say that p is inert, i.e., it stays prime.
In the next exercises, we will see that we can determine which case occurs
by using quadratic reciprocity.
7.4. QUADRATIC FIELDS 89
√
Exercise 7.4.1 Find the discriminant of K = Q( d) when:
(a) d ≡ 2, 3 (mod 4); and
(b) d ≡ 1 (mod 4).
Exercise 7.4.3 Assume that p is an odd prime. Show that (d/p) = 0 if and only
if pOK = ℘2 , where ℘ is prime.
Exercise 7.5.1 Show that there are infinitely many primes of the form 4k + 1.
Exercise 7.5.2 Show that there are infinitely many primes of the form 8k + 7.
The results we have just derived are just a special case of a theorem
proved by Dirichlet. Dirichlet proved that if l and k are coprime inte-
gers, then there must exist an infinite number of primes p such that p ≡ l
(mod k). What is interesting about these two exercises, however, is the fact
that we used a proof similar to Euclid’s proof for the existence of an infinite
92 CHAPTER 7. QUADRATIC RECIPROCITY
Example 7.5.4 Show there are an infinite number of primes in the arith-
metic progession 15k + 4.
f (x) = x4 − x3 + 2x2 + x + 1.
7.5. PRIMES IN SPECIAL PROGRESSIONS 93
Finally, equation (7.4) tells us that (5/p) = 1. But we know this only
happens when p ≡ 1, 4 (mod 5).
When we combine all these results, we find that any prime divisor of
f (x) must be congruent to either 1 (mod 15) or 4 (mod 15).
We can now begin the Euclid-type proof. Suppose that there were only
a finite number of primes such that p ≡ 4 (mod 15). Let p1 , . . . , pn be
these primes. We now consider the integer d = f (15p1 p2 · · · pn + 1). From
what we have just said, d is divisible by some prime p such that p ≡ 1, 4
(mod 5). Not all the prime divisors have the form p ≡ 1 (mod 5). This
follows from the fact that d = f (15p1 · · · pn +1) = 15p1 · · · pn g(p1 · · · pn )+4,
where g(x) is some polynomial. So, there is a divisor p such that p ≡ 4
94 CHAPTER 7. QUADRATIC RECIPROCITY
One item that we did not discuss is how to derive a polynomial that we
can use in a Euclid-style proof. One method involves a little ingenuity and
some luck. By playing around with some equations, you may happen upon
such a polynomial. Murty, on the other hand, describes [Mu] an explicit
construction for these polynomials. Though interesting in their own right,
we will refrain from going into any detail about these polynomials.
Exercise 7.6.3 If p ≡ 1 (mod 3), prove that there are integers a, b such that
p = a2 − ab + b2 .
Exercise 7.6.4 If p ≡ ±1 (mod 8), show that there are integers a, b such that
a2 − 2b2 = ±p.
Exercise 7.6.5 If p ≡ ±1 (mod 5), show that there are integers a, b such that
a2 + ab − b2 = ±p.
Exercise 7.6.9 From Exercises 7.6.7 and 7.6.8, deduce that any prime divisor p
of n8 − n4 + 1 satisfies
$ % $ % $ % $ % $ % $ %
−1 2 −2 3 6 −6
= = = = = = 1.
p p p p p p
Deduce that p ≡ 1 (mod 24). Prove that there are infinitely many primes p ≡ 1
(mod 24).
x2 + y 2 ≡ 1 (mod p),
with 0 < x < p, 0 < y < p, (p an odd prime) is even if and only if p ≡ ±3
(mod 8).
Exercise 7.6.15 (Reciprocity Law for the Jacobi Symbol) Let P and Q
be odd, positive, and coprime. Show that
$ %$ %
P Q P −1 Q−1
= (−1) 2 · 2 .
Q P
Exercise 7.6.16 (The Kronecker Symbol) We can define (a/n) for any in-
teger a ≡ 0 or 1 (mod 4), as follows. Define
-a. $ a % 0 if a ≡ 0 (mod 4),
= = 1 if a ≡ 1 (mod 8),
2 −2
−1 if a ≡ 5 (mod 8).
Exercise 7.6.17 If p is an odd prime show that the least positive quadratic
√
nonresidue is less than p + 1.
(It is a famous conjecture of Vinogradov that the least quadratic non-residue
mod p is O(pε ) for any ε > 0.)
√
Exercise 7.6.21 Compute the class number of Q( 33).
√
Exercise 7.6.22 Compute the class number of Q( 21).
√
Exercise 7.6.23 Show that Q( −11) has class number 1.
√
Exercise 7.6.24 Show that Q( −15) has class number 2.
√
Exercise 7.6.25 Show that Q( −31) has class number 3.
Chapter 8
ε = ζεn1 1 · · · εnr r ,
Exercise 8.1.1 (a) Let K be an algebraic number field. Show that there are
only finitely many roots of unity in K.
(b) Show, similarly, that for any positive constant c, there are only finitely many
α ∈ OK for which |α(i) | ≤ c for all i.
99
100 CHAPTER 8. THE STRUCTURE OF UNITS
polynomials
n
6
fα,h (x) = (x − α(i)h )
i=1
(i)h
cannot all be distinct since |α | = 1. If fα,h (x) is identical with fα,k (x)
where h < k (say), then the roots must coincide. If αh = αk , then α
is a root of unity and we are done. If not, after a suitable relabelling
we may suppose that α(1)h = α(2)k , α(2)h = α(3)k , . . . , α(n−1)h = α(n)k ,
α(n)h = α(1)k . Therefore,
2
α(1)h = α(2)kh
n n−1
= α(3)k hn−2
= · · · = α(1)k
n
Definition.
Let σ1 , . . . , σr1 , σr1 +1 , σ r1 +1 , . . . , σr1 +r2 , σ r1 +r2 be the real and complex
conjugate embeddings of K in C. Let E = {k ∈ Z : 1 ≤ k ≤ r1 + r2 }. For
k ∈ E, set ;
k if k ≤ r1 ,
k=
k + r2 if k > r1 .
If A ⊆ E, set A = {k : k ∈ A}. Note that E∪E = {k ∈ Z : 1 ≤ k ≤ r1 +2r2 }
and that, if E = A ∪ B is a partition of E, then E ∪ E = (A ∪ A) ∪ (B ∪ B)
is a partition of E ∪ E.
5n
Proof. (a) Let δ = max1≤j≤m i=1 |dij |. Then, for
0 != x = (x1 , . . . , xn ) ∈ Zn≥0
the given cube into hm equal subcubes so that each will have side length
2δt/h. Now, for each x = (x1 , . . . , xn ) ∈ [0, t]n ∩ Zn , y = (y1 , . . . , ym ) ∈
[−δt, δt]m which means that there are (t + 1)n such points y ∈ [−δt, δt]m .
Thus, if hm < (t + 1)n , then two of the points must lie in the same subcube.
That is, for some x$ != x$$ , we have that, if y = (x$ − x$$ )∆ = (y1 , . . . , ym ),
then each |yi | ≤ 2δt/h.
Since t > 1 and n/m ≥ 1, (t + 1)n/m > tn/m + 1 so there exists an
integer h with tn/m < h < (t + 1)n/m (in particular, hm < (t + 1)n ). Then
|yi | ≤ 2δt/h < 2δt1−n/m for each i.
(b) Let {ω1 , . . . , ωn } ⊂ OK be linearly independent over Q and suppose
5n 5n (k)
that (x1 , . . . , xn ) ∈ Zn . If α = j=1 xj ωj , we have α(k) = i=1 xi ωi .
Let k1 , . . . , ku be the elements of A with ki = ki and let l1 , . . . , lv be the
elements of A with li != li , so that m = u + 2v. Let
(kj )
ωi for 1 ≤ j ≤ u,
dij = Re ωi(lj ) for u < j ≤ u + v,
(l )
Im ωi j for u + v < j ≤ 2u + v = m,
ε = ζεn1 1 · · · εnr r ,
f : UK → Rr
ε 9→ (log |ε(1) |, . . . , log |ε(r) |).
But, since ε ∈ UK ,
n
6
1 = |NK (ε)| = |ε(i) | = |ε(r1 +r2 ) ||ε(r1 +2r2 ) | = |ε(r1 +r2 ) |2 ,
i=1
(b) If −M < log |ε(i) | < M , for i = 1, . . . , r, then e−M < |ε(i) | < eM
for all i !∈ S = {r1 + r2 , r1 + 2r2 }. But
|NK (ε)| r1 +2(r2 −1)
|ε(r1 +r2 ) |2 = 2 i|
< eM < eM n .
i*∈S |ε
Thus, we have that each |ε(i) | < eM n/2 . By Exercise 8.1.1, there are only
finitely many ε for which this inequality holds. Therefore, any bounded
region in Rm contains only finitely many points of Γ so, by Theorem 8.1.3,
Γ is a lattice of dimension t ≤ r.
By Lemma 8.1.5 (b), we can find for each 1 ≤ i ≤ r, a unit 6i such that
(i) (j)
|6i | > 1 and |6i | < 1 for j != i and 1 ≤ j ≤ r. Let xi be the image of
6i under the map f . We claim that x1 , ..., xr are linearly independent. For
suppose that
c1 x1 + · · · + cr xr = 0,
with the ci ’s not all zero. We may suppose without loss of generality that
c1 > 0 and c1 ≥ cj for 1 ≤ j ≤ r. Then,
r
* r
*
(i) (i)
0= ci log |61 | ≥ c1 log |61 |,
i=1 i=1
so that
r
* (i)
log |61 | ≤ 0.
i=1
Now the product of the conjugates of 61 has absolute value 1. By our choice
(i)
of 61 , we see that |61 | < 1 and we deduce that
r
* (i)
log |61 | > 0,
i=1
√
Exercise 8.1.8 (a) Show that, for any real quadratic field K = Q( d), where d
is a positive squarefree integer, UK 3 Z/2Z×Z. That is, there is a fundamen-
tal unit ε ∈ UK such that UK = {±εk : k ∈ Z}. Conclude that the equation
x2 − dy 2 = 1 (erroneously dubbed Pell’s equation) has infinitely many integer
solutions for d ≡ 2, 3 mod 4 and that the equation x2 − dy 2 = 4 has infinitely
many integer solutions for d ≡ 1 mod 4.
(b) Let d ≡ 2, 3 (mod 4). Let b be the smallest positive
√ integer such that one of
db2 ± 1 is a square, say a2 , a > 0. Then a + b d is a unit. Show that it is the
fundamental
√ √ unit. Using this algorithm, determine the fundamental units of
Q( 2), Q( 3).
√
(c) Devise a similar algorithm to compute the fundamental √ unit in Q( d), for
d ≡ 1 (mod 4). Determine the fundamental unit of Q( 5).
√
Exercise 8.1.9 (a) For an imaginary quadratic field K = Q( −d) (d a positive,
squarefree integer), show that
Z/4Z for d = 1,
UK 3 Z/6Z for d = 3,
Z/2Z otherwise.
(c) Let K = Q(ζp ), p an odd prime. For any unit ε ∈ UK , ε = ζpk u, for
some real unit u ∈ UK ∩ R, k ∈ Z/pZ.
If ε = −ζpk ε, then εp ≡ −εp (mod p). This implies that 2εp ≡ 0 (mod p)
and so εp ≡ 0 (mod p). In other words, εp ∈ (p), a contradiction, since ε
is a unit.
Thus ε/ε = ζpk , for some k. Let r ∈ Z such that 2r ≡ k (mod p) and let
ε1 = ζp−r ε. Then ε1 = ζpr ε = ζpr−k ε = ζp−r ε = ε1 ⇒ ε1 ∈ R and ε = ζpr ε1 .
(d) Let α = ζp + ζp−1 . Then α = 2 cos(2π/p) ∈ R so L = Q(α) ⊆ K ∩ R.
But since [K : K ∩ R] = 2 and ζp2 − αζp + 1 = 0 so that [K : Q(α)] ≤ 2, we
have that L = K ∩ R. Thus, ε ∈ UK , meaning ε = ±ζpk ε1 , for some ε1 ∈ L,
so that
UK = :ζp ; × UL .
√
(e) It remains only to show that Q(ζ5 +ζ5−1 ) = Q( 5). Let α = ζ5 +ζ5−1 .
ζ54 + ζ53 + ζ52 + ζ5 + 1 = 0,
⇒ ζ52 + ζ5 + 1 + ζ5−1 + ζ5−2 = 0,
or α2 +
√ α − 1 = 0. Since α = 2 cos(2π/5) >
√ 0, this implies that α =
(−1 + 5)/2 and we conclude that Q(α) = Q( 5). !
108 CHAPTER 8. THE STRUCTURE OF UNITS
Exercise 8.1.11 Let [K : Q] = 3 and suppose that K has only one real embed-
ding. Then, by Exercise 8.1.9 (c), WK = {±1} implies that UK = {±uk : k ∈ Z},
where u > 1 is the fundamental unit in K.
(a) Let u, ρeiθ , ρe−iθ be the Q-conjugates of u. Show that u = ρ−2 and that
dK/Q (u) = −4 sin2 θ(ρ3 + ρ−3 − 2 cos θ)2 .
(b) Show that |dK/Q (u)| < 4(u3 + u−3 + 6).
(c) Conclude that u3 > d/4 − 6 − u−3 > d/4 − 7, where d = |dK |.
√
3
Exercise 8.1.12 Let α = 2, K = Q(α). Given that dK = −108:
(a) Show that, if u is the fundamental unit in K, u3 > 20.
(b) Show that β = (α − 1)−1 = α2 + α + 1 is a unit, 1 < β < u2 . Conclude that
β = u.
Definition.
1
a0 + ,
1
a1 +
1
a2 + · · · +
1
an−1 +
an
where each ai ∈ R and ai ≥ 0 for 1 ≤ i ≤ n. We use the notation
[a0 , . . . , an ] to denote the above expression.
Exercise 8.2.1 (a) Consider the continued fraction [a0 , . . . , an ]. Define the se-
quences p0 , . . . , pn and q0 , . . . , qn recursively as follows:
p0 = a0 , q0 = 1,
p1 = a0 a1 + 1, q1 = a 1 ,
pk = ak pk−1 + pk−2 , qk = ak qk−1 + qk−2 ,
Exercise 8.2.2 Let {ai }i≥0 be an infinite sequence of integers with ai ≥ 0 for
i ≥ 1 and let Ck = [a0 , . . . , ak ]. Show that the sequence {Ck } converges.
[a0 , a1 , . . . ] = lim Ck .
k→∞
110 CHAPTER 8. THE STRUCTURE OF UNITS
1
ak = [αk ], αk+1 = .
αk − ak
Show that α = [a0 , a1 , . . . ] is a representation of α as a simple continued fraction.
then s ≥ qk+1 .
Proof. (a) Suppose, on the contrary, that 1 ≤ s < qk+1 . For each k ≥ 0,
consider the system of linear equations
pk x + pk+1 y = r,
qk x + qk+1 y = s.
We will show that x and y are nonzero and have opposite signs.
8.2. UNITS IN REAL QUADRATIC FIELDS 111
If x = 0, then
r pk+1
=
s qk+1
and since (pk+1 , qk+1 ) = 1, this implies that qk+1 |s, and so qk+1 ≤ s,
contradicting our hypothesis.
If y = 0, then r = pk x, s = qk x, so that
pk pk+1
<α< ,
qk qk+1
while, if k is odd,
pk+1 pk
<α< .
qk+1 qk
Thus, in either case, qk α − pk and qk+1 α − pk+1 have opposite signs so that
x(qk α − pk ) and y(qk+1 α − pk+1 ) have the same sign.
1
|qk α − pk | ≤ |sα − r| = s|α − r/s| < ,
1 1 2s
1
1 pk 11 1
⇒ 1α − qk 1 < 2sqk .
112 CHAPTER 8. THE STRUCTURE OF UNITS
Exercise 8.2.5√ Let d be a positive integer, not a perfect square. Show that, if
|x2 −dy 2 | <√ d for positive integers x, y, then x/y is a convergent of the continued
fraction of d.
Exercise 8.2.7 Show that the simple continued fraction expansion of a quadratic
irrational α is periodic.
p2k−1 − dqk−1
2
= (−1)k Qk ,
8.2. UNITS IN REAL QUADRATIC FIELDS 113
for all k ≥ 1, where pk /qk is the kth convergent of the continued fraction of α
and Qk is as defined in Exercise 8.2.6.
√
Let n be the period of the continued fraction of d. We can show,
using properties of purely periodic continued fractions, that n is the smallest
positive integer such that Qn = 1 (so that Qj != 0, for 0 < j < n) and that
Qn != −1 for all n. In particular, this implies that (−1)k Qk = ±1 if and
only if n|k. For the sake of brevity, we omit the proof of this fact.
√
Theorem 8.2.9 Let n be the period of the continued fraction of d.
(a) All integer solutions to the equation x2 − dy 2 = ±1 are given by
√ √
x + y d = ±(pn−1 + qn−1 d)l : l ∈ Z,
we have that, in particular, the least positive solution to the given equation
is x1 = pn−1 , y1 = qn−1 . We√will now show √ that all positive solutions
(xm , ym ) are given by xm + ym d√ = (x1 + y1 d)√m , m > 0.
Taking Q-conjugates, xm − ym d = (x1 − y1 d)m
√ √
(xm + ym d)(xm − ym d) = (x21 − dy12 )m = (±1)m = ±1,
114 CHAPTER 8. THE STRUCTURE OF UNITS
or √ √ √
1 < (x1 + y1 d)−κ (X + Y d) < x1 + y1 d.
√ √
But x21 − dy12 = ±1 which implies that (x1 + y1 d)−κ = [±(x1 − y1 d)]κ .
Define the integers s, t such that
√ √ √ √ √
s + t d = (x1 + y1 d)−κ (X + Y d) = ±(x1 − y1 d)κ (X + Y d).
Then
√ √ √ √
s2 − dt2 = [±(x1 − y1 d)κ (X + Y d)][±(x1 + y1 d)κ (X − Y d)]
= X 2 − dY 2 = ±1.
Also, √ √ √
0 < (x1 + y1 d)−1 < (s + t d)−1 < 1 < s + t d.
But this implies that
√ √ √ √
2s = s + t d ± [±(s − t d)] = s + t d ± (s + t d)−1 > 0,
√ √ √
2t d = s + t d ∓ [±(s − t d)] > 0,
Exercise 8.3.2 Determine the units of an imaginary quadratic field from first
principles.
Exercise
√ 8.3.3 Suppose that 22n √
+ 1 = dy 2 with d √
squarefree.
√ Show that 2 +
n
Exercise 8.3.6 Let p be an odd prime > 3 and supose that it does not divide
the class number of Q(ζp ). Show that
xp + y p + z p = 0
is impossible for integers x, y, z such that p ! xyz.
Exercise 8.3.8 Given an ideal a of a quadratic field K, let a( denote the conju-
gate ideal. If K has discriminant d, write
|d| = pα1
1 p2 · · · pt
Exercise 8.3.10 With the notation as in the previous two questions, show that
there is exactly one relation of the form
℘a1 1 · · · ℘at t = ρOK , NK (ρ) > 0,
&t
with ai = 0 or 1, i=1 ai > 0.
Exercise 8.3.13 If a real quadratic field K has odd class number, show that K
has a unit of norm −1.
√ √
Exercise 8.3.14 Show that 15 + 4 14 is the fundamental unit of Q( 14).
√
Exercise 8.3.15 In Chapter 6 we showed that Z[ 14] is a PID√(principal ideal
domain). Assume the following hypothesis: given α, β ∈ Z[ 14], such that
gcd(α, β) √
= 1, there is a prime π ≡ α (mod β) for which the fundamental √unit
ε = 15 + 4 14 generates the coprime residue classes (mod π). Show that Z[ 14]
is Euclidean.
√
It is now known that Z[ 14] is Euclidean and is the main theorem of
the doctoral thesis of Harper [Ha]. The hypothesis of the previous exer-
cise is still unknown however and is true if the Riemann hypothesis holds
for Dedekind zeta functions of number fields (see Chapter 10). The hy-
pothesis in the question should be viewed as a number field version of a
classical conjecture of Artin on primitive roots. Previously the classifica-
tion of Euclidean rings of algebraic integers relied on some number field
generalization of the Artin primitive root conjecture. But recently, Harper
and Murty [HM] have found new techniques which circumvent the need of
such a hypothesis in such questions. No doubt, these techniques will have
further applications.
2g
Exercise 8.3.17 Suppose that n √is odd, n ≥ 5, and that n +1 = d is squarefree.
Show that the class group of Q( d) has an element of order 2g.
Chapter 9
Exercise 9.1.1 If π is a prime of Z[ρ], show that N (π) is a rational prime or the
square of a rational prime.
Exercise 9.1.2 If π ∈ Z[ρ] is such that N (π) = p, a rational prime, show that π
is a prime of Z[ρ].
Exercise 9.1.4 Let π be a prime of Z[ρ]. Show that αN (π)−1 ≡ 1 (mod π) for
all α ∈ Z[ρ] which are coprime to π.
Exercise 9.1.5 Let π be a prime not associated to (1 − ρ). First show that
3 | N (π) − 1. If (α, π) = 1, show that there is a unique integer m = 0, 1, or 2
such that
α(N (π)−1)/3 ≡ ρm (mod π).
(i) (α/π)3 = 0 if π | α;
(ii) α(N (π)−1)/3 ≡ (α/π)3 (mod π) where (α/π)3 is the unique cube root
of unity determined by the previous exercise.
117
118 CHAPTER 9. HIGHER RECIPROCITY LAWS
Exercise 9.1.8 If q ≡ 2 (mod 3), show that χq (α) = χq (α2 ) and χq (n) = 1 if n
is a rational integer coprime to q.
This exercise shows that any rational integer is a cubic residue mod q.
If π is prime in Z[ρ], we say π is primary if π ≡ 2 (mod 3). Therefore if
q ≡ 2 (mod 3), then q is primary in Z[ρ]. If π = a + bρ, then this means
a ≡ 2 (mod 3) and b ≡ 0 (mod 3).
Exercise 9.1.9 Let N (π) = p ≡ 1 (mod 3). Among the associates of π, show
there is a unique one which is primary.
*
ga (χ) = χ(t)ζ at ,
t∈Fp
where ζ = e2πi/p is a primitive pth root of unity. We also write g(χ) for
g1 (χ).
√
Theorem 9.1.10 If χ != χ0 , then |g(χ)| = p.
9.1. CUBIC RECIPROCITY 119
= g(χ).
since this is just the sum of the pth roots of unity. Finally, g0 (χ) = 0 if
χ != χ0 and g0 (χ0 ) = p.
Now, by our first observation,
*
ga (χ)ga (χ) = |g(χ)|2 (p − 1).
a∈Fp
If s != t, the innermost sum is zero, being the sum of all the pth roots of
unity. If s = t, the sum is p. Hence |g(χ)|2 = p. !
We are now ready to prove the cubic reciprocity law. It will be conve-
nient to work in the ring Ω of all algebraic integers.
120 CHAPTER 9. HIGHER RECIPROCITY LAWS
Lemma 9.1.13 Let π be a prime of Z[ρ] such that N (π) = p ≡ 1 (mod 3).
The character χπ introduced above can be viewed as a character of the finite
field Z[ρ]/(π) of p elements. J(χπ , χπ ) = π.
t
*
≡ χ3 (t)ζ 3t (mod 3Ω)
t
*
≡ ζ 3t (mod 3Ω)
t*=0
≡ −1 (mod 3Ω).
* p−2
*
tj = g aj ≡ 0 (mod π)
t a=0
Proof. Let χπ = χ, and consider the Jacobi sum J(χ, . . . , χ) with q terms.
Since 3 | q + 1, we have, by Exercise 9.1.12, g(χ)q+1 = pJ(χ, . . . , χ). By
Exercise 9.1.14, g(χ)3 = pπ so that
g(χ)q+1 = (pπ)(q+1)/3 .
Recall that *
J(χ, . . . , χ) = χ(t1 ) · · · χ(tq ),
χ(q)2 χ(t1 )q = 1
We raise both sides of this congruence to the (q − 1)st power (recalling that
q − 1 ≡ 1 (mod 3)):
2
p(q−2)(q−1)/3 π (q −1)/3
≡ χ(q)q−1 ≡ χ(q) (mod q).
2
Exercise 9.2.1 Show that q ≡ 1 (mod m) and that 1, ζm , ζm m−1
, . . . , ζm are dis-
tinct coset representatives mod ℘.
We can now define the power residue symbol. For α ∈ Z[ζm ], and ℘ a
prime ideal not containing m, define (α/℘)m as:
(i) (α/℘)m = 0 if α ∈ ℘; and
(ii) if α !∈ ℘, (α/℘)m is the unique mth root of unity such that
! "
α
α(N (℘)−1)/m ≡ (mod m)
℘ m
as determined by Exercise 9.2.2.
Exercise 9.2.5 Suppose a and b are ideals coprime to (m). Show that:
(a) (αβ/a)m = (α/a)m (β/a)m ;
(b) (α/ab)m = (α/a)m (β/b)m ; and
(c) if α is prime to a and xm ≡ α (mod a) is solvable in Z[ζm ], then (α/a)m = 1.
Exercise 9.2.6 Show that the converse of (c) in the previous exercise is not
necessarily true.
124 CHAPTER 9. HIGHER RECIPROCITY LAWS
We can now state the Eisenstein reciprocity law: let 7 be an odd prime,
a ∈ Z prime to 7 and let α ∈ Z[ζ) ] be primary. If α and a are coprime, then
EαF EaF
= .
a ) α )
We will now apply this to establish the theorems of Wieferich and Furt-
wangler on Fermat’s Last Theorem: let 7 be an odd prime and suppose
x) + y ) + z ) = 0 for three mutually coprime integers x, y, z with 7 ! xyz.
(This is the so-called first case.) We let ζ = ζ) be a primitive 7th root of
unity and factor the above equation as
Exercise 9.2.9 Show that the ideals (x + ζ i y) are perfect 1th powers.
α = (x + y)*−2 (x + ζy).
Show that:
(a) the ideal (α) is a perfect 1th power.
(b) α ≡ 1 − uλ (mod λ2 ) where u = (x + y)*−2 y.
x* + y * + z * = 0
x* + y * + z * = 0
for some integers x, y, z coprime to 1, then show that p*−1 ≡ 1 (mod 12 ) for every
p | xyz. Deduce that 2*−1 ≡ 1 (mod 12 ).
Exercise 9.3.2 Let a be a nonsquare integer greater than 1. Show that there
are infinitely many primes p such that (a/p) = −1.
Exercise 9.3.3 Suppose that x2 ≡ a (mod p) has a solution for all but finitely
many primes. Show that a is a perfect square.
Exercise 9.3.4 Let K be a quadratic extension of Q. Show that there are in-
finitely many primes which do not split completely in K.
Exercise 9.3.6 Let p ≡ 1 (mod 3). Show that there are integers A and B such
that
4p = A2 + 27B 2 .
A and B are unique up to sign.
Exercise 9.3.7 Let p ≡ 1 (mod 3). Show that x3 ≡ 2 (mod p) has a solution if
and only if p = C 2 + 27D2 for some integers C, D.
x3 − 2y 3 = 23z m
Analytic Methods
#$ 1
%−1
ζ(s) = 1− s ,
p
p
Exercise 10.1.2 Let K be an algebraic number field and OK its ring of integers.
The Dedekind zeta function ζK (s) is defined for Re(s) > 1 as the infinite series
, 1
ζK (s) = ,
a
(N a)s
where the sum is over all ideals of OK . Show that the infinite series is absolutely
convergent for Re(s) > 1.
#$ 1
%−1
ζK (s) = 1− .
℘
(N ℘)s
127
128 CHAPTER 10. ANALYTIC METHODS
*∞
am
m=1
ms
Since @ m+1
dx
m−s − (m + 1)−s = s ,
m xs+1
we get
*M @ M
am A(M ) A(x) dx
s
= s
+ s .
m=1
m M 1 xs+1
For Re(s) > δ, we find
A(M )
lim = 0,
M →∞ M s
since A(x) = O(xδ ). Hence, the partial sums converge for Re(s) > δ and
we have @ ∞
*∞
am A(x) dx
s
= s
m=1
m 1 xs+1
in this half-plane. !
Exercise 10.1.5 Show that (s − 1)ζ(s) can be extended analytically for Re(s) >
0.
we find that *
A(x) = an
n≤x
is equal to the number of lattice points lying in the positive quadrant defined
by the circle a2 + b2 ≤ x. We will call such a lattice point (a, b) internal
if (a + 1)2 + (b + 1)2 ≤ x. Otherwise, we will call it a boundary lattice
point. Let I be the number of internal lattice points, and B the number of
boundary lattice points. Then
π
I≤ x ≤ I + B.
4
Any boundary point (a, b) is contained in the annulus
√ √ √ √
( x − 2)2 ≤ a2 + b2 ≤ ( x + 2)2
and an upper bound for B is provided by the area of the annulus. This is
easily seen to be
√ √ √ √ √
π( x + 2)2 − π( x − 2)2 = O( x).
√
Thus A(x) = πx/4 + O( x). By Theorem 10.1.4, we deduce that
@ ∞ @ ∞
π dx E(x)
ζK (s) = s + s dx,
4 1 x s
1 xs+1
√
where E(x) = O( x), so that the latter integral converges for Re(s) > 12 .
Thus @ ∞
π E(x)
(s − 1)ζK (s) = s + s(s − 1) dx
4 1 xs+1
is analytic for Re(s) > 12 .
Exercise 10.1.9 Show that the number of integers (a, b) with a > 0 satisfying
a2 + Db2 ≤ x is
πx √
√ + O( x).
2 D
√
Exercise 10.1.10 Suppose K = Q( −D) where D > 0 and −D #≡ 1 (mod 4)
and OK has class number 1. Show that (s − 1)ζK (s) extends analytically to
Re(s) > 12 and find
lim (s − 1)ζK (s).
s→1
- .
Exercise 10.2.2 Show that for an odd prime p, ap = 1 + d
p
.
√
Exercise 10.2.3 Let
- d.K be the discriminant of K = Q( d). Show that for all
primes p, ap = 1 + dpK .
Exercise 10.2.6 Let dK be-the .discriminant of the quadratic field K. Show that
there is an n > 0 such that dnK = −1.
and define
*
G(x) = g(n),
n≤x
*
H(x) = h(n).
n≤x
Proof. We have
* *
f (n) = g(δ)h(e)
n≤x δe≤x
* *
= g(δ)h(e) + g(δ)h(e)
δe≤x δe≤x
δ≤y δ>y
* ExF H ExF * I
= g(δ)H h(e) G + − G(y)
δ e
δ≤y e≤ x
y
as desired. !
where
∞ !
* "
dK 1
c= .
δ δ
δ=1
y = x. We get
* * ! dK " J x K * ExF √ √
an = + G − G( x)[ x].
√ δ δ √ δ
n≤x δ≤ x δ< x
Finally,
* ! dK " 1 * ∞ ! "
dK 1 * ! dK " 1
= −
√ δ δ δ δ √ δ δ
δ≤ x δ=1 δ> x
and by Theorem 10.1.4 we see that
∞ !
* "
dK 1
c=
δ δ
δ=1
converges and
* ! dK " 1 !
1
"
=O √ .
√ δ δ x
δ> x
Therefore * √
an = cx + O( x).
n≤x
Exercise 10.3.1 Show that L(s, χ) converges absolutely for Re(s) > 1.
*∞
an
f (s) =
n=1
ns
for Re(s)
5∞> σ0 . (σ0 is called the abscissa of convergence of the (Dirichlet)
series n=1 an n−s .)
*∞
an
f (s) =
n=1
ns
D = {s : |s − σ1 | < δ},
converges absolutely for any s ∈ D. By the lemma, we can write this series
as the double series
∞
* ∞
(σ1 − s)k * an (log n)k
.
k! n=1
n σ1
k=0
Show that cn ≥ 0.
By Exercise 10.3.3, each L(s, χ) extends to an analytic function for Re(s) >
0. By Exercise 10.1.5, ζ(s) (and hence L(s, χ0 )) is analytic for Re(s) > 0,
s != 1. Thus, f (s) is analytic for Re(s) > 0. By Theorem 10.4.2, the
abscissa of convergence of the Dirichlet series
*∞
cn
n=1
ns
√
Exercise 10.5.2 Show that |g(χ)| = m.
Exercise 10.5.4 Let p be prime. Let χ be a character mod p. Show that there
is an a ≤ p1/2 (1 + log p) such that χ(a) =
# 1.
Exercise 10.5.6 Let D be a bounded open set in R2 and let N (x) denote the
number of lattice points in xD. Show that
N (x)
lim = vol(D).
x→∞ x2
Exercise 10.5.9 Let K be a real quadratic field with discriminant dK , and fun-
damental unit ε. Let C be an ideal class of OK . Show that
N (x, C) 2 log ε
lim = √ ,
x→∞ x dK
where N (x, C) denotes the number of integral ideals of norm ≤ x lying in the
class C.
Exercise 10.5.11 Let K be a real quadratic field. Let N (x; K) denote the
number of integral ideals of norm ≤ x. Show that
N (x; K) 2h log ε
lim = 8 ,
x→∞ x |dK |
where h is the class number of K.
ψ(x) = x + O(x1/2+ε ),
Density Theorems
Given an algebraic number field K, we may ask how the ideals are dis-
tributed in the ideal classes. We may ask the same of the distribution of
prime ideals. It turns out that in both cases, they are equidistributed in the
sense of probability. For many reasons, it has been customary to view the
latter set of results as generalizations of the celebrated theorem of Dirichlet
about primes in arithmetic progressions.
where the summation is over all ideal classes of OK . Let us fix an ideal b
in C −1 and note that if a is an ideal in C with norm ≤ x, then ab = (α)
with α ∈ b and |N (α)| ≤ xN (b). Conversely, if α ∈ b and |N (α)| ≤ xN (b),
then a = (α)b−1 is an integral ideal in C with norm ≤ x. Thus, N (x, C)
is the number of principal ideals (α) contained in b with norm less than or
equal to xN (b).
If β1 , ..., βn is an integral basis of b, then we may write
α = x1 β1 + · · · + xn βn
for some integers x1 , ..., xn . Thus, N (x, C) is the number of such α’s (up to
associates), with |N (α)| ≤ xN (b). We will now try to extract a single ele-
ment from the set of such associates by means of inequalities. Let 61 , ..., 6r
be a system of fundamental units (with r = r1 + r2 − 1 as in Theorem
8.1.6). Recall that it is customary (as we did in Chapter 8) to order our
139
140 CHAPTER 11. DENSITY THEOREMS
Following Hecke [He], we will call the cj ’s the exponents of α. We now want
to show that this equation also holds for i = r + 1. Setting ei = 1 if K (i) is
real, and ei = 2 if K (i) is non-real, we see that
r+1
* E F
ei log |α(i) ||N (α)|−1/n = 0,
i=1
because
|α(1) · · · α(n) | = |N (α)|.
Also,
r+1
* (i)
ei log |6j | = 0.
i=1
Consequently,
r
* E F
(r+1)
cj log |6j | = log |α(r+1) ||N (α)|−1/n ,
j=1
ζ6n1 1 · · · 6nr r
c1 + n1 , ..., cr + nr .
α = x1 β1 + · · · + xn βn ;
11.1. COUNTING IDEALS IN A FIXED IDEAL CLASS 141
and
N (α) = α(1) · · · α(n) ,
then Bx is the set of n-tuples (x1 , ..., xn ) ∈ Rn satisfying
Exercise 11.1.3 Show that N (x, C) = O(x). Deduce that N (x; K) = O(x).
To see this, we may associate each lattice point lying inside Bx with an ap-
propriate translate of the standard unit cube, namely [0, 1]n . Each translate
lying entirely within Bx contributes 1 to the volume of Bx . The error term
arises from the cubes intersecting with the boundary. In view of Exer-
cise 11.1.2, it is intuitively clear (see also Exercise 11.1.12 below) that by
enlarging the region by some fixed quantity and reducing the region by a
fixed quantity δ in the way indicated, the above inequalities are assured.
Thus,
(x1/n − δ)n vol(B1 ) ≤ wN (x, C) ≤ (x1/n + δ)n vol(B1 )
so that
1
wN (x, C) = vol(B1 )x + O(x1− n ).
The essential feature of the theorem below is that this volume is indepen-
dent of the ideal class under consideration.
2r1 (2π)r2 RK
vol(B1 ) = L .
|dK |
(i)
Proof. Let M be the maximal value of | log |6j || for j = 1, ..., r. We first
complete the domain Bx by adding the points of the space lying in the
subspace α(i) = 0 for some i and that also satisfy the inequalities
and
0 < |α(1) · · · α(n) | ≤ N (b),
so that there exist cj ’s for 1 ≤ j ≤ r satisfying 0 ≤ cj < 1 and
E F *r
(i)
log |α(i) ||N (α)|−1/n = cj log |6j |, 1≤i≤n
j=1
11.1. COUNTING IDEALS IN A FIXED IDEAL CLASS 143
or
|α(i) | ≤ erM (N (b))1/n , 1≤i≤n
and at least one α(i) = 0. As noted, the region defined by these latter
conditions are manifolds of lower dimension and thus make no contribution
to the n-fold integral and thus, these conditions may be omitted in the
evaluation of J. To evaluate the integral, we change variables:
n
* (i)
ui := α(i) = yj βj , 1 ≤ i ≤ r1 ,
j=1
n
*
√ (i)
ui + ui+r2 −1 := yj βj , r1 + 1 ≤ i ≤ r1 + r2 .
j=1
n
/ (i) (i+r2 )
0
* βj − βj
ui+r2 = yj √ ,
j=1
2 −1
for r1 + 1 ≤ i ≤ r1 + r2 . The absolute value of the Jacobian for this change
of variables is easily computed to be
L
2−r2 N (b) |dK |.
Hence, @ @
2r2
vol(B1∗ ) = L ··· du1 · · · dun ,
N (b) |dK | B̃1∗
where B̃1∗ is the image of B1∗ under the change of variables. The variables
u1 , ..., ur1 may take one of two signs and so if we insist ui ≥ 0 for i = 1, ..., r1 ,
we must multiply our volume (with this additional constraint) by a factor
of 2r1 . We now shift to polar co-ordinates. Put
ρj = uj 1 ≤ j ≤ r1
and
ρj cos θj = uj , ρj sin θj = uj+r2 , r1 + 1 ≤ j ≤ r1 + r2 ,
with 0 ≤ θj < 2π and ρj ≥ 0. The Jacobian of this transformation is easily
seen to be
ρr1 +1 · · · ρr1 +r2 .
Thus,
@ @
2r1 +r2 (2π)r2
vol(B1∗ ) = L ··· ρr1 +1 · · · ρr1 +r2 dρ1 · · · dρr1 +r2
N (b) |dK | C1∗
144 CHAPTER 11. DENSITY THEOREMS
−1/n
r
6 r
*
e (i)
log ρi ρj j = cj log |6j |
j=1 j=1
2−r2 ρ−1 −1
r1 +1 · · · ρr1 +r2
2r1 (2π)r2 RK 1
N (x, C) = L x + O(x1− n ).
w |dK |
11.1. COUNTING IDEALS IN A FIXED IDEAL CLASS 145
2r1 (2π)r2 hK RK 1
N (x; K) = L x + O(x1− n ),
w |dK |
Note that the Dedekind zeta function defined in the previous chapter may
now be written as *
ζK (s) = ζ(s, C)
C
1
Exercise 11.1.6 Prove that ζ(s, C) extends to the region 5(s) > 1 − n
except
for a simple pole at s = 1 with residue
2r1 (2π)r2 RK
8 .
w |dK |
Deduce that ζK (s) extends to 5(s) > 1 − n1 except for a simple pole at s = 1
with residue
2r1 (2π)r2 hK RK
ρK := 8 ,
w |dK |
where hK denotes the class number of K. (This is usually called the analytic
class number formula.)
Exercise 11.1.7 Prove that there are infinitely many prime ideals ℘ in OK which
are of degree 1.
Exercise 11.1.8 Prove that the number of prime ideals ℘ of degree ≥ 2 and
with norm ≤ x is O(x1/2 log x).
unless a = OK .
146 CHAPTER 11. DENSITY THEOREMS
A(x)
lim =1
x→∞ B(x)
Let t = x1/n . Show that there is a δ > 0 such that for each lattice point P
contained in V(t−δ)n , all the points contained in the translate of the standard
cube by P belong to Vx .
Let H be the ideal class group of K. Following Hecke, we define for each
character
χ : H → C∗
the Hecke L-function
* χ(a)
L(s, χ) := ,
a
N (a)s
where χ(a) is simply χ(C) if a belongs to the ideal class C. If χ is the trivial
character χ0 , note that L(s, χ0 ) = ζK (s), the Dedekind zeta function. Since
H is a finite abelian group of order hK , its character group is also finite of
order hK and so, in this way, we have attached hK L-functions to K.
Exercise 11.2.1 Show that L(s, χ) converges absolutely for 5(s) > 1 and that
#$ χ(℘)
%−1
L(s, χ) = 1− ,
℘
N (℘)s
ξK (s) = ξK (1 − s),
where /L 0s
|dK |
ξK (s) := Γ(s/2)r1 Γ(s)r2 ζK (s)
2r2 π n/2
with Γ(s) denoting the Γ-function. Recall that this is defined by
@ ∞
Γ(s) = e−t ts−1 dt
0
for <(s) > 0 and can be extended meromorphically to the entire complex
plane via the functional equation
Γ(s + 1) = sΓ(s).
148 CHAPTER 11. DENSITY THEOREMS
Our goal is to show that each ideal class contains infinitely many prime
ideals. This is analogous to Dirichlet’s theorem about primes in arithmetic
progressions. Indeed, as we will indicate later, the result is more than an
analogue. It is a generalization that includes the celebrated theorem of
Dirichlet.
Exercise 11.2.5 Let C be an ideal class of OK . For 5(s) > 1, show that
, , 1
χ(C) log L(s, χ) = hK
χ ℘m ∈C
mN (℘)ms
where the first summation is over the characters of the ideal class group and the
second summation is over all prime ideals ℘ of OK and natural numbers m such
that ℘m ∈ C.
Clearly, any set of prime ideals with a positive Dirichlet density is infinite.
Exercise 11.2.8 Let C be a fixed ideal class in OK . Show that the set of prime
ideals ℘ ∈ C has Dirichlet density 1/hK .
Exercise 11.2.9 Let m be a natural number and (a, m) = 1. Show that the set
of primes p ≡ a(mod m) has Dirichlet density 1/φ(m).
Exercise 11.2.10 Show that the set of primes p which can be written as a2 +5b2
is 1/4.
as x tends to infinity.
It is possible to go further. Let f0 be an ideal of OK and f∞ a subset
of real embeddings of K. We write formally f = f0 f∞ and define the f-ideal
class group as follows. We define an equivalence relation on the set of ideals
coprime to f0 by declaring that two ideals a and b are equivalent if
(α)a = (β)b
Exercise 11.2.11 Show that if K = Q, the principal ray class group mod m is
isomorphic to (Z/mZ)∗ .
Exercise 11.3.1 Show that the action of the Galois group on the set of prime
ideals lying above a fixed prime of k is a transitive action.
σ(x) ≡ x(mod ℘) ∀x ∈ OK .
x 9→ xN (p) .
pOK = ℘1 · · · ℘n
where n = [K : k]. This is equivalent to the assertion that the Artin symbol
σp is equal to 1. Thus, from Chebotarev’s density theorem, we immediately
deduce:
Exercise 11.3.8 Let q be prime. Show that the set of primes p for which p ≡ 1
(mod q) and
p−1
2 q ≡ 1(mod p),
has Dirichlet density 1/q(q − 1).
ρ : G → GL(V )
where the product is over all prime ideals p of k and ℘ is any prime ideal of
K lying above p, which is well-defined modulo the inertia group I℘ . Thus
taking the characteristic polynomial of ρ(σ℘ ) acting on the subspace V I℘ ,
which is the subspace of V fixed by I℘ , we get a well-defined factor for each
prime ideal p. The product over all prime ideals p is easily seen to converge
absolutely for <(s) > 1 (why?). As these L-functions play a central role
in number theory, we will briefly give a description of results pertaining to
them and indicate some of the open problems of the area. The reader may
find it useful to have some basic knowledge of the character theory of finite
groups as explained for instance in [Se].
The celebrated Artin’s conjecture predicts that if ρ is a non-trivial irre-
ducible representation, then L(s, ρ; K/k) extends to an entire function of
s. If ρ is one-dimensional, then Artin proved his famous reciprocity law by
showing that in this case, his L-function coincides with Hecke’s L-function
attached to a suitable generalized ideal class group of k. This theorem is
so-called since it entails all of the classical reciprocity laws including the
law of quadratic reciprocity. Subsequently, R. Brauer proved that for any ρ
L(s, ρ; K/k) extends to a meromorphic function for all s ∈ C. He did this by
proving an induction theorem which is really a statement about irreducible
11.4. SUPPLEMENTARY PROBLEMS 153
and that
L(s, IndG
H ψ, K/k) = L(s, ψ; K/K )
H
Exercise 11.4.1 Let G be a finite group and for each subgroup H of G and each
irreducible character ψ of H define aH (ψ, χ) by
,
IndG
H ψ = aH (ψ, χ)χ
χ
Exercise 11.4.3 Deduce from the previous exercise that some positive integer
power of the Artin L-function L(s, χ; K/k) attached to an irreducible character
χ admits a meromorphic continuation to 5(s) = 1.
Exercise 11.4.5 Fix a complex number s0 ∈ C with 5(s0 ) ≥ 1 and any finite
Galois extension K/k with Galois group G. For each subgroup H of G define the
Heilbronn character θH by
,
θH (g) = n(H, χ)χ(g)
χ
where the summation is over all irreducible characters χ of H and n(H, χ) is the
order of the pole of L(s, χ; K/K H ) at s = s0 . By Exercise 11.4.3, the order is a
rational number. Show that θG |H = θH .
Exercise 11.4.6 Show that θG (1) equals the order at s = s0 of the Dedekind
zeta function ζK (s).
L(s, χ; K/k)
Exercise 11.4.11 Show that ζK (s)/ζk (s) is entire. (This is called the Brauer-
Aramata theorem.)
Exercise 11.4.12 (Stark) Let K/k be a finite Galois extension of algebraic num-
ber fields. If ζK (s) has a simple zero at s = s0 , then L(s, χ; K/k) is analytic at
s = s0 for every irreducible character χ of Gal(K/k).
Solutions
Chapter 1
Elementary Number
Theory
1.1 Integers
Exercise 1.1.7 Show that
1 1 1
1+ + + ··· +
3 5 2n − 1
Each of the numbers on the right side of this equation is an integer, except
for m/3k . If m/3k were an integer, then there would be some integer b such
that m = 3k b, but 3k does not divide 3, 5, . . . , 3k − 2, 3k + 2, . . . , 2n − 1
so it cannot divide their least common multiple. Therefore mS is not an
integer, and clearly neither is S.
159
160 CHAPTER 1. ELEMENTARY NUMBER THEORY
Exercise 1.1.9 Prove that if n is a composite integer, then n has a prime factor
√
not exceeding n.
Exercise 1.1.10 Show that if the smallest prime factor p of the positive integer
√
n exceeds 3 n, then n/p must be prime or 1.
!Exercise
" 1.1.11 Let p be prime. Show that each of the binomial coefficients
p
k
, 1 ≤ k ≤ p − 1, is divisible by p.
Exercise 1.1.12 Prove that if p is an odd prime, then 2p−1 ≡ 1 (mod p).
1.1. INTEGERS 161
Solution.
p−1 ! "
* p
2 = (1 + 1)
p p
= 1+ +1
k
k=1
≡ 1+1 (mod p)
3p ≡ 3 (mod p).
2≡1 (mod n0 )
by interpreting the left-hand side as the probability that a random number chosen
from 1 ≤ a ≤ n is coprime to n.
Solution. We find the residue class that 31000 belongs to in Z/100Z. This
is the same as finding the last two digits. By Euler’s theorem, 340 ≡ 1
(mod 100), since
Therefore,
31000 = (340 )25 ≡ 1 (mod 100).
The last two digits are 01.
pk+1 ≤ p1 p2 · · · pk + 1.
2 < k ≤ n.
Then
pn+1 ≤ p1 p2 · · · p n + 1
1 2
< 22 22 · · · 22 + 1
n
= 22 −2 + 1
n+1
< 22
n+1
.
n−1 n
For x > 2, choose an integer n so that ee < x ≤ ee . Then
log x ≤ en log e = en
and
log(log x) ≤ n log e = n.
164 CHAPTER 1. ELEMENTARY NUMBER THEORY
≥ π(22 )
n
≥ n
≥ log(log x).
Exercise 1.1.24 By observing that any natural number can be written as sr2
with s squarefree, show that √
x ≤ 2π(x) .
Deduce that
log x
π(x) ≥ .
2 log 2
Solution. For any set of primes S define fS (x) to be the number of integers
n such that 1 ≤ n ≤ x with γ(n) ⊂ S where γ(n) is the set of primes
dividing n. Suppose that S is a finite set with t elements. Write such an
n in√the form n = r2 s with s squarefree. Since 1 ≤ r2 s ≤ x, we see that
r ≤ x and there are at most 2t choices for s corresponding
√ to the various
subsets of S since s is squarefree. Thus fS (x) ≤ 2t x.
Put π(x) = m so that pm+1 > x. If S = {p1 , . . . , pm }, then fS (x) = x.
Then √ √
x ≤ 2m x = 2π(x) x.
√
Thus x ≤ 2π(x) and hence 12 log x ≤ π(x) log 2, or equivalently,
log x
π(x) ≥ .
2 log 2
&
Exercise 1.1.25 Let ψ(x) = pα ≤x log p where the summation is over prime
powers pα ≤ x.
(i) For 0 ≤ x ≤ 1, show that x(1 − x) ≤ 14 . Deduce that
' 1
1
xn (1 − x)n dx ≤ n
0 4
for every natural number n.
(1 n
(ii) Show that eψ(2n+1) 0
x (1−x)n dx is a positive integer. Deduce that ψ(2n+
1) ≥ 2n log 2.
(iii) Prove that ψ(x) ≥ 12 x log 2 for x ≥ 6. Deduce that
x log 2
π(x) ≥
2 log x
for x ≥ 6.
1.1. INTEGERS 165
For x ≥ 6, x − 3 > x/2 so that ψ(x) > x log 2/2. Since ψ(x) ≤ π(x) log x,
we deduce that
x log 2
π(x) ≥
2 log x
for x ≥ 6.
show that
9x log 2
π(x) ≤
log x
for every integer x ≥ 2.
we deduce that *
log p ≤ 2n log 2
n<p≤2n
because ! "
2n
≤ 22n .
n
Therefore
θ(2n) − θ(n) ≤ 2n log 2,
where *
θ(n) = log p.
p≤n
An easy induction shows that θ(2r ) ≤ 2r+1 log 2 for every positive integer
r. given an integer x ≥ 2, determine r so that
2r ≤ x < 2r+1 .
166 CHAPTER 1. ELEMENTARY NUMBER THEORY
Then
θ(x) ≤ θ(2r+1 ) ≤ 2r+2 log 2 ≤ 4x log 2.
We deduce, in particular,
*
log p ≤ 4x log 2,
√
x<p≤x
so that #1 $# √ $
2 log x π(x) − π( x) ≤ 4x log 2.
This means
√ 8x log 2
π(x) − π( x) ≤
log x
and
8x log 2 √ 9x log 2
π(x) ≤ + x≤
log x log x
because
√ x log 2
x≤
log x
for x ≥ 10, as is easily checked by examining the graph of
√
f (x) = x log 2 − log x.
= c2
= p2γ 1 2γr
1 · · · pr · q12θ1 · · · qs2θs ,
Solution. Assume that x and y are odd. Then both x2 ≡ 1 (mod 4) and
y 2 ≡ 1 (mod 4). Hence z 2 ≡ 2 (mod 4). But there is no z ∈ Z satisfying
z 2 ≡ 2 (mod 4), so one of x or y is even.
Without loss of generality, suppose x is even and y is odd. Then z is
odd. We have x2 = z 2 − y 2 , so
x2 z2 − y2
= ,
4 4
E x F2 (z + y) (z − y)
⇒ = .
2 2 2
Since (x, y) = (y, z) = (x, z) = 1, we see that ((z + y)/2, (z − y)/2) = 1. By
Exercise 1.2.1, there exist a, b ∈ Z such that (z + y)/2 = a2 and (z − y)/2 =
b2 . Hence we have the two equations z + y = 2a2 and z − y = 2b2 .
Thus the solution is x = 2ab, y = a2 −b2 , and z = a2 +b2 where (a, b) = 1
and a and b have opposite parity since y and z are odd. Conversely, any
such triple (x, y, z) gives rise to a solution.
x2 = 2ab, (1.1)
2 2 2
y = b −a , (1.2)
z = b2 + a2 , (1.3)
y 2 = b2 − a2 ≡ −1 ≡ 3 (mod 4).
Exercise 1.2.5 Prove that if f (x) ∈ Z[x], then f (x) ≡ 0 (mod p) is solvable for
infinitely many primes p.
Exercise 1.2.6 Let q be prime. Show that there are infinitely many primes p so
that p ≡ 1 (mod q).
Exercise 1.2.7 Show that Fn divides Fm − 2 if n is less than m, and from this
deduce that Fn and Fm are relatively prime if m #= n.
=
22 n + 1
t2 − 1
k
= t2 −1 − t2 −2 + · · · − 1,
k k
=
t+1
n
Exercise 1.2.8 Consider the nth Fermat number Fn = 22 +1. Prove that every
prime divisor of Fn is of the form 2n+1 k + 1.
Exercise 1.3.3 Assuming the ABC Conjecture, show that there are infinitely
many primes p such that 2p−1 #≡ 1 (mod p2 ).
Exercise 1.3.5 Show that if the Erdös conjecture above is true, then there are
infinitely many primes p such that 2p−1 #≡ 1 (mod p2 ).
2
Solution. Suppose for p > p0 that 2p−1 ≡ 1 (mod p2 ). Let t = p≤p0 p.
Then 6
φ(t) = (p − 1).
p≤p0
Exercise 1.3.6 Assuming the ABC Conjecture, prove that there are only finitely
many n such that n − 1, n, n + 1 are squarefull.
Exercise 1.3.7 Suppose that a and b are odd positive integers satisfying
rad(an − 2) = rad(bn − 2)
for every natural number n. Assuming ABC, prove that a = b. (This problem is
due to H. Kisilevsky.)
Solution. Suppose without loss that a < b. Hence log b > log a so we can
choose ε > 0 so that log b > (1 + ε) log a. Now apply the ABC Conjecture
to the equation (bn − 2) + 2 = bn . Then
# $1+ε
bn ≤ κ(ε) 2b rad(bn − 2)
# $1+ε
≤ κ(ε) 2b rad(an − 2)
# $1+ε
≤ κ(ε) 2ban .
rad(an − 2) = rad(bn − 2)
Solution. Suppose there is an ideal I for which this is not true. Then
show that there exist elements a, b ∈ I such that gcd(a, b) = 1.
Exercise 1.4.3 Prove that if the number of prime Fermat numbers is finite, then
the number of primes of the form 2n + 1 is finite.
174 CHAPTER 1. ELEMENTARY NUMBER THEORY
Exercise 1.4.7 Show that there are no integer solutions to the equation x4 −y 4 =
2z 2 .
(x2 + y 2 )(x2 − y 2 ) = 2z 2 .
x2 + y 2 = 2a2 ,
x2 − y 2 = 4b2 ,
It is easy to see that the two factors are coprime, and so we can write
x+y = 2c2 ,
x−y = 2d2 .
Exercise 1.4.8 Let p be an odd prime number. Show that the numerator of
1 1 1
1+ + + ··· +
2 3 p−1
is divisible by p.
Exercise 1.4.9 Let p be an odd prime number greater than 3. Show that the
numerator of
1 1 1
1 + + + ··· +
2 3 p−1
is divisible by p2 .
Solution. Pair up 1/i and 1/(p − i) and consider the sum mod p.
Exercise 1.4.10 (Wilson’s Theorem) Show that n > 1 is prime if and only
if n divides (n − 1)! + 1.
Exercise 1.4.11 For each n > 1, let Q be the product of all numbers a < n
which are coprime to n. Show that Q ≡ ±1 (mod n).
Exercise 1.4.13 Use Exercises 1.2.7 and 1.2.8 to show that there are infinitely
many primes ≡ 1 (mod 2r ) for any given r.
Exercise 1.4.17 Assuming ABC, show that there are only finitely many con-
secutive cubefull numbers.
Solution. Clearly,
* 1 6! 1
"−1 6 !
1 1
"
≤ 1− = 1 + + 2 + ···
n p p p
n≤x p≤x p≤x
&
(b) Let T (x) = n≤x ψ(x/n), where ψ(x) is defined as in Exercise 1.1.25. Show
that
T (x) = x log x − x + O(log x).
(c) Show that
-x. , -x.
T (x) − 2T = (−1)n−1 ψ = (log 2)x + O(log x).
2 n
n≤x
(a) is immediate.
178 CHAPTER 1. ELEMENTARY NUMBER THEORY
By comparing areas,
* @ x
log n = (log t) dt + O(log x)
n≤x 1
implies (b).
The first part of (c) is now clear. Since ψ(x/n) is a decreasing function
of n, we apply (a) to get
ExF ExF
ψ(x) − ψ +ψ ≥ (log 2)x + O(log x).
2 3
By Exercise 1.1.14, ψ(x) ≤ 2x log 2. Therefore,
ExF
ψ(x) − ψ ≥ 13 (log 2)x + O(log x).
2
Hence, there is a prime between x/2 and x for x sufficiently large.
(This simple proof is due to S. Ramanujan. We can deduce ψ(x) ≤
2x log 2 directly from (a) and (b) without using the solution to Exercise
1.1.26.)
Chapter 2
Euclidean Rings
2.1 Preliminaries
√
Exercise 2.1.2 Let D be squarefree. Consider R = Z[ D]. Show that every
element of R can be written as a product of irreducible elements.
√
Solution. We define a map n : R → N such that for a + b D ∈ R,
√
n(a + b D) = |a2 − Db2 |.
We must check that this map satisfies conditions (i) and (ii) from the pre-
vious example. √ √
(i) For a + b D, c + d D ∈ R,
√ √ √
n[(a + b D)(c + d D)] = n[(ac + bdD) + (ad + bc) D]
= |(ac + bdD)2 − (ad + bc)2 D|
= |(a2 − b2 D)(c2 − d2 D)|
√ √
= n(a + b D)n(c + d D),
√ √ √
Exercise 2.1.3 Let R = Z[ −5]. Show that 2, 3, 1 + −5, and 1 − −5 are
irreducible in R, and that they are not associates.
179
180 CHAPTER 2. EUCLIDEAN RINGS
√
Solution. We define a map n : R → N such that n(a + b −5) = a2 + 5b2 .
If 2 is not irreducible, then there are elements r, s ∈ R such that rs = 2,
with r, s not units. But then n(r)n(s) = n(2) = 4, and since r, s are not
units, it must be that n(r) = n(s) = 2. Then we must find integers a, b
such that a2 +5b2 = 2, which is clearly impossible, so 2 must be irreducible.
If 3 is not irreducible then we can find r, s ∈ R with rs = 3, and r, s
not units. Since n(3) = 9, we must have n(r) = n(s) = 3. But by the same
argument √ as above, we see that this is impossible.
n(1√+ −5) = 6. The only proper divisors of 6 are 2 and 3, and so
if 1 + √ −5 is not irreducible, then we can find r ∈ R, r not a unit and
r | (1 + −5) with either n(r) √ = 2 or n(r) = 3. But we showed √ above that
this √
is not possible, so 1 + −5 is irreducible. Since n(1 − −5) = 6, then
1 − −5 must also be irreducible.
If two elements of R are associates, then they must have the same norm,
a fact which√follows immediately from the condition that all units have norm
1. If a + b −5 is a unit, then a2 +√5b2 = 1. This will only occur √ when
a = ±1, and so the only units of Z[ −5] are 1 and −1. Of 2, 3, 1 ± √−5,
we see that√ the only two which could possibly be associates are 1 + −5
and √1 − −5 because they have the√same norm. However, if√we multiply
1 + −5 by either of the units of Z[ −5], we will not get 1 − −5, and so
they cannot be associates. √ √
We conclude that 2, 3, 1 + −5 and 1 − −5 are all irreducible and are
not associates.
Exercise 2.1.4 Let R be a domain satisfying (i) above. Show that (ii) is equiv-
alent to (ii$ ): if π is irreducible and π divides ab, then π | a or π | b.
a = τ1 τ 2 · · · τ r ,
b = γ1 γ2 · · · γs ,
and τi , γj are irreducible.
We know that π | ab = τ1 · · · τr γ1 · · · γs , so it follows that ab = πλ1 · · · λn
where each λi is irreducible. By condition (ii), π ∼ τi for some i, or π ∼ γj
for some j. Thus, if π | ab, then π | a or π | b.
Now suppose that R is a domain satisfying conditions (i) and (ii0 ) above,
and suppose that we have an element a which has two different factoriza-
tions into irreducibles: a = τ1 · · · τr and a = π1 · · · πs . Consider τ1 . We
know that τ1 | a, and so τ1 | π1 · · · πs . By (ii0 ) we know that τ1 | πi for
some i, and since both are irreducible, they must be associates.
We can now remove both τ1 and πi from our factorization of a. We
next consider τ2 . Following the same process, we can pair up τ2 with its
associate, and we can continue to do this until we have paired up each of
2.2. GAUSSIAN INTEGERS 181
Exercise 2.1.8 If F is a field, prove that F [x], the ring of polynomials in x with
coefficients in F , is Euclidean.
h(x) = an b−1
m g(x)x
n−m
.
α (a + bi)
=
γ (c + di)
(ac + bd) (bc − ad)
= + 2 i
(c2 + d2 ) (c + d2 )
= r + si,
We have shown that our map φ satisfies the properties specified above, and
so Z[i] is Euclidean.
k(p − k) ≡ −k 2 (mod p)
implies ! "
(p−1)/2 p−1 2
(−1) ! ≡ −1 (mod p).
2
Thus, if p ≡ 1 (mod 4), there is an x ∈ Fp so that x2 ≡ −1 (mod p).
The converse follows from Fermat’s little Theorem:
y + i = a3 + 3a2 bi + 3ab2 − b3 i.
a2 + b2 = 4x2 + 4y 2 + 4y + 1
= 4(x2 + y 2 + y) + 1
≡ 1 (mod 4).
Exercise 2.2.5 A positive integer a is the sum of two squares if and only if
a = b2 c where c is not divisible by any positive prime p ≡ 3 (mod 4).
b2 · c = n(b(t + ri))
= n(bt + bri)
= b 2 · t2 + b 2 · r 2
= (bt)2 + (br)2 .
α αβ
= .
β ββ
αβ
= s + tρ
ββ
y = a3 − 6ab2
188 CHAPTER 2. EUCLIDEAN RINGS
and
1 = (3a2 b − 2b3 )
= b(3a2 − 2b2 ).
This gives infinitely many solutions to x2√− 2y 2 = ±1. It is easy to see that
all of these solutions are distinct: ε ∈ Z[ 2] and ε > 1 so εn+1 > εn for all
positive n.
√ √
Exercise 2.4.7 Show that there is no unit η in Z[
√ 2] such that 1 < η < 1 √
+ 2.
Deduce that every unit (greater than zero) of Z[ 2] is a power of ε = 1 + 2.
2.5. SUPPLEMENTARY PROBLEMS 189
Solution. Since −1 is a unit, for any unit ξ, −ξ is also a unit, and negative
and positive units are in√one-to-one correspondence;√we shall only consider
the positive units
√ of Z[ √2]. We write η as a + b 2. Since√ η is a unit,
φ(η) =√(a + b √ 2)(a − b 2) = ±1. By assumption (a + b 2) > 1 and
|(a + b 2)(a − b 2)| = 1, so it follows that
√
−1 < (a − b 2) < 1.
√ √
Also, by assumption, 1 < a + b 2 < 1 + 2. So, adding these two inequal-
ities gives √
0 < 2a < 2 + 2.
Since a ∈ Z this implies that a = 1. Notice now that there is no integer b
such that √ √
1 < 1 + b 2 < 1 + 2.
If any unit, ψ, did exist which was not some power of√ε, then by our
Euclidean algorithm√we would be able√ tok+1divide by (1 + 2)k , where k is
chosen so that (1 + 2) < ψ √
k
< (1 + 2) and this would produce √a new
unit ψ $ where 1 < ψ $ <
√ 1 + 2. So the only positive units of Z[ 2] are
those of the form (1 + 2)n ; there are infinitely many.
This implies that b | 8 and so we have 8 possibilities: b = ±1, ±2, ±4, ±8.
Substituting these back into the equations to find a, x, and y, and remem-
bering that a ≡ b (mod 2) and that a, x, y ∈ Z will give all solutions to the
equation.
2.5. SUPPLEMENTARY PROBLEMS 191
Case 2. δ = 2.
If δ = 2, then y is even and x is odd. We can write y = 2y1 , which gives
the equation ! √ "! √ "
x + −11 x − −11
= 2y13 .
2 2
Since 2 divides the right-hand side of this equation, it must divide the
left-hand side, so 1! √ "
1 x + −11
1
2 1
2
or 1! √ "
1 x − −11
1
2 1 .
2
However, since x is odd, 2 divides neither of the factors above. We conclude
that δ != 2, and thus we found all the solutions to the equation in our
discussion of Case 1.
√
Exercise 2.5.4 Prove that Z[ 3] is Euclidean.
√ √
Solution. Given α, β ∈ Z[ 3] we want to find γ, δ ∈ Z[ 3] such that
α = βγ + δ, with N (δ) < N (β). Put √ another way, we want to show
√ that
N (α/β − γ) < 1. Let α/β = x + y 3, x, y ∈ Q. Let γ = u + v 3, with
u, v ∈ Z.
Now, N (α/β − γ) = |(x − u)2 − 3(y − v)2 |. This will be maximized when
(x − u) is small and (y − v) is large. Choose for u and v the closest integers
to x and y, respectively. Then the minimum value for (x − u) is 0, while
the maximum value for (y − v) is 1/2. Then N (α/β − γ) ≤ |− 3/4| < 1.
The conclusion follows.
√
Exercise 2.5.5 Prove that Z[ 6] is Euclidean.
√
Solution. Assume √ that Z[√ 6] is not Euclidean. This means that
√ there √ is
at least one x + y 6 ∈ Q( 6) such that there is no γ = u + v 6 ∈ Z[ 6]
such that |(x − u)2 − 6(y − v)2 | < 1. Without loss, we can suppose that
0 ≤ x ≤ 1/2, and 0 ≤ y ≤ 1/2. We assert that there exist such a pair (x, y)
such that
(x − u)2 ≥ 1 + 6(y − v)2 ,
or
6(y − v)2 ≥ 1 + (x − u)2 ,
for every u, v ∈ Z. In particular, we will use the following inequalities:
If x = y = 0, then both first inequalities fail, so we can rule out this case.
Next, we look at the first two inequalities on the left. Since x2 , (1 − x)2 ≤ 1
and 1+6y 2 ≥ 1 and x, y are not both 0, these two inequalities fail so (2.1 (b))
and (2.2 (b)) must be true. Now consider (2.3 (a)). If (1 + x)2 ≥ 1 + 6y 2
and 6y 2 ≥ 1 + (1 − x)2 as we just showed, then
(1 + x)2 ≥ 1 + 6y 2 ≥ 2 + (1 − x)2
which implies that 4x ≥ 2 and since x ≤ 1/2, we conclude that x = 1/2.
Substituting this into the previous inequalities, we get that
9
4 ≥ 1 + 6y 2 ≥ 94 ,
so 6y 2 = 54 . Let y = r/s with gcd(r, s) = 1. We now have that 24r2 = 5s2 .
Since r ! s, then r2 | 5, so r = 1. But then 24 = 5s2 , a contradiction.
Therefore, (2.3 (b)) is true, which implies that
6y 2 ≥ 1 + (1 + x)2 ≥ 2.
However, since y ≤ 1/2, 6y 2 ≥ 2 implies that√6 ≥ 8, a contradiction. Then
neither (2.3 (a)) nor (2.3 (b)) are true, so Z[ 6] must be Euclidean.
√
Exercise 2.5.6 Show that Z[(1 + −19)/2] is not Euclidean for the norm map.
√
Exercise 2.5.8 Show that there are only finitely many rings Z[ d] with d ≡ 2
or 3 (mod 4) which are norm Euclidean.
√ √
Solution. If Z[ d]√is Euclidean for the norm map, then for any δ ∈ Q( d),
we can find α ∈ Z[ d] such that
|N (δ − α)| < 1.
√ √
Write δ = r + s d, α = a + b d, a, b ∈ Z, r, s ∈ Q. Then
z 2 − t2 = d(x2 − 5)
or
z 2 − t2 = d(x2 − 6)
is true. We consider this modulo 8. Then t2 ≡ 1 (mod 8) since t is odd.
Also, x2 , z 2 ≡ 0, 1, or 4 (mod 8) and d ≡ 3 or 7 (mod 8). We are easily led
to t2 − z 2 ≡ 0, 1, or 5 (mod 8). This means
d(x2 − 5) ≡ 5, 4, or 1 (mod 8)
or
d(x2 − 6) ≡ 6, 5, 2, or 1 (mod 8).
All of these congruences are impossible. In case d ≡ 2 (mod 4), we choose
t odd satisfying 2d < t2 < 3d and proceed as above.
(The case d ≡ 1 (mod 4) is more difficult and has been handled by
Heilbronn who was the first to show that there are only finitely many real
quadratic fields which are norm-Euclidean.)
A more general and analogous result for imaginary quadratic fields will
be proved in Exercise 4.5.21 in Chapter 4.
194 CHAPTER 2. EUCLIDEAN RINGS
2 = v 3 − u3 = (v − u)(v 2 + vu + u2 )
v − u = ±1, v 2 + vu + u2 = ±2
or
v − u = ±2, v 2 + vu + u2 = ±1.
This gives rise to four cases. The only case that leads to a solution is
v − u = 2 and v 2 + vu + u2 = 1. This yields the solution (x, y) = (−1, 0).
Now suppose (y − 1, y + 1) = 2. This gives rise to two cases
x3 + y 3 = 2z 3 .
N (a + bρ) = a2 + ab + b2 .
Its unit group is {±1, ±ρ, ±ρ2 }. It is also easily checked that 1, ρ, ρ2 repre-
sent all the distinct coprime residue classes modulo 2Z[ρ]. We see that the
cube of every coprime residue class is 1 (modulo 2). If u is a unit ≡ 1 (mod
2), then u = ±1. Now we claim that any coprime solution (x, y, z) of
x3 + y 3 = 2uz 3
in Z[ρ] satisfies N (xyz) = 1. Suppose not. Let (x, y, z) be such that N (xyz)
is minimal and ≥ 2. We may let
A = x + y, B = ρx + ρ2 y, C = ρ2 x + ρy
so that
ABC = 2uz 3 , A + B + C = 0.
2.5. SUPPLEMENTARY PROBLEMS 195
u1 α3 + u2 β 3 = 2u3 γ 3 ,
α3 + u$ β 3 = 2u4 γ 3
for some units u$ and u4 . Observe that (β, 2) = 1 for otherwise, 2|α and 2|γ
which implies that α, β, γ are not coprime, a contradiction. Reducing the
above equation mod 2 shows that u$ is a cube mod 2, and by our remark
above u$ must be ±1. Thus u$ is a cube and we have
α3 + β 3 = 2uγ 3 .
Solution. Let V (x1 , ..., xn ) denote the value of the determinant. If we fix
x2 , ..., xn , we may view the determinant as a polynomial in x1 of degree
n − 1. Since the determinant is zero if x1 = xi for i ≥ 2, the roots of the
polynomial are x2 , ..., xn . It is easy to see that the leading coefficient is
197
198 CHAPTER 3. ALGEBRAIC NUMBERS AND INTEGERS
√
Exercise 3.1.6 Find the minimal polynomial of n where n is a squarefree
integer.
*k
1 pk
= ,
n=1
2 3n qk
with qk = 23 . As before,
k
1 1 11 ∞ 1
1 p k 1 1 * 1 11 S
1α − 1 = 1 1≤ .
1 qk 1 1 23n 1 23k+1
n=k+1
S c(α)
≥ 2+ε .
qk3 qk
f (n + 1)
lim > 2.
n→∞ f (n)
Solution. Suppose
∞
* 1
α=
n=1
2f (n)
3.3. ALGEBRAIC NUMBER FIELDS 199
which implies that qk+1 > qk2+δ . By Roth’s theorem, we can deduce that
c(α) S
≤ ,
qk2+ε qk+1
c(α) S
⇒ ≤ ,
qk2+ε qk2+δ
S ε
⇒ qkδ ≤ q .
c(α) k
So p$ (α) = 0 and from Theorem 3.1.4, p(x) | p$ (x). But deg(p$ ) < deg(p),
and we have a contradiction. If β is a repeated root of p(x), then by the
following exercise, β has the same minimal polynomial and repeating the
above argument with β leads to a contradiction. Thus p has no repeated
roots.
Solution. Let p(x) be the minimal polynomial of α, and let q(x) be the
minimal polynomial of β. By the definition of conjugate roots, β is a
common root of p(x) and q(x).
Using the division algorithm, we can write p(x) = a(x)q(x) + r(x) for
some a(x), r(x) ∈ Q[x] and either r = 0 or deg(r) < deg(q). But
and q(β) = 0 so r(β) must also be 0. Since q is the minimal polynomial for
β, r = 0. Thus p(x) = a(x)q(x), but, by Theorem 3.1.4, p is irreducible,
and both p(x) and q(x) are monic, so p(x) = q(x).
Solution.
(2) (n)
ω1 ω1 ··· ω1
(2) (n)
ω2 ω2 ··· ω2
Ω=
.. .. .. .
. . .
(2) (n)
ωn ωn ··· ωn
Since θ is an algebraic number of degree n, α1 = 1, α2 = θ, . . . , αn = θn−1
(j)
also forms a basis for K over Q. Let A = (αi ). Then,
1 1
1 1 1 ··· 1 11
1
1 θ
1 θ(2) ··· θ(n) 11
det A = 1 . .. .. 1
1 .. . . 11
1
1θn−1 θ(2)n−1 · · · θ(n)n−1 1
Exercise 3.3.8 Show that Z[x] is not (a) Euclidean or (b) a PID.
Solution. Suppose it is algebraic, and call the sum α. Look at the partial
sum
* k
1 pk
αk = n!
= ,
n=0
a qk
with qk = ak! . Then
1 1
1 *
∞
1 11
1
|α − αk | = 1 1
1 an! 1
n=k+1
1
≤ M,
a(k+1)!
where ! "2
1 1
M =1+ + + ··· ,
ak+2 ak+2
an infinite geometric series with a finite sum. Thus,
1 1
1 * ∞
1 11 M
1
1 n! 1 ≤ k+1 .
1 a 1 qk
n=k+1
is transcendental for a ∈ Z, a ≥ 2.
*k
1 pk
αk = 3n
= ,
n=1
a qk
with qk = a3 . We have
k
1 1 11 ∞ 1
1 1 * 1 11
1α − pk 1 = 11 n 1 ≤
S
,
1 qk 1 1 3
a 1 a k+13
n=k+1
S c(α, ε)
≥ 2+ε .
qk3 qk
f (k + 1)
> 2 + δ,
f (k)
204 CHAPTER 3. ALGEBRAIC NUMBERS AND INTEGERS
and so
qk+1 af (k+1) (1+δ)
= f (k) > a(1+δ)f (k) = qk .
qk a
This implies that qk+1 > qk2+δ . By Roth’s theorem, we can deduce that
c(α) S c(α) S
2+ε ≤ q ⇒ 2+ε ≤ 2+δ ,
qk k+1 qk qk
S ε
⇒ qkδ ≤ q .
c(α) k
As qk → ∞, we find δ ≤ ε, a contradiction for sufficiently small ε.
Exercise 3.4.5 Using Thue’s theorem, show that f (x, y) = x6 +7x5 y −12x3 y 3 +
6xy 5 + 8y 6 = m has only a finite number of solutions for m ∈ Z∗ .
Solution. Use the previous exercise to prove that the polynomial is irre-
ducible. The result follows from Thue’s theorem and Example 3.2.3.
Solution. Since
m−1
6
xm − 1 = (x − ζm
i
),
i=0
we see that the constant term is
m−1
6
(−1)m i
ζm = −1.
i=0
Solution. Every mth root of unity is a primitive dth root of unity for some
d | m. Conversely, every dth root of unity is also an mth root of unity for
d | m. The result is now immediate.
we have by induction v(x) ∈ Z[x]. Since v(x) is monic, and v(x) | (xm − 1),
we find by long division that (xm − 1)/v(x) = φm (x) ∈ Z[x].
f (ζm
p
)g(ζm
p
) = 0.
Suppose f (ζmp
) != 0. Then g(ζm p
) = 0. Since g(xp ) ≡ g(x)p (mod p) we
deduce that g(x) and f (x) have a common root in Fp , a contradiction since
xm − 1 has no multiple roots in Fp . Thus, f (ζm p
) = 0 for any (p, m) = 1.
It follows that f (ζm ) = 0 for any (i, m) = 1. Therefore deg(f ) = ϕ(m) =
i
deg(φm ).
Exercise 3.4.10 Let I be a subset of the positive integers ≤ m which are coprime
to m. Set # i
f (x) = (x − ζm ).
i∈I
Exercise 3.4.13 Let ζm denote a primitive mth root of unity. Show that Q(ζm )
is normal over Q.
Exercise 3.4.14 Let a be squarefree and greater than 1, and let p be prime.
Show that the normal closure of Q(a1/p ) is Q(a1/p , ζp ).
Integral Bases
By Lemma 4.1.1, if α ∈ OK , then the trace and norm are integers. Also, α
is a root of the monic polynomial x2 − 2r1 x + r12 + 5r22 which is in Z[x]
√ when
the trace and norm are integers. We conclude that for α = r1 + r2 −5 to
be in OK , it is necessary and sufficient that 2r1 and r12 + 5r22 be integers.
This implies that r1 has a denominator at most 2, which forces the same for
r2 . Then by setting r1 = g1 /2 and r2 = g2 /2 we must have (g12 +5g22 )/4 ∈ Z
or, equivalently, g12 + 5g22 ≡ 0 (mod 4). Thus, as all squares are congruent
to 0 or 1 (mod 4), we conclude that g1 and g2 are√themselves even, and
thus r1 , r2 ∈ Z. We conclude then that OK = Z + Z −5.
207
208 CHAPTER 4. INTEGRAL BASES
A = P T BP,
(j)
If we introduce now the matrices, C = (cij ), Ω = (ωi ), then the above
becomes
In = ΩΩT C ⇒ C −1 = ΩΩT .
We conclude that C is nonsingular and that ω1 ∗ , ω2 ∗ , . . . , ωn ∗ forms a Q-
basis for K.
Let α be an arbitrary element of OK . We write
n
*
α= aj ωj∗ with aj ∈ Q
j=1
so
n
*
αωi = aj ωi ωj∗ ∀i,
j=1
and *
TrK (αωi ) = aj TrK (ωi ωj∗ ) = ai ∀i.
But αωi ∈ OK implies the left-hand side above is in Z, and thus ai ∈ Z for
all i. It follows then that OK ⊆ Zω1∗ + Zω2∗ + · · · + Zωn∗ .
for all i, where cij and dij are all integers. Then (cij ) and (cij )−1 both have
entries in Z. So det(cij ), det(cij )−1 ∈ Z, meaning that det(cij ) = ±1.
Then
// 0/ 00
* *
Tr(ωi ωj ) = Tr cil θl cjm θm
l m
*
= cil cjm Tr (θl θm ) .
l,m
(j) (j)
Now if we define Ω = (ωi ), C = (cij ), Θ = (θi ), then we can write
the above as the matrix equation ΩT Ω = C(ΘT Θ)C T from which it follows
that the determinants of Ω and Θ are equal, up to sign. Hence, det(ΘT Θ) =
det(ΩT Ω).
Exercise 4.2.5 Show that the discriminant is well-defined. In other words, show
that given ω1 , ω2 , . . . , ωn and θ1 , θ2 , . . . , θn , two integral bases for K, we get the
same discriminant for K.
Solution. First#we note$ that σi (a) takes a to its ith conjugate, a(i) . Define
the matrix Ω = σi (aj ) . Then it is easy to see that
1 a ··· an−1
. .. ..
Ω = .. . . ,
1 a(n) ··· a(n)n−1
which is a Vandermonde matrix, and so
6# $ 6# $
det Ω = a(i) − a(j) = σi (a) − σj (a) .
i>j i>j
It follows that
< =2
dK/Q (a) = det(σi (aj ))
6# $2
= σi (a) − σj (a) .
i<j
&n
Exercise 4.2.7 Suppose that ui = j=1 aij vj with aij ∈ Q, vj ∈ K. Show that
! "2
dK/Q (u1 , u2 , . . . , un ) = det(aij ) dK/Q (v1 , v2 , . . . , vn ).
< =2
Solution. By definition, dK/Q (u1 , u2 , . . . , un ) = det(σi (uj )) .
/ n 0 n
* *
σi (uj ) = σi ajk vk = ajk σi (vk ).
k=1 k=1
If we define the matrices U = (σi (uj )), A = (aij ), V = (σi (vj )), then it is
clear that U = V AT and so (det U )2 = (det V AT )2 , and we get the desired
result:
dK/Q (u1 , u2 , . . . , un ) = (det(aij ))2 dK/Q (v1 , v2 , . . . , vn ).
4.3 Examples
Exercise 4.3.2 Let m ∈ Z, α ∈ OK . Prove that dK/Q (α + m) = dK/Q (α).
2
Solution. By definition, dK/Q (α) = i<j (α(i) − α(j) )2 . We note that the
ith conjugate of α + m is simply α(i) + m, and so
6# $2
dK/Q (α + m) = α(i) + m − (α(j) + m)
i<j
6# $2
= α(i) − α(j)
i<j
= dK/Q (α),
as desired.
Exercise 4.3.3 Let α be an algebraic integer, and let f (x) be the minimal poly-
n /
nomial of α. If f has degree n, show that dK/Q (α) = (−1)( 2 ) n (i)
i=1 f (α ).
(
and 6
f $ (α(i) ) = (α(i) − α(k) ).
k*=i
Therefore
n
6 n 6
6
(i)
# $
f (α )
$
= α(i) − α(k)
i=1 i=1 k*=i
6J # $2 K
= − α(i) − α(k)
i<k
Exercise 4.3.7 Let ζ be any primitive pth root of unity, and K = Q(ζ). Show
that 1, ζ, . . . , ζ p−2 form an integral basis of K.
1
= ±pp−1 2p−1
k=1 (1 − ζk)
= ±pp−2 ,
2p−1
since k=1 (1 − ζ k ) = Φ(1) = p. We know that dK/Q (ζ) = pp−2 = m2 dK
and also that p ! m because F , the minimal polynomial for ζ − 1, is p-
Eisensteinian, and dK/Q (ζ − 1) = dK/Q (ζ). Then m must be 1, meaning
that OK = Z[1, ζ, . . . , ζ p−2 ].
4.4. IDEALS IN OK 213
4.4 Ideals in OK
Exercise 4.4.1 Let a be a nonzero ideal of OK . Show that a ∩ Z #= {0}.
Exercise 4.4.3 Show that if a is a nonzero ideal in OK , then a has finite index
in OK .
Exercise 4.4.4 Show that every nonzero prime ideal in OK contains exactly one
integer prime.
Solution. By definition
(j) # $
dK = det(ωi )2 = det Tr(ωi ωj ) ,
Exercise 4.5.2 Let K/Q be an algebraic number field of degree n. Show that
dK ≡ 0 or 1 (mod 4). This is known as Stickelberger’s criterion.
det(σi (ωj )) = P − N,
where P is the contribution arising from the even permutations and N the
odd permutations in the definition of the determinant. Then
dK = (P − N )2 = (P + N )2 − 4P N.
dK/Q (θ) = m2 dK ,
< =
where m = OK : Z[θ] . Hence m = 1.
f $ (x) = nxn−1 + a
1
= (nxn + ax)
x
so that
(−n(aα(i) + b) + aα(i) )
f $ (α(i) ) = .
α(i)
Hence
n
6 n
6
f $ (α(i) ) = (−1)n b−1 (a(1 − n)α(i) − nb)
i=1 i=1
! "
nb
= b−1 an (1 − n)n f .
a(1 − n)
Solution. Note that if (a) is not true, then f has a linear factor and by
the rational root theorem, this factor must be of the form x − a where a | 8.
A systematic check rules out this possibility. (b) can be checked directly.
(c) This is easy to deduce from the formula
β2 = 6 + 2θ + 3β,
2
θ = 2β − θ,
θβ = 2β + 4,
we find
so that
dK/Q (1, x, x2 ) ≡ −503(BC)2 (3C + B)2 (mod 2),
which is an even number in all cases.
By (d), dK/Q (α) is even and hence is not equal to −503, which proves
(e).
(1 − ζm )ϕ(m) = pOK .
1 − ζm 1 − ζm
ab
= = 1 + ζm
b
+ · · · + ζm
b(a−1)
1 − ζm
b 1 − ζm
b
(−1)ϕ(m)/2 mϕ(m)
dK/Q (ζm ) = .
pm/p
4.5. SUPPLEMENTARY PROBLEMS 217
Let 6
θ= (ζm − ζm
b
).
1≤b<m
(b,m)=1
Clearly θ = φ$m (ζm ) and NK/Q (θ) is the discriminant we seek. Since
xm − 1
φm (x) = ,
xm/p − 1
we find
m−1
mζm
φ$m (ζm ) = m/p
,
ζm −1
and the norm of this element is
mm
m/p
.
NK/Q (ζm − 1)
m/p
Because η = ζm is a primitive pth root of unity,
NK/Q (ζm
m/p
− 1) = NQ(η)/Q (η − 1)m/p .
ϕ(m)−1
Exercise 4.5.9 Let m = pa , with p prime. Show that {1, ζm , . . . , ζm } is
an integral basis for the ring of integers of K = Q(ζm ).
for each (c, m) = 1. We solve for bj using Cramer’s rule. Moreover, by the
previous question, we see that
cj
bj = cj ∈ Z.
pdj
(−1)ϕ(m)/2 mϕ(m)
dK = .
pm/p
Exercise 4.5.11 Show that Z[ζn + ζn−1 ] is the ring of integers of Q(ζn + ζn−1 ),
where ζn denotes a primitive nth root of unity, and n = pα .
ζnN α = aN + · · · + aN ζn2N
2N ≤ φ(n) − 2 ≤ φ(n) − 1,
since δ and each of γi are algebraic integers. Hence dK mij /r are all integers.
It follows that r divides all dK mij . Since gcd(r, gcd(mij )) = 1, we deduce
r | dK .
∆ = dK/Q (a1 , . . . , an ).
∆ = dK/Q (a1 , . . . , an ).
For each i, choose the least natural number dii so that for some dij ∈ Z, the
number
,i
wi = ∆−1 dij aj ∈ OK .
j=1
by Exercise 4.2.7, and the right-hand side is nonzero. Observe now that if
α ∈ OK can be written as
α = ∆−1 (c1 a1 + · · · + cj aj )
for some j, then djj | cj . Indeed, write cj = sdjj + r, 0 ≤ r < djj , so that
# $
α − swj = ∆−1 (c1 − dj1 )a1 + · · · + raj ∈ OK ,
4.5. SUPPLEMENTARY PROBLEMS 221
∆−1 (x1 a1 + · · · + xj aj )
y = ∆−1 (x1 a1 + · · · + xk ak ) ∈ OK
aj = cj1 w1 + · · · + cjj wj ,
cij ∈ Z, j = 1, . . . , n.
1 (1) (m)
12
1w ··· w1 0 ··· 0 1
1 1 1
1 . 1
1 .. 1
1 1
1 (1) (m) 1
1 wm ··· wm 0 ··· 0 1
= 1 (1) (m) (m+1) (1) (mn) (m) 1
1wm+1 ··· wm+1 wm+1 − wm+1 ··· wm+1 − wm+1 11
1
1 .. 1
1 . 1
1 (1) (m) (m+1) (1) (mn)
1
(m) 1
1w ··· wmn wmn − wmn ··· w −w
mn mn mn
= dK · a,
r1 + 2r2 = [K : Q] = n
Exercise 4.5.21 Show that only finitely many imaginary quadratic fields K are
Euclidean.
4.5. SUPPLEMENTARY PROBLEMS 223
Solution. The argument of the previous exercise shows that not all residue
classes mod 2 and mod 3 are represented by elements of smaller ψ-value.
(I ⊗ U )−1 (A ⊗ B)(I ⊗ U ) = A ⊗ (U −1 BU )
which is
Ac11 Ac12 ··· Ac1n
0 Ac22 ··· Ac2n
.. .. .. .
. . .
0 0 ··· Acnn
Again, by linear algebra, we see that
n
6
det(A ⊗ B) = det(Acii )
i=1
6n
= (cm
ii det A)
i=1
= (det B)m (det A)n ,
as desired.
dKL = dn m
K · dL .
If we set
log |dM |
δ(M ) =
[M : Q]
deduce that δ(KL) = δ(K) + δ(L) whenever gcd(dK , dL ) = 1.
Exercise 4.5.25 Let ζm denote a primitive mth root of unity and let K =
Q(ζm ). Show that OK = Z[ζm ] and
(−1)φ(m)/2 mϕ(m)
dK = / φ(m)/(p−1)
.
p|m p
4.5. SUPPLEMENTARY PROBLEMS 225
Solution. Factor 6
m= pα .
pα -m
log |dK | * ! 1
"
= α− log p.
φ(m) α
p−1
p -m
dK/Q (θ) = m2 dK .
Dedekind Domains
{xi x1 , xi x2 , . . . , xi xn } = {x1 , x2 , . . . , xn }.
Exercise 5.1.5 Show that every unique factorization domain is integrally closed.
227
228 CHAPTER 5. DEDEKIND DOMAINS
Exercise 5.2.4 Show that any principal ideal domain is a Dedekind domain.
√
Solution. Z[ −5] is not a unique factorization
√ √ domain as was seen in
Chapter 2 by taking 6 = 2 × 3 = (1 + −5)(1 − −5), and so cannot be a
principal ideal domain.
To see that it is a Dedekind domain, it is enough to show that √ it is
the set of algebraic integers of the algebraic number field K =√Q( −5).
However, we have already proved this, in Exercise 4.1.3. So Z[ −5] is a
Dedekind domain.
Exercise 5.3.2 Show that the sum and product of two fractional ideals are again
fractional ideals.
mn(AB) = (mA)(nB)
⊆ OK ,
so A + B is a fractional ideal.
Exercise 5.3.7 Show that any fractional ideal A can be written uniquely in the
form
℘1 . . . ℘ r
,
℘(1 . . . ℘(s
where the ℘i and ℘(j may be repeated, but no ℘i = ℘(j .
230 CHAPTER 5. DEDEKIND DOMAINS
a1 · · · a v
A= ,
b1 · · · bw
with no ai = bj , then
a1 · · · av m1 · · · ms = b1 · · · bw n1 · · · nt .
Solution. Let
℘1 · · · ℘r
A= .
℘$1 · · · ℘$s
Then
℘$1 · · · ℘$s
A−1 =
℘1 · · · ℘r
Exercise 5.3.9 Show that if a and b are ideals of OK , then b | a if and only if
there is an ideal c of OK with a = bc.
/r min(ei ,fi )
Exercise 5.3.10 Show that gcd(a, b) = a + b = i=1 ℘i .
Solution. a ⊆ a + b, b ⊆ a + b, so a + b | a, a + b | b.
If e | a and e | b, then a ⊆ e, b ⊆ e, so that a + b ⊆ e, i.e., e | a + b.
Therefore, a + b = gcd(a, b).
5.3. FRACTIONAL IDEALS AND UNIQUE FACTORIZATION 231
2r min(ei ,fi )
Let d = i=1 ℘i , and let min(ei , fi ) = ai ,
r
6
a = ℘ei i
i=1
6r r
6
= ℘ei i −ai ℘ai i
i=1 i=1
6r
= ℘ei i −ai d, ei − ai ≥ 0 ∀i.
i=1
where ℘1 , . . . , ℘r , ℘$1 , . . . , ℘$s are distinct prime ideals. Suppose ki < ei for
some i,
e ⊆ a,
⇒ ℘k11 · · · ℘kr r (℘$1 )t1 · · · (℘$s )ts ⊆ ℘e11 · · · ℘err ,
ki−1 ki+1
⇒ ℘k11 · · · ℘i−1 ℘i+1 · · · ℘kr r (℘$1 )t1 · · · (℘$s )ts ⊆ ℘e11 · · · ℘ei i −ki · · · ℘err
⊆ ℘ei i −ki
⊆ ℘i .
232 CHAPTER 5. DEDEKIND DOMAINS
a = ℘e11 · · · ℘err
and
b = (℘$1 )f1 · · · (℘$t )ft
where ℘1 , . . . , ℘r , ℘$1 , . . . , ℘$t are distinct prime ideals since gcd(a, b) = 1.
Now let c = ℘a1 1 · · · ℘ar r (℘$1 )b1 · · · (℘$t )bt . Since ab = cg , we must have
ei = ai g, 1 ≤ i ≤ r,
fi = bi g, 1 ≤ i ≤ t,
d = ℘a1 1 · · · ℘rar ,
e = (℘$1 )a1 · · · (℘$t )at ,
as desired.
Exercise 5.3.14 Show that ord℘ (ab) = ord℘ (a) + ord℘ (b), where ℘ is a prime
ideal.
5n
where αωi = j=i rji αj , rji ∈ Z. Therefore, by definition,
R and R−1 have integer entries, since {αωi } and {αi } are both Z-bases
for (α). Thus, det(R) = ±1. So, det(C) = ±det(P ), det(P ) ≥ 0. Thus,
|det(C)| = det(P ).
Thus, |NK (α)| = |det(C)| = det(P ) = N ((α)).
pn = N (℘e11 ) · · · N (℘egg )
= N (℘1 )e1 · · · N (℘g )eg .
i=1
234 CHAPTER 5. DEDEKIND DOMAINS
5.5 Factorization in OK
Exercise 5.5.2 If in6 the previous
7 theorem we do not assume that OK = Z[θ]
but instead that p ! OK : Z[θ] , show that the same result holds.
OK /(p) 1 Z[θ]/(p),
then
OK /(p, fi (θ)) 1 Z[θ]/(p, fi (θ)).
The rest of the proof will be identical to what was written above.
Exercise 5.5.3 Suppose that f (x) in the previous exercise is Eisensteinian with
respect to the prime p. Show that p ramifies totally in K. That is, pOK = (θ)n
where n = [K : Q].
5.6. SUPPLEMENTARY PROBLEMS 235
< =
Solution. By Example 4.3.1, we know that p ! OK : Z[θ] . Moreover,
f (x) ≡ xn (mod p). The result is now immediate from Exercise 5.5.2.
x2 − x + 1 ≡ x2 + 6x + 1 ≡ (x + 2)(x + 4) (mod 7)
Exercise 5.6.3 Find a prime ideal factorization of (2), (5), (11) in Z[i].
x2 + 1 ≡ x2 − 4 ≡ (x + 2)(x − 2) (mod 5)
√
Exercise 5.6.4 Compute the different D of K = Q( −2).
N (D) = 3 = a2 − ab + b2 .
Since this is equivalent to (2a − b)2 + 3b2 = 12, we note that |b| cannot be
greater than 2. Checking all possibilities, we find all the elements of Z[ρ]
of norm 3: 2 + ρ, −1 + ρ, −2 − ρ, 1 − ρ, 1 + 2ρ and −1 − 2ρ. Some further
checking reveals that these six elements are all associates, and so they each
generate the same principal ideal. Thus, D = (2 + ρ).
b0 bn−1
,... , ( .
f ( (α) f (α)
Deduce that
1
D −1 = (Zb0 + · · · + Zbn−1 ).
f ( (α)
*n
f (x) αr
gr (x) = · $ i − xr .
i=1
x − αi f (αi )
5.6. SUPPLEMENTARY PROBLEMS 237
Consider gr (α1 ). Note that f (α1 )/(α1 − αi ) = 0 for all i except i = 1. Also,
! "
f (x)
= f $ (α1 ).
x − α1 x=α1
But,
! " n−1
* ! "
f (x)αr bi α r
Tr = Tr xi = xr .
(x − α)f $ (α) i=0
f $ (α)
Thus, ! "
bi α r
Tr =0
f $ (α)
unless i = r, in which case the trace is 1. Recall that if ω1 , . . . , ωn is a basis,
its dual basis ω1∗ , . . . , ωn∗ is characterized by Tr(ωi ωj∗ ) = δij , the Kronecker
delta function. Thus, we have found a dual basis to 1, α, . . . , αn−1 , and it
is
b0 bn−1
,... , $ .
f $ (α) f (α)
By Exercise 5.4.1,
1
D−1 = (Zb0 + · · · + Zbn−1 ).
f $ (α)
Since f (x) is monic, bn−1 = 1. Also, an−1 = bn−2 − αbn−1 which means
that bn−2 = an−1 + α, where an−1 is an integer.
238 CHAPTER 5. DEDEKIND DOMAINS
and so D = (f $ (α)).
xp − 1
f (x) = = xp−1 + xp−2 + · · · + x + 1,
x−1
pxp−1 (x − 1) − (xp − 1)
f $ (x) = ,
(x − 1)2
pζp−1
f $ (ζp ) = .
ζp − 1
From Exercise 5.5.4 we know that (p) = (1−ζp )p−1 , and so D = (1−ζp )p−2 .
implies that φm (0) = ±1. Thus, the constant term of φm (x) is ±1 so that
φm (a) is coprime to a and hence coprime to m. (If φm (a) = ±1, one can
replace a by any suitable power of a, so that |φm (a)| > 1.)
By what we have proved, any prime divisor p of φm (a) coprime to m
must be congruent to 1 (mod m). The prime p is distinct from p1 , . . . , pr .
Exercise 5.6.12 Let p be prime and let a be squarefree and coprime to p. Set
θ = a1/p and consider K = Q(θ). Show that OK = Z[θ] if and only if ap−1 #≡ 1
(mod p2 ).
℘ = (p, θ − a).
(θ − a) = ℘a
N (θ − a) = ap − a = pN a
so that ap !≡ a (mod p2 ).
Conversely, suppose that ap !≡ a (mod p2 ). Then the polynomial
(x + a)p − a
< =
is Eisenstein with respect to the prime p. Therefore p ! OK< : Z[θ − a]=
by Example 4.3.1. But Z[θ − a] = Z[θ] so we deduce that p ! OK : Z[θ] .
In addition, xp − a is Eisenstein with respect
< to every
= prime divisor of a.
Again, by Example 4.3.1, we deduce that OK : Z[θ] is coprime to a. By
Exercises 4.3.3 and 4.2.8,
< =2
dK/Q (θ) = (−1)(2) pp ap−1 = OK : Z[θ] · dK .
p
Solution. We will use the result of Theorem 5.5.1. Let f (x) be the minimal
polynomial of Z[θ]. Suppose that p | dK , and
Exercise 5.6.14 Let K = Q(θ) and suppose that p | dK/Q (θ), p2 ! dK/Q (θ).
Show that p | dK and p ramifies in K.
< =
Solution. Recall that dK/Q (θ) = m2 dK where m = OK : Z[θ] . Clearly,
since p | dK/Q (θ) but p2 ! dK/Q (θ), p | dK , and p ! m. We can now apply the
result of Exercise 5.5.2. Using the same argument as in 5.6.13, we deduce
that f (x) has a multiple root mod p and so p ramifies in K.
5.6. SUPPLEMENTARY PROBLEMS 241
Exercise 5.6.18
√ Determine the prime ideal factorization of (7), (29), and (31)
in K = Q( 3 2).
29OK = ℘1 ℘2
as desired.
Exercise 5.6.21 Let L/K be a finite extension of algebraic number fields. Show
that the map
T rL/K : L × L → K
is non-degenerate.
Exercise 5.6.22 Let L/K be a finite extension of algebraic number fields. Let
a be a finitely generated OK -module contained in L. The set
−1
DL/K (a) = {x ∈ L : T rL/K (xa) ⊆ OK }
−1
Solution. We have x ∈ DM/L if and only if T rM/L (xOM ) ⊆ OL which is
equivalent to
−1 −1 −1
DL/K T rM/L (xOM ) ⊆ DL/K OL iff T rL/K (DL/K T rM/L (xOM )) ⊆ OK
−1
which by transitivity of the trace is equivalent to T rM/K (xDL/K ) ⊆ OK .
That is, we must have
−1 −1
xDL/K ⊆ DM/K
which means
−1 −1
DM/L = DL/K DM/K ,
which gives the result. We remark here that Dedekind’s theorem concerning
ramification extends to relative extensions L/K. More precisely, a prime
ideal p of OK is said to ramify in L if there is a prime ideal ℘ of OL such
that ℘2 |pOL . One can show that p ramifies in L if and only if p|dL/K . The
easy part of this assertion that if p is ramified then p|dL/K can be proved
following the argument of Exercise 5.4.5. The converse requires further
theory of relative differents. We refer the interested reader to [N].
Exercise 5.6.26 Let L/K be a finite extension of algebraic number fields. Sup-
pose that OL = OK [α] for some α ∈ L. If f (x) is the minimal polynomial of α
over OK , show that DL/K = (f ( (α)).
244 CHAPTER 5. DEDEKIND DOMAINS
Solution. This result is identical to Exercises 5.6.6 and 5.6.7. More gen-
erally, one can show the following. For each θ ∈ OL which generates L over
K, let f (x) be its minimal polynomial over OK . Define δL/K (θ) = f $ (θ).
Then DL/K is the ideal generated by the elements δL/K (θ) as θ ranges over
such elements. We refer the interested reader to [N].
dK1 /K dK2 /K
also divides dL/K . Suppose now that p is a prime ideal of OK which divides
dL/K but not dK1 /K . We have to show that p divides dK2 /K . By the defini-
tion of the relative discriminant, there is a prime ideal ℘ of OL lying above
p which divides the different DL/K . This ideal cannot divide DK1 /K OL
for this would imply that p divides dK1 /K , contrary to assumption. Since
DL/K = DL/K1 DK1 /K , we deduce that ℘ divides DL/K1 . Now let α ∈ OK2
so that α generates K2 over K. Let f (x) be its minimal polynomial over
K1 and g(x) its minimal polynomial over K. (We have assumed that we
have fixed a common algebraic closure which contains K1 and K2 .) Then
L = K1 (α) and g(x) = f (x)h(x) for some polynomial h over K1 . Hence,
g $ (x) = f $ (x)h(x)+f (x)h$ (x) which implies g $ (α) = f $ (α)h(α). Thus, g $ (α)
is in the ideal generated by f $ (α). By the remark in the solution of Exercise
5.6.26, we deduce that f $ (α) ∈ DL/K ⊆ ℘. Therefore, g $ (α) ∈ ℘. The same
remark enables us to deduce that g $ (α) ∈ DK2 /K implying that p divides
dK2 /K .
Exercise 6.2.3 Show that each equivalence class of ideals has an integral ideal
representative.
245
246 CHAPTER 6. THE IDEAL CLASS GROUP
We now have
b ceb
(t)A = (t) = = eb ⊆ OK .
c c
Thus, A ∼ be ⊆ OK , and the result is proved.
Exercise 6.2.4 Prove that for any integer x > 0, the number of integral ideals
a ⊆ OK for which N (a) ≤ x is finite.
Solution. Since the norm is multiplicative and takes values > 1 on prime
ideals, and since integral ideals have unique factorization, it is sufficient to
prove that there are only a finite number of prime ideals ℘ with N (℘) ≤ x.
Now, any prime ℘ contains exactly one prime p ∈ Z, as shown in Exer-
cise 4.4.4. Thus, ℘ occurs in the factorization of (p) ⊆ OK into prime ideals.
Since N (℘) ≥ 2, we have N (℘) = pt for some t ≥ 1. This implies 2 there are
s
at most n possibilities for such2s ℘, since the factorization (p) = ai
i=1 ℘i
implies that p = N ((p)) = i=1 N (℘i ) leading to s ≤ n. Moreover,
n ai
Exercise 6.2.6 Show that the product defined above is well-defined, and that H
together with this product form a group, of which the equivalence class containing
the principal ideals is the identity element.
Thus, A1 A2 ∼ B1 B2 .
Now, it is easy to check that H with the product defined above is closed,
associative, commutative, and has the class of principal ideals as the iden-
tity element. Thus, to finish the exercise, we need to show that each element
of H does have an inverse. Suppose C is an arbitrary element of H. Let
a ⊆ OK be a representative of C (we showed in Exercise 6.2.3 that every
equivalence class of ideals contains an integral representative). If we pro-
ceed as we did when deriving equation (6.1), we conclude that there exists
an integral ideal b such that ab is principal. It then follows immediately
that the class containing b is the inverse of C.
Exercise 6.2.7 Show that the constant CK in Theorem 6.2.2 could be taken to
be the greatest integer less than or equal to HK , the Hurwitz constant.
Then |S| ≥ N (a) + 1. Since N (a) = [OK : a], we can find distinct a, b ∈ S
such that a ≡ b (mod a). Thus, (a − b) ⊆ a. This implies that there exists
an integral ideal b such that
5n(a − b) = ab. It is easy to observe that b ∈ C.
We may write a − b = i=1 pi ωi . Since a, b ∈ S, |pi | ≤ (N (a))1/n + 1,
and so we have
1 n ! n "1
16 * (j) 11
|N (a − b)| = 1 1 pi ω i 1
j=1 i=1
n
/ n 0
6 * (j)
≤ |pi ||ωi |
j=1 i=1
n
/ n 0
< =n 6 * (j)
≤ (N (a))1/n + 1 |ωi |
j=1 i=1
< =n
≤ (N (a))1/n + 1 HK .
However, observe that we can always replace a by the ideal ca, in the same
equivalence class, for< any c ∈ OK \{0},
=n and with |N (c)| arbitrarily large; we
can therefore make 1 + (N (a))−1/n arbitrarily close to 1.
Thus, every equivalence class C has an integral representative b with
N (b) ≤ HK . This implies that every ideal is equivalent to another integral
ideal with norm less than or equal to HK .
x2 + k = y 3 . (6.4)
√
Show that if 3 does not divide the class number of Q( −k), then this equation
has no integral solution.
Solution. Since
√ d is even√ and squarefree, d ≡ 2 (mod 4). The ring of
integers of Q( −d) is Z[ −d]. We have the ideal factorization:
√ √
(n)g = (ng ) = (1 + d) = (1 + −d)(1 − −d).
√ √
The ideals (1 + −d) and (1 − −d) are √ coprime since
√ n is odd. Thus
by Theorem 5.3.13, each of the ideals (1 + −d), (1 − −d) must be gth
powers. Thus
√
ag = (1 + −d),
√
(a$ )g = (1 − −d),
with aa$ = (n). Hence a has √ order dividing g in the class group.
Suppose am = (u + v −d) for some u, v ∈ Z. Note that v cannot
be zero for otherwise am = (u) implies that (a$ )m = (u) so that (u) =
gcd(am , (a$ )m ), contrary to gcd(a, a$ ) = 1. Therefore
√ v != 0.
Now take norms of the equation am = (u + v −d) to obtain
nm = u2 + v 2 d ≥ d = ng − 1.
Since 12 − 14 − 61 = 12
1
, we see N > 3g/2 . By the previous exercise, each of
these values gives rise to a distinct quadratic field whose class group has
exponent divisible by g. By applying this result for powers of g we deduce
that there are infinitely many imaginary quadratic fields of class number
divisible by g.
(This argument is due to Ankeny and Chowla.)
These are all principal ideals√and thus are all equivalent. This shows that
the class number of K = Q( −19) is 1.
Since ψ(x) takes only integer values, we must have ψ(x) ≥ 2 for some x.
Therefore, there are two distinct points P +γ, P +γ $ in C so their difference
is a lattice point.
Exercise 6.5.4 Show in the previous question if the volume ≥ 2n , the result is
still valid, if C is closed.
lim Cε = C,
ε→0
Exercise 6.5.5 Show that there exist bounded, symmetric convex domains with
volume < 2n that do not contain a lattice point.
|Li (x)| ≤ λi , 1 ≤ i ≤ n.
so we get
λ1 · · · λn
vol(C/2) = ≥1
|det A|
from which the result follows because C is closed.
Exercise 6.5.7 Suppose that among the n linear forms above, Li (x), 1 ≤ i ≤ r1
are real (i.e., aij ∈ R), and 2r2 are not real (i.e., some aij may be nonreal).
Further assume that
That is,
n
,
Lr1 +r2 +j (x) = ar1 +j,k xk , 1 ≤ j ≤ r2 .
k=1
|Li (x)| ≤ λi , 1 ≤ i ≤ n,
Solution. We replace the nonreal linear forms by real ones and apply the
previous result. Set
λr1 +j
|L$r1 +j | ≤ √ ,
2
λr1 +j
|L$$r1 +j | ≤ √ ,
2
254 CHAPTER 6. THE IDEAL CLASS GROUP
then
|Lr1 +j | ≤ λr1 +j ,
so we replace |Lr1 +j | ≤ λr1 +j , |Lr1 +r2 +j | ≤ λr1 +j with L$r1 +j and L$$r1 +j
satisfying the inequalities above. We deduce by the results established in
the previous questions, that this domain contains a nonzero lattice point
provided
λ1 · · · λn 2−r2
> 1,
|det A$ |
where A$ is the appropriately modified matrix. A simple linear algebra
computation shows det A$ = 2−r2 det A.
Exercise 6.5.8 Using the previous result, deduce that if K is an algebraic num-
ber field
8with discriminant dK , then every ideal class contains an ideal b satisfying
N b ≤ |dK |.
ω = x1 α1 + · · · + xn αn ∈ a.
by construction. Also,
|∆|2 = (N a)2 |dK |
L
by Exercise 4.4.5. Hence, |N b| ≤ |dK |. Given any ideal a we have L found
an ideal b in the inverse class whose norm is less than or equal to |dK |.
(x1 , . . . , xr , y1 , z1 , . . . , ys , zs )
where
Yt = {(x1 , . . . , xr , ρ1 , . . . , ρs ) : xi , ρj ≥ 0, x1 + · · · + xr + ρ1 + · · · + ρs ≤ t}.
Let fr,s (t) denote the value of the above integral. By changing variables, it
is clear that
@ 1
fr,s (1) = fr−1,s (1 − x1 ) dx1
0
@ 1
= fr−1,s (1) xr−1+2s
1 dx1
0
1
= fr−1,s (1).
r + 2s
Proceeding inductively, we get
(2s)!
fr,s (1) = f0,s (1).
(r + 2s)!
256 CHAPTER 6. THE IDEAL CLASS GROUP
show that there exist rational integers x1 , . . . , xn (not all zero) such that
x1 a1 + · · · + xn an ∈ C.
x1 a1 + · · · + xn an ∈ C.
as desired.
2r1 −r2 π r2 tn
.
n!
If t is chosen so that this volume is greater than 2n |det A|, then Xt contains
a lattice point (x1 , . . . , xn ) so that
0 != x1 a1 + · · · + xn an ∈ Xt .
as desired.
so that
nn E π Fr 2 nn E π Fn/2
|dK |1/2 ≥ ≥ = cn ,
n! 4 n! 4
say. Then
cn+1 E π F1/2 ! 1
"n
= 1+ ,
cn 4 n
which is greater than 1 for every positive n. Hence cn+1 > cn . We have
c2 > 1 so that |dK | > 1, if n ≥ 2.
Exercise 6.5.14 If K and L are algebraic number fields such that dK and dL
are coprime, show that K ∩ L = Q. Deduce that
Let L = Q(θ) and g its minimal polynomial over Q. If [KL : K] < [L : Q],
then the minimal polynomial h of θ over K divides g and has degree smaller
than that of g. Thus the coefficients of h generate a proper extension T
(say) of Q which is necessarily contained in K. Hence, dT |dK . If we let L̃
be the normal closure of L over Q, then h ∈ L̃[x]. We now need to use the
fact that primes which ramify in L are the same as the ones that ramify in
L̃ (see Exercise 5.6.28). Since T is contained in L̃, we see that dT |dL̃ and
by the quoted fact, we deduce that dL and dK have a common prime factor
if dT > 1, which is contrary to hypothesis. Thus, dT = 1 and by 6.5.13 we
deduce T = Q, a contradiction.
√
Exercise 6.5.15 Using Minkowski’s bound, show that Q( 5) has class number
1.
√
Solution. The discriminant of Q( 5) is 5 and the Minkowski bound is
√
2! √ 5
5= = 1.11 . . . .
22 2
√
The only ideal of norm less than 5/2 is the trivial ideal which is principal.
√
Exercise 6.5.16 Using Minkowski’s bound, show that Q( −5) has class number
2.
√
Solution. The discriminant of Q( −5) is −20 and the Minkowski bound
is
2√ 4
20 = (2.236 . . . ) = 2.84 . . . .
π π
6.5. SUPPLEMENTARY PROBLEMS 259
We need to look at ideals of norm 2.√There is only one ideal of norm 2 and
by Exercise 5.2.5 we know that Z[ −5] is not a principal ideal domain.
Hence the class number must be 2.
√ √
Exercise
√ 6.5.17 Compute the class numbers of the fields Q( 2), Q( 3), and
Q( 13).
The only ideal of norm less than 1.8 is the trivial ideal, which is principal, so
the class number is 1. (Recall that
√ in Exercises √ 2.4.5 and 2.5.4 we showed
that the ring of integers of Q( 2) and Q( 3) are Euclidean and hence
PIDs. So that the class number is 1 for each of these was already known
to us from Chapter 2.)
√
Exercise 6.5.18 Compute the class number of Q( 17).
√
Solution. The discriminant of Q( 17) is 17 and the Minkowski bound is
1
√
2 17 = 2.06 . . . .
Solution. We give a brief hint of the proof. From the preceding question,
the degree n of K is bounded. By Minkowski’s theorem, we can find an
element α != 0 in OK so that
L
|α(1) | ≤ |dK |, |α(i) | < 1, i = 2, . . . , r.
We must show α generates K, but this is not difficult. With these inequal-
ities, the coefficients of the minimal polynomial of α are bounded. Since
the coefficients are integers, there are only finitely many such polynomials.
Exercise 6.5.24 Let p be a prime ≡ 11 (mod 12). If p > 3n , show that the
√
ideal class group of Q( −p) has an element of order greater than n.
2
Solution. Since
√ −p ≡ 1 (mod 3), x ≡ −p (mod 3) has a solution and so
3 splits in Q( −3). Write
(3) = ℘1 ℘$1 .
We claim the order of ℘1 in the ideal class group is at least n. If not, ℘m
1
√
is principal and equals (u + v −p) (say) for some m < n. Taking norms,
we see
3m ≡ u2 + pv 2
6.5. SUPPLEMENTARY PROBLEMS 261
Solution. By Exercise 6.5.13 and 5.6.25, any proper subfield would intro-
duce a power into the discriminant of K.
Chapter 7
Quadratic Reciprocity
7.1 Preliminaries
Exercise 7.1.1 Let p be a prime and a = # 0. Show that x2 ≡ a (mod p) has a
(p−1)/2
solution if and only if a ≡ 1 (mod p).
Because the order of g is p − 1, p − 1|k(p − 1)/2. But this implies that 2|k.
So k = 2k $ . So, we can write a (mod p) as
" "
a ≡ g k ≡ g 2k ≡ (g k )2 (mod p).
263
264 CHAPTER 7. QUADRATIC RECIPROCITY
Solution. The case when p = 2 is trivial since every odd number is con-
gruent to 1 (mod 2). Then we will assume that p is an odd prime. To
begin, we recall Wilson’s theorem (proved in Exercise 1.4.10) which states
that for any prime p, we have (p − 1)! ≡ −1 (mod p). We note that (p − 1)!
can be expressed as
! " ! ! ""
p−1 p−1 p−1
(p − 1)! = 1 · 2 · 3 · · · · p− · p− −1 · · · (p − 1).
2 2 2
Similarly, ! "
a
≡ a(p−1)/2 (mod p),
p
and ! "
b
≡ b(p−1)/2 (mod p).
p
But then
! " ! "! "
ab (p−1)/2 (p−1)/2 (p−1)/2 a b
≡ (ab) =a b ≡ (mod p).
p p p
Exercise 7.1.7 Show that the number of quadratic residues mod p is equal to
the number of quadratic nonresidues mod p.
Solution. From Exercise 7.1.7, the number of residues equals the number
of nonresidues. So, there are (p − 1)/2 residues, and (p − 1)/2 nonresidues.
Thus
p−1 ! "
* a p−1 p−1
= (1) + (−1) = 0.
a=1
p 2 2
This follows from the fact that (x1 + · · · + xn )q ≡ xq1 + · · · + xqn (mod q).
Also, because q is an odd prime, and because (a/p) only takes on the values
±1, we have (a/p)q = (a/p). Hence
* !a"
q
S ≡ ζ aq (mod qR).
p p
a mod p
But as a runs through all the residue classes mod p, so will aq. Thus,
! "
q
Sq ≡ S (mod qR).
p
From this, it follows that
! "
q
q
S ≡ S (mod q),
p
completing the proof.
Since q ≡ 3 (mod 4), we know that (−1/q) = −1. So, (p/q) = −1. Thus,
! " ! "
q p
= (−1)(p−1)/2 = (−1)(−1) = 1.
p q
Solution. We will first compute (5/p). Since 5 ≡ 1 (mod 4), we can use
part (a) of Exercise 7.3.2. So (5/p) = 1 if and only if p ≡ r (mod 5), where
r is a quadratic residue mod 5. It is easy to determine which r are quadratic
residues mod 5; 12 ≡ 1, 22 ≡ 4, 32 ≡ 4, 42 ≡ 1. So, 1 and 4 are quadratic
residues mod 5, while 2 and 3 are not. Thus
1 if p ≡ 1, 4 (mod 5),
! "
5
= −1 if p ≡ 2, 3 (mod 5),
p
0 if p = 5.
Now, we will find (7/p). Since 7 ≡ 3 (mod 4), we must use part (b) of
Exercise 7.3.2. So, we have to compute all the residues mod 28 of all the
squares of odd integers prime to 7. Some calculation reveals that
and
52 , 92 , 172 , 232 ≡ 25 (mod 28).
Thus
1 if p ≡ ±1, ±9, ±25 (mod 28),
! "
7
= −1 if p ≡ ±5, ±11, ±13 (mod 28),
p
0 if p = 7.
270 CHAPTER 7. QUADRATIC RECIPROCITY
Exercise 7.4.3 Assume that p is an odd prime. Show that (d/p) = 0 if and only
if pOK = ℘2 , where ℘ is prime.
√
Solution. ⇒ We claim that pOK = (p, d)2 . Notice that
√ √ √
(p, d)2 = (p2 , p d, d) = (p)(p, d, d/p).
√
Because d√is squarefree, d/p and p are relatively prime. So, (p, d, d/p) = 1.
Since (p, d) is a prime ideal (for the same reason given above), we have
shown that pOK ramifies. √
⇐ Once again, let m be the discriminant of K = Q( d). Since √ pOK
ramifies we can find some a ∈ ℘, but a !∈ pOK . So, a = x + y(m + m)/2.
Since a2 ∈ pOK , we get
Solution. This follows immediately from Theorem 7.4.3 and Exercise 7.4.3.
If (d/p) = −1, then we know that pOK does not split, nor does it ramify.
So pOK must stay inert. Conversely, if pOK is inert, the only possible value
for (d/p) is −1.
Solution. Suppose there are only a finite number of primes of this form.
Let p1 , p2 , . . . , pn be these primes. Let d = (2p1 p2 · · · pn )2 + 1. Let p be
a prime that divides this number. So d ≡ 0 (mod p), which implies that
(−1/p) = 1.
From Exercise 7.1.2, we know that this only occurs if p ≡ 1 (mod 4).
So, p ∈ {p1 , p2 , . . . , pn }. But for any pi ∈ {p1 , p2 , . . . , pn }, pi does not
divide d by construction. So p !∈ {p1 , p2 , . . . , pn }. Thus, our assumption is
incorrect, and so, there must exist an infinite number of primes of the form
4k + 1.
Exercise 7.5.2 Show that there are infinitely many primes of the form 8k + 7.
Solution. Suppose the statement is false, that is, there exist only a finite
number of primes of the form 8k + 7. Let p1 , p2 , . . . , pn be these primes.
Construct the following integer, d = (4p1 p2 · · · pn )2 − 2. Let p be any
prime that divides this number. But then (4p1 p2 · · · pn )2 ≡ 2 (mod p), so
(2/p) = 1. From Theorem 7.1.6, we can deduce that p ≡ ±1 (mod 8).
We claim that all the odd primes that divide d cannot have the form
8k + 1. We observe that 2 | (4p1 p2 · · · pn )2 − 2. So, any odd prime that
divides d must divide 8(p1 p2 · · · pn )2 − 1. If all primes were of the form
8k + 1, we would have
But now consider this equation mod 8. We find that −1 ≡ 1 (mod 8),
which is clearly false. So, there must be at least one odd prime p of the
form 8k + 7 that divides d.
So, p ∈ {p1 , p2 , . . . , pn }. But p cannot be in {p1 , p2 , . . . , pn } since every
pi leaves a remainder of −2 when dividing d. So, {p1 , p2 , . . . , pn } does not
contain all the primes of the form 8k + 7. But we assumed that it did.
We have arrived at a contradiction. Therefore, there must be an infinite
number of primes of the form 8k + 7.
Solution. Since 11 ≡ 3 (mod 4), we use Exercise 7.3.2 (b) which says that
11 is a quadratic residue mod p if and only if p ≡ ±b2 (mod 44) where b
is an odd integer prime to p. If we compute b2 (mod 44) for all possible b,
we get
Exercise 7.6.3 If p ≡ 1 (mod 3), prove that there are integers a, b such that
p = a2 − ab + b2 .
√ √
Solution.
√ Let ρ = (1+ −3)/2 and consider Q( −3). The ring of integers
of Q( −3) is Z[ρ]. Since p ≡ 1 (mod 3), x2 +3 ≡ 0 (mod p) has a solution.
Hence p splits in Z[ρ]. Now use the fact that Z[ρ] is Euclidean.
Exercise 7.6.4 If p ≡ ±1 (mod 8), show that there are integers a, b such that
a2 − 2b2 = ±p.
√
Solution. Consider the Euclidean ring Z[ 2].
Exercise 7.6.5 If p ≡ ±1 (mod 5), show that there are integers a, b such that
a2 + ab − b2 = ±p.
√
Solution. Let ω = (1 + 5)/2 and consider Z[ω].
x2 + y 2 ≡ 1 (mod p),
0 < x < p, 0 < y < p (p an odd prime), is even if and only if p ≡ ±3 (mod 8).
Solution. Pair up the solutions, (x, y) with (y, x). The number of solutions
is even unless (x, x) is a solution which means that 2 is a square mod p.
Solution. All of these are evident from the properties of the Legendre
symbol. For (c), note that a ≡ a$ (mod Q) implies a ≡ a$ (mod qi ) for
i = 1, 2, . . . , s.
Solution.
! " 6 s ! " 6 s !s
−1 −1
= = (−1)(qj −1)/2 = (−1) j=1 (qj −1)/2 .
Q j=1
qj j=1
Hence
a−1 b−1 ab − 1
+ ≡ (mod 2).
2 2 2
Applying this observation repeatedly in our context gives
! "
−1
= (−1)(Q−1)/2 .
Q
2
Exercise 7.6.14 If Q is odd and positive, show that (2/Q) = (−1)(Q −1)/8
.
so we have
a2 − 1 b2 − 1 a2 b2 − 1
+ ≡ (mod 8).
8 8 8
Again applying this repeatedly in our context gives the result.
Exercise 7.6.15 (Reciprocity Law for the Jacobi Symbol) If P and Q are
odd, positive, and coprime, show that
$ %$ %
P Q P −1 Q−1
= (−1) 2 · 2 .
Q P
2r 2s
Solution. Write P = i=1 pi and Q = j=1 qj . Then
! " s !
6 " r !
s 6
6 "
P P pi
= =
Q j=1 j q j=1 i=1
qj
6s 6r ! "
qj pi −1 qj −1
= (−1) 2 · 2
j=1 i=1
pi
! " !
Q pi −1 qj −1
2 · 2
= (−1) i,j
P
by the reciprocity law for the Legendre symbol. But, as noted in the pre-
vious exercises,
* r
pi − 1 P −1
≡ (mod 2)
i=1
2 2
and
*s
qj − 1 Q−1
≡ (mod 2),
j=1
2 2
Exercise 7.6.16 (The Kronecker Symbol) We can define (a/n) for any in-
teger a ≡ 0 or 1 (mod 4), as follows. Define
-a. $ a % 0 if a ≡ 0 (mod 4),
= = 1 if a ≡ 1 (mod 8),
2 −2
−1 if a ≡ 5 (mod 8).
and $ % $ %
d d
= sgn d for n ≡ −m (mod d).
n m
and similarly
! " ! a $" EmF
d 2 d 2 "
= = (−1)a(m −1)/8 $ (−1)(m−1)(d −1)/2 .
m m d
Since 4 | d, the first factors coincide for m and n. The same is true for the
other factors in the case n ≡ m (mod d). But if n ≡ −m (mod d), they
differ by sgn d$ which is sgn d.
In the case a = 0, we note d ≡ 1 (mod 4). Then
! " ! " ! " b ! " ! "b ! "
d d d d 2 d
= = =
n 2b n $ 2 n$ d n$
since (d/2) = (2/d) for d ≡ 1 (mod 4). Thus (d/n) = (n/d) and (−1/d) =
sgn d. Therefore
! " ! "
d d
= for m, n > 0, m ≡ n (mod d)
m n
Exercise 7.6.17 If p is an odd prime show that the least positive quadratic
√
nonresidue is less than p + 1.
(It is a famous conjecture of Vinogradov that the least quadratic nonresidue
mod p is O(pε ) for any ε > 0.)
Solution. Let n be the least positive quadratic nonresidue and m the least
such that mn > p, so that n(m − 1) ≤ p. Since p is prime, n(m − 1) < p <
mn. Now mn − p < n so that
! " ! " ! "
mn − p mn m
1= = =− .
p p p
√
Exercise 7.6.22 Compute the class number of Q( 21).
8 = u2 + 31v 2 .
278 CHAPTER 7. QUADRATIC RECIPROCITY
Now,
an−j = (−1)j sj (α(1) , . . . , α(n) ),
where sj (α(1) , . . . , α(n) ) is the jth symmetric function in the α(i) , i.e., the
sum of all products of the α(i) , taken j at a time. This implies that
! "
n
|an−j | ≤ ≤ n!.
j
Thus, since the aj ’s are bounded, there are only finitely many choices for
the coefficients of the characteristic polynomial of a root of unity α ∈ K
and, hence, only finitely many such roots of unity.
(b) Suppose that α ∈ OK such that |α(i) | ≤ c for all i = 1, . . . , n. As in
(a), let
n
6
fα (x) = (x − α(i) ) = xn + an−1 xn−1 + · · · + a0 ∈ Z[x]
i=1
279
280 CHAPTER 8. THE STRUCTURE OF UNITS
Exercise 8.1.11 Let [K : Q] = 3 and suppose that K has only one real embed-
ding. Then, by Exercise 8.1.8 (c), WK = {±1} implies that UK = {±uk : k ∈ Z},
where u > 1 is the fundamental unit in K.
(a) Let u, ρeiθ , ρe−iθ be the Q-conjugates of u. Show that u = ρ−2 and that
dK/Q (u) = −4 sin2 θ(ρ3 + ρ−3 − 2 cos θ)2 .
(b) Show that |dK/Q (u)| < 4(u3 + u−3 + 6).
(c) Conclude that u3 > d/4 − 6 − u−3 > d/4 − 7, where d = |dK |.
8.1. DIRICHLET’S UNIT THEOREM 283
Solution. (a) NK (u) = uρ2 = ±1, so u = ±ρ−2 , but u > 1 implies that
u = ρ−2 ,
6
dK/Q (u) = (u(r) − u(s) )2
1≤r<s≤3
(b)
|dK/Q (u)| = 4 sin2 θ(ρ3 + ρ−3 − 2 cos θ)2 .
Now set x = ρ3 + ρ−3 , c = cos θ and consider
This function attains a maximum when x = −2(1−c2 )/c and this maximum
is 4(1 − c2 ) ≤ 4. Therefore
d d
u3 > − 6 − u−3 > − 7.
4 4
√
3
Exercise 8.1.12 Let α = 2, K = Q(α). Given that dK = −108:
(a) Show that, if u is the fundamental unit in K, u3 > 20.
(b) Show that β = (α − 1)−1 = α2 + α + 1 is a unit, 1 < β < u2 . Conclude that
β = u.
K = Q(α).
p0 = a0 , q0 = 1,
p1 = a0 a1 + 1, q1 = a 1 ,
pk = ak pk−1 + pk−2 , qk = ak qk−1 + qk−2 ,
1 a0 a1 + 1 p1
C1 = [a0 , a1 ] = a0 + = = .
a1 a1 q1
8.2. UNITS IN REAL QUADRATIC FIELDS 285
Exercise 8.2.2 Let {ai }i≥0 be an infinite sequence of integers with ai ≥ 0 for
i ≥ 1 and let Ck = [a0 , . . . , ak ]. Show that the sequence {Ck } converges.
Moreover, each C2j+1 > a0 so that the sequence {C2j+1 }j≥0 is decreasing
and bounded from below and is, thus, convergent, say limj→∞ C2j+1 = α1 .
Also,
C0 < C2 < C4 < · · ·
and C2j < C2k+1 for all j, k ≥ 0. In particular, each C2j < C1 . The
sequence {C2j }j≥0 is increasing and bounded from above and, therefore,
also converges, say limj→∞ C2j = α2 . We will show that α1 = α2 .
Since each ai ≥ 1, q0 , q1 ≥ 1, we easily see, by induction on k, that
qk = ak qk−1 + qk−2 ≥ 2k − 3. By Exercise 8.2.1 (c),
1 1
C2j+1 − C2j = ≤ → 0,
q2j+1 q2j (4j − 1)(4j − 3)
1
ak = [αk ], αk+1 = .
αk − ak
1
α = α0 = [α0 ] + (α0 − [α0 ]) = a0 +
α1
= [a0 , α1 ] = [a0 , a1 , α2 ] = · · · = [a0 , a1 , . . . , ak , αk+1 ],
αk+1 pk + pk−1
α=
αk+1 qk + qk−1
8.2. UNITS IN REAL QUADRATIC FIELDS 287
so that
1 1
1 αk+1 pk + pk−1 pk 11
1
|α − Ck | = 1 − 1
αk+1 qk + qk−1 qk
1 1
1 −(pk qk−1 − pk−1 qk ) 1
1
= 1 1
(αk+1 qk + qk−1 )qk 1
1 1
1
1 1 1
1
= 1
(αk+1 qk + qk−1 )qk 1
1 1
< ≤ →0
qk2 (2k − 3)2
as k → ∞. Thus,
α = lim Ck = [a0 , a1 , . . . ].
k→∞
Exercise 8.2.5√ Let d be a positive integer, not a perfect square. Show that, if
|x2 −dy 2 | <√ d for positive integers x, y, then x/y is a convergent of the continued
fraction of d.
√
Solution. Suppose first that 0 < x2 − dy 2 < d. Then
√ √ √
(x + y d)(x − y d) > 0 ⇒ x > y d,
1 1 √
1√ 1 x √ x−y d
⇒ 1 d − x1 = − d=
1 y1 y y
2 2
x − dy x2 − dy 2 1
= √ < √ < 2.
y(x + y d) y(2y d) 2y
Thus,
√ by Theorem 8.2.4 (b), x/y is a convergent of the continued fraction
of d. √
Similarly, if − d < x2 − dy 2 < 0, then
1 1 x
0 < y 2 − x2 < √ ⇒ y>√ ,
d d d
1 1 √
1 1 1 1 y − x/ d
⇒ 1√ − y 1 =
y
−√ =
1 d x1 x d x
2 2
y − x /d y 2 − x2 /d 1
= √ < √ < 2.
x(y + x/ d) 2x / d
2 2x
√
Thus y/x is a convergent of the continued fraction of 1/ d.
Let α be any irrational number. Then α = [a0 , a1 , . . . ] implies 1/α =
[0, a0 , a1 , . . . ]. We therefore have that the (k + 1)th convergent of the con-
tinued fraction of 1/α is the reciprocal of the kth convergent of α, for all
k ≥ 0.
Using this fact, we √ find that, as before, x/y is a convergent of the
continued fraction of d.
288 CHAPTER 8. THE STRUCTURE OF UNITS
Exercise 8.2.7 Show that the simple continued fraction expansion of a quadratic
irrational α is periodic.
we have
P0 = 0, P1 = d,
Q0 = 1, Q1 = 1,
L L
α0 = d2 + 1, α1 = d + d2 + 1,
a0 = d, a1 = 2d.
αk pk−1 + pk−2
α=
αk qk−1 + qk−2
α$ − Ck−2
→ 1.
α$ − Ck−1
√
Therefore, αk$ < 0, and, since αk > 0, αk − αk$ = 2 d/Qk > 0, for all
2
sufficiently large k. We also have Qk Qk+1 = d − Pk+1 so
2
Qk ≤ Qk Qk+1 = d − Pk+1 ≤d
and
2
Pk+1 ≤ d − Qk ≤ d
for sufficiently large k. Thus there are only finitely many possible values
for Pk , Qk and we conclude that there exist integers i < j such that Pi =
Pj , Qi = Qj . Then ai = aj and, since the ai are defined recursively, we
have
α = [a0 , a1 , . . . , ai−1 , ai , . . . , aj−1 ].
290 CHAPTER 8. THE STRUCTURE OF UNITS
√
Exercise 8.2.11 (a) Show that [d, 2d] is the continued fraction of d2 + 1.
2
(b) Conclude that,√if d + 1 is squarefree,
√ d ≡ 1, 3 (mod 4), then the fundamen-
tal√unit of√Q( d2 + √ 1) is d + d + 1. Compute the fundamental unit of
2
Solution.
√ (a) Observing that d2 < d2 + √
1 < (d + 1)2 for all d > 0, we see
that [ d2 + 1] = d and setting α = α0 = d2 + 1, we have
P0 = 0, P1 = d,
Q0 = 1, Q1 = 1,
L L
α0 = d2 + 1, α1 = d + d2 + 1,
a0 = d, a1 = 2d.
√
This implies
√ that the period of the continued fraction of d + 1 is 1.
2
P0 = 0, P1 = d, P2 = d,
Q0 = 1, Q1 = 2, Q2 = 1,
L √ L
d+ d2 + 2
α0 = d2 + 2, α1 = α2 = d + d2 + 2,
2
a0 = d, a1 = d, a2 = 2d.
√
Therefore the period of the continued fraction of d2 + 2 is 2, so
L
d2 + 2 = [a0 , a1 , a2 ] = [d, d, 2d]
and thus
p1 1 d2 + 1
=d+ = .
q1 d d
(d) For√all d, d2 +2 ≡ 2, 3 (mod
√ 4) so, if d is squarefree,
√ the fundamental
unit in Q( d2 + 2) is p1 + q1 d2 + 2 = d2 + 1 + d d2 + 2.
Exercise 8.3.2 Determine the units of an imaginary quadratic field from first
principles.
√
Solution. We have already determined the units of Q(i) and Q( −3).
2
They are ±1, ±i and ±1, ±ρ, √ ±ρ , respectively. Now we determine the
√ field Q( −d) where −d ≡ 2, 3 (mod 4). All units are
units of an arbitrary
of the form a + b −d with a2 + db2 = 1. It is easy to see that since d ≥ 2,
this has no solution except√for a = ±1, b = 0. If −d ≡ 1 (mod 4), then units
will be of the form (a + b −d)/2 where a ≡ b (mod 2) and a2 + db2 = 4.
Since we already know the units for d = 3, then d ≥ 7 and once again, the
only solution is a = ±1, b = 0.
Exercise
√ 8.3.3 Suppose that 22n √
+ 1 = dy 2 with d √
squarefree.
√ Show that 2 +
n
√
(b) Prove from first principles that all units of Q( 51) are given by εn , n ∈ Z.
Solution. Consider η = 1 + θ.
Exercise 8.3.6 Let p be an odd prime > 3 and suppose that it does not divide
the class number of Q(ζp ). Show that
xp + y p + z p = 0
Solution. We factor
p−1
6
xp + y p = (x + ζpi y) = −z p .
i=0
(x + ζpi y) = api
for some ideal ai . Since p does not divide the class number, ai itself must
be principal. Therefore
(x + ζpi y) = εαp ,
where ε is a unit in Q(ζp ) and α is an integer of Q(ζp ).
By Exercise 4.3.7, Z[ζp ] is the ring of integers of Q(ζp ) and
1, ζp , ζp2 , . . . , ζpp−2
α = a0 + a1 ζp + · · · + ap−2 ζpp−2 ,
so that
αp ≡ ap0 + · · · + app−2 ≡ a (mod p),
where a = a0 +· · ·+ap−2 , by a simple application of Fermat’s little Theorem
(Exercise 1.1.13). Also, we may write ε = ζps η where 0 ≤ s < p and η is a
real unit, by Theorem 8.1.10. Hence, for i = 1,
Then
ζp−s (x + ζp y) − ζps (x + ζp−1 y) ≡ 0 (mod p).
Since 1, ζp , . . . , ζpp−2 is an integral basis for Q(ζp ),
that is,
ζp3 (x + ζp y) ≡ ζp x + y (mod p),
which gives p | x (since p ≥ 5), again a contradiction.
Solution. If d < 0, the norm of any nonzero element is greater than 0 and
so the notions of restricted class group and ideal class group coincide. If
d > 0, then H = √ H0 if and only if there exists a unit in K of norm −1.
This
√ is because dOK is in the same coset of OK (mod√P0 ) if and only if
d = εα with NK (α) > 0, and ε is a unit. Since NK ( d) < 0, this can
happen if and only if ε is a unit of norm −1.
8.3. SUPPLEMENTARY PROBLEMS 295
Exercise 8.3.8 Given an ideal a of a quadratic field K, let a( denote the conju-
gate ideal. If K has discriminant d, write
|d| = pα1
1 p2 · · · pt ,
a = r℘a1 1 · · · ℘at t ,
r > 0, ai = 0, 1 uniquely.
℘a1 1 · · · ℘at t , ai = 0, 1.
r℘a1 1 · · · ℘at t ,
Exercise 8.3.10 With the notation as in the previous two questions, show that
there is exactly one relation of the form
Solution. This is now immediate from the previous two exercises. Since
the restricted class group has index 1 or 2 in the ideal class group, the
divisibility assertion follows.
Solution. If d has t distinct prime divisors, then 2t−1 divides the class
number. Thus t ≤ 1. Since the discriminant is either squarefree or four
times a squarefree number, the result is now clear.
Exercise 8.3.13 If a real quadratic field K has odd class number, show that K
has a unit of norm −1.
[3, 1, 2, 1, 6]
√
Exercise 8.3.15 In Chapter 6 we showed that Z[ 14] is a PID√(principal ideal
domain). Assume the following hypothesis: given α, β ∈ Z[ 14], such that
gcd(α, β) √
= 1, there is a prime π ≡ α (mod β) for which the fundamental √unit
ε = 15 + 4 14 generates the coprime residue classes (mod π). Show that Z[ 14]
is Euclidean.
and √
(a$ )2g = ( d + 1).
2g
√ to g. Observe that n +1
We claim that a has order greater than or equal √ ≡
2 (mod 4) because n is odd. Therefore 1, d is an integral basis of Q( d).
If am were principal, then
√
am = (u + v d)
298 CHAPTER 8. THE STRUCTURE OF UNITS
implies that
nm = |u2 − dv 2 |,
√
which by the previous exercise is either 0, 1 or > d. The former cannot
hold since n ≥ 5. Thus,
√ L
nm > d = n2g + 1 > ng .
Hence, m > g. Since m | 2g, we must have that a has order 2g.
It is conjectured that there are infinitely many squarefree numbers of
the form n2g + 1. Thus, this argument does not establish that there are
infinitely many real quadratic fields whose class number is divisible by g.
However, by a simple modification of this argument, we can derive such a
result. We leave it as an exercise to the interested reader.
Chapter 9
Exercise 9.1.2 If π ∈ Z[ρ] is such that N (π) = p, a rational prime, show that π
is a prime of Z[ρ].
Hence p ≡ (2a − b)2 (mod 3), a contradiction since 2 is not a square mod
3.
Finally, if p ≡ 1 (mod 3), then by quadratic reciprocity:
! " ! "! " E F ! "
−3 −1 3 p 1
= = = =1
p p p 3 3
299
300 CHAPTER 9. HIGHER RECIPROCITY LAWS
Exercise 9.1.4 Let π be a prime of Z[ρ]. Show that αN (π)−1 ≡ 1 (mod π) for
all α ∈ Z[ρ] which are coprime to π.
Exercise 9.1.5 Let π be a prime not associated to (1 − ρ). First show that
3 | N (π) − 1. If (α, π) = 1, show that there is a unique integer m = 0, 1 or 2 such
that
α(N (π)−1)/3 ≡ ρm (mod π).
β 3 − 1 = (β − 1)(β − ρ)(β − ρ2 ).
Since π is prime and divides β 3 − 1, it must divide one of the three factors
on the right. If π divides at least two factors, then π | (1 − ρ) which means
π is an associate of 1 − ρ, contrary to assumption. Thus, β ≡ 1, ρ, or ρ2
(mod π) as desired.
r(N (π) − 1)
≡0 (mod N (π) − 1).
3
Hence 3 | r, and α is a cube mod π. That is, x3 ≡ α (mod π) has a solution.
This proves (a).
9.1. CUBIC RECIPROCITY 301
For (b),
! "
αβ
≡ (αβ)(N (π)−1)/3 ≡ α(N (π)−1)/3 β (N (π)−1)/3
π 3
EαF !β "
≡ (mod π).
π 3 π 3
implies
α(N (π)−1)/3 ≡ χπ (α) (mod π)
on the one hand. On the other hand,
Exercise 9.1.8 If q ≡ 2 (mod 3), show that χq (α) = χq (α2 ) and χq (n) = 1 if n
is a rational integer coprime to q.
Solution. Since q = q,
Exercise 9.1.9 Let N (π) = p ≡ 1 (mod 3). Among the associates of π, show
there is a unique one which is primary.
302 CHAPTER 9. HIGHER RECIPROCITY LAWS
If ζm
i j
≡ ζm (mod ℘) (say), then ζm
j−i
≡ 1 (mod ℘) so that m ≡ 0 (mod ℘),
contrary to m !∈ ℘. Thus 1, ζm , . . . , ζm
m−1
are distinct mod ℘. Moreover,
the cosets#they represent
$∗ form a multiplicative subgroup of Z[ζm ]/℘ of order
m. Since Z[ζm ]/℘ has order q − 1 = N (℘) − 1, we must have m | q − 1.
α(q−1)/m ≡ ζm
i
(mod ℘).
Solution. By definition,
! "
ζm (N (℘)−1)/m
≡ ζm (mod ℘).
℘ m
(N (℘)−1)/m
Since both (ζm /℘)m and ζm are mth roots of unity and by Exer-
cise 9.2.1, distinct roots represent distinct classes, we must have
! "
ζm (N (℘)−1)/m
= ζm .
℘ m
Exercise 9.2.5 Suppose a and b are ideals coprime to (m). Show that:
9.2. EISENSTEIN RECIPROCITY 305
Solution. Parts (a) and (b) are immediate from Exercise 9.2.3. For (c), we
note that xm ≡ α (mod ℘) has a solution for every prime ideal ℘ dividing
a. Thus by Exercise 9.2.3 (a), (α/℘)m = 1 for every prime ideal ℘ dividing
a. thus, (α/a)m = 1.
Exercise 9.2.6 Show that the converse of (c) in the previous exercise is not
necessarily true.
α ≡ β (mod ℘1 ),
α ≡ γ (mod ℘2 ).
Solution. Let λ = 1 − ζ) . Since the prime ideal (λ) has degree 1, there
is an a ∈ Z such that α ≡ a (mod λ). Hence (α − a)/λ ≡ b (mod λ). So
we can write α ≡ a + bλ (mod λ2 ). Since ζ) = 1 − λ, we have ζ)c ≡ 1 − cλ
(mod λ2 ). Thus,
(ζ j − ζ i )x = ζ j (x + ζ i y) − ζ i (x + ζ j y) ∈ a.
Exercise 9.2.9 Show that the ideals (x + ζ i y) are perfect 1th powers.
α = (x + y)*−2 (x + ζy).
Show that:
(a) the ideal (α) is a perfect 1th power.
(b) α ≡ 1 − uλ (mod λ2 ) where u = (x + y)*−2 y.
(x + y))−1 ≡ 1 (mod 7)
Solution. We have
x* + y * + z * = 0
pf − 1
ug ≡0 (mod 7).
7
x* + y * + z * = 0
for some integers x, y, z coprime to 1, then show that p*−1 ≡ 1 (mod 12 ) for every
p | xyz. Deduce that 2*−1 ≡ 1 (mod 12 ).
Exercise 9.3.2 Let a be a nonsquare integer greater than 1. Show that there
are infinitely many primes p such that (a/p) = −1.
x ≡ 1 (mod qi ), 1 ≤ i ≤ k,
x ≡ 1 (mod 8),
x ≡ 1 (mod ri ), 1 ≤ i ≤ m − 1,
x ≡ c (mod rm ),
with c any nonresidue mod rm . (It is here that we are assuming a has at
least one odd prime divisor.) Let b be a solution greater than 1 and write
b = p1 · · · pt as its prime decomposition with pi not necessarily distinct.
9.3. SUPPLEMENTARY PROBLEMS 309
Since b ≡ 1 (mod 8), (2/b) = 1. Also, by the quadratic reciprocity law for
the Jacobi symbol, (ri /b) = (b/ri ). Thus
EaF ! "e 6
m E F ! "
2 ri c
= = = −1.
b b i=1 b rm
Also,
EaF t !
6 "
a
= .
b i=1
pi
Exercise 9.3.3 Suppose that x2 ≡ a (mod p) has a solution for all but finitely
many primes. Show that a is a perfect square.
Exercise 9.3.4 Let K be a quadratic extension of Q. Show that there are in-
finitely many primes which do not split completely in K.
√
Solution. Let K = Q( D), with D squarefree and greater than 1. By
the previous question, there are infinitely many primes p such that x2 ≡
D (mod p) has no solution. Hence (D/p) = −1. By the theory of the
Kronecker symbol, we deduce that p does not split in K.
Solution. We must show that if a is not a qth power, then there are
infinitely many primes p such that xq ≡ a (mod p) has no solution. We
will work in the field K = Q(ζq ). Write
where the ℘i are distinct prime ideals of OK . We claim that q ! ei for some
i. To see this, let pi = ℘i ∩ Z. Since (q, a) = 1 we have q != pi for any i, so
each pi is unramified in OK . Thus
x ≡ 1 (mod qi ), 1 ≤ i ≤ k,
x ≡ 1 (mod q),
x ≡ 1 (mod ℘j ), 1 ≤ j ≤ n − 1,
x ≡ c (mod ℘n ),
Exercise 9.3.6 Let p ≡ 1 (mod 3). Show that there are integers A and B such
that
4p = A2 + 27B 2 .
A and B are unique up to sign.
p = a2 − ab + b2 .
Thus,
Exercise 9.3.7 Let p ≡ 1 (mod 3). Show that x3 ≡ 2 (mod p) has a solution if
and only if p = C 2 + 27D2 for some integers C, D.
p = a2 − ab + b2
4p = A2 + 27B 2 ,
x3 − 2y 3 = 23z m
Analytic Methods
#$ 1
%−1
ζ(s) = 1− s ,
p
p
the term 1/ns occurs exactly once. The assertion is now evident.
Exercise 10.1.2 Let K be an algebraic number field and OK its ring of integers.
The Dedekind zeta function ζK (s) is defined for Re(s) > 1 as the infinite series
, 1
ζK (s) = ,
a
(N a)s
where the sum is over all ideals of OK . Show that the infinite series is absolutely
convergent for Re(s) > 1.
Solution. For any s with Re(s) > 1, it suffices to show that the partial
sums
* 1
(N a)s
N a≤x
313
314 CHAPTER 10. ANALYTIC METHODS
* 1 6 ! 1
"−1
≤ 1 − .
(N a)σ (N ℘)σ
N a≤x N ℘≤x
For each prime ideal ℘, we have a unique prime number p such that N (℘) =
pf for some integer f . Moreover, there are at most [K : Q] prime ideals
corresponding to the same prime p. In fact, they are determined
5g from the
e
factorization pOK = ℘e11 · · · ℘gg and by Exercise 5.3.17, i=1 ei fi = [K : Q]
where N ℘i = pfi . Since ei ≥ 1 and fi ≥ 1, we find g ≤ [K : Q]. Hence
* 1 6! 1
"−[K:Q]
≤ 1− σ .
(N a)σ p
N a≤x p≤x
2
Since the product p (1 − p )
−σ −1
converges absolutely for σ > 1, the result
follows.
#$ 1
%−1
ζK (s) = 1− .
℘
(N ℘)s
Exercise 10.1.5 Show that (s − 1)ζ(s) can be extended analytically for Re(s) >
0.
*∞ @ ∞
1 [x] dx
s
=s .
n=1
n 1 xs+1
10.1. THE RIEMANN AND DEDEKIND ZETA FUNCTIONS 315
Exercise 10.1.9 Show that the number of integers (a, b) with a > 0 satisfying
a2 + Db2 ≤ x is
πx √
√ + O( x).
2 D
√
Solution. Corresponding to each such (a, b) we associate (a, Db) which
lies inside the circle√u2 + v 2 ≤ x. We now count these “lattice” points.
We will call (a, Db) internal if (a + 1)2 + D(b + 1)2 ≤ x. Otherwise,
call it a boundary lattice point. Let I be the number of internal lattice
points, and
√ B the number of boundary lattice points. Each lattice point
has area D. Thus √ √
π
DI ≤ x ≤ D(I + B)
2
since in our count a > 0 and b ∈ Z. A little reflection shows that any
boundary point is contained in the annulus
#√ √ $2 #√ √ $2
x − D + 1 ≤ u2 + v 2 ≤ x+ D+1
√
which has area O( xD). Thus
√ √
DB = O( xD)
√
and we get B = O( x). Thus,
πx √
I = √ + O( x).
2 D
316 CHAPTER 10. ANALYTIC METHODS
√
Exercise 10.1.10 Suppose K = Q( −D) where D > 0 and −D #≡ 1 (mod 4)
and OK has class number 1. Show that (s − 1)ζK (s) extends analytically to
Re(s) > 12 and find
lim (s − 1)ζK (s).
s→1
(In the next section, we will establish a similar result for any quadratic field
K.)
i=1
10.2. ZETA FUNCTIONS OF QUADRATIC FIELDS 317
Solution. Since dK = d or 4d, the result is clear for odd primes p from
the previous exercise. We therefore need only consider p = 2. If 2 | dK ,
then by Theorem 7.4.5, 2 ramifies and there is only one ideal of norm 2. If
2 ! dK , then
! " ;
dK 1 if dK ≡ 1 (mod 8),
=
2 −1 if dK ≡ 5 (mod 8),
by the definition of the Kronecker symbol. The result is now immediate
from Theorem 7.4.5.
Solution. The norm of any prime ideal is either p or p2 , the latter occurring
if and only if (d/p) = −1. Thus for α = 2, the formula is established and
for α = 1, the previous exercise applies. If (d/p) = −1, then clearly apα = 0
if α is odd and if α is even, then there is only one ideal of norm pα . If p
splits, then any ideal of norm pα must be of the form
℘j (℘$ )α−j
for some j, where pOK = ℘℘$ . It is now clear that apα = α + 1 which is
the sum
*α ! "
dK
.
j=0
pj
318 CHAPTER 10. ANALYTIC METHODS
by the previous exercise. The result is now immediate upon expanding the
product.
Exercise 10.2.6 Let dK be the discriminant of the quadratic field K. Show that
there is an n > 0 such that (dK /n) = −1.
Choose n0 such that (n0 , |dK |) = 1 and (dK /n0 ) = −1. (This is possible
by the previous exercise.) Then
! " * ! "
dK dK
S= .
n0 nn0
n mod |dK |
(n,|dK |)=1
so that S = 0.
Now define v by
v|dK | ≤ x < (v + 1)|dK |.
Then,
* ! dK " * !
dK
"
=
n n
n≤x |dK |v≤n<x
since
* ! dK " * v * ! "
dK
=
n n
n<|dK |v j=1 (j−1)|dK |<n<j|dK |
where σ = Re(s). The latter series converges absolutely for Re(s) > 1.
Hence *
χ(b) = 0.
b mod m
(b,m)=1
Now, partition the interval [1, x] into subintervals of length m and suppose
that km < x ≤ (k + 1)m. Then
* * *
χ(n) = χ(n) + χ(n).
n≤x n≤km km<n≤x
The first sum on the right-hand side is zero and the second sum is bounded
by m .
#$ χ(p)
%−1
L(s, χ) = 1− s .
p
p
10.3. DIRICHLET’S L-FUNCTIONS 321
since the series converges absolutely in Re(s) > 1. By Exercise 10.3.5, the
inner sum on the right-hand side is ϕ(m) when pn ≡ a (mod m) and zero
otherwise. The result is now immediate.
Solution. The primes that split completely in Q(ζm ) are those primes
p ≡ 1 (mod m). Thus, because there are φ(m) ideals of norm p for p ≡ 1
(mod m),
6! 1
"−1
ζK (s) = 1−
℘
(N ℘)s
6 ! "−φ(m)
1
= 1− s g(s),
p
p≡1 mod m
6 6 ! "−φ(m)
1
L(s, χ) = 1− h(s),
ps
χ p≡1 mod m
Show that cn ≥ 0.
10.4. PRIMES IN ARITHMETIC PROGRESSIONS 323
Since *1
= +∞,
p
p
As s → 1+ , we see that
* 1
= +∞
p
p≡1 mod m
Note that * 1 * 1
≤ < ∞.
npn p
p(p − 1)
n≥2
p
As s → 1+ , we see that
* 1
= +∞
p
p≡a mod m
Hence there are infinitely many primes in any given coprime residue class
mod m.
Solution.
*
χ(n)g(χ) = χ(n)χ(a)e2πia/m
a mod m
*
= χ(b)e2πibn/m
b mod m
upon setting a = bn in the first sum. Observe that as a ranges over coprime
residue classes (mod m), so does bn since (n, m) = 1.
10.5. SUPPLEMENTARY PROBLEMS 325
√
Exercise 10.5.2 Show that |g(χ)| = m.
Solution.
1 12
m−1
* *1
m−1
1 *
1
2 2 2πibn/m 1
|χ(n)| |g(χ)| = 1 χ(b)e 1
1 1
n=1 n=0 b mod m
* m−1
*
= χ(b1 )χ(b2 ) e2πi(b1 −b2 )n/m .
b1 ,b2 n=0
π(m−b)
m | = |sin
Since |sin πb m |, we may suppose b ≤ m/2. In that case,
1 1 ! "
1sin πb 1 ≥ 2 πb
1 1
1 m1 π m
326 CHAPTER 10. ANALYTIC METHODS
Exercise 10.5.4 Let p be prime. Let χ be a character mod p. Show that there
is an a ≤ p1/2 (1 + log p) such that χ(a) #= 1.
where ;
0 if x < u,
A(x) = 5
u<n≤x χ(n) if x ≥ u.
√
By Pólya–Vinogradov, A(x) = O( m log m) and so
* χ(n) @ ∞ A(x) dx !√ "
m log m
= = O .
n>u
n u x2 u
Therefore
*∞ * χ(n) !√ "
χ(n) m log m
L(1, χ) = = +O .
n=1
n n u
n≤u
10.5. SUPPLEMENTARY PROBLEMS 327
Exercise 10.5.6 Let D be a bounded open set in R2 and let N (x) denote the
number of lattice points in xD. Show that
N (x)
lim = vol(D).
x→∞ x2
Solution. Without loss of generality, we may translate our region by a
lattice point to the first quadrant. A lattice point (u, v) ∈ xD if and only if
(u/x, v/x) ∈ D. This suggests we partition the plane into squares of length
1/x with sides parallel to the coordinate axes. This then partitions D into
small squares each of area 1/x2 . The number of lattice points of xD is then
clearly the number of “interior squares.” For this partition of the region, we
write down the lower and upper Riemann sums. Let Ix denote the number
of “interior” squares, and Bx the number of “boundary” squares. Then, by
the definition of the Riemann integral
Ix Ix + Bx
≤ vol(D) ≤
x2 x2
so that
Ix Ix + Bx
lim 2
= lim = vol(D).
x→∞ x x→∞ x2
On the other hand,
Ix ≤ N (x) ≤ Ix + Bx .
Thus,
N (x)
lim = vol(D).
x→∞ x2
Exercise 10.5.7 Let K be an algebraic number field, and C an ideal class of K.
Let N (x, C) be the number of nonzero ideals of OK belonging to C with norm
≤ x. Fix an integral ideal b in C −1 . Show that N (x, C) is the number of nonzero
principal ideals (α) with α ∈ b with |NK (α)| ≤ xN (b).
wN (x, C)
lim = vol(D).
x→∞ x
Set u1 = Re(uβ1 + vβ2 ), u2 = Im(uβ1 + vβ2 ) so that
@@
vol(D) = du dv
|uβ1 +vβ2 |2 <1
@@
2
= L · du1 du2
N b |dK |
u21 +u22 <1
2π
= L .
N b |dK |
Exercise 10.5.9 Let K be a real quadratic field with discriminant dK , and fun-
damental unit ε. Let C be an ideal class of OK . Show that
N (x, C) 2 log ε
lim = √ ,
x→∞ x dK
where N (x, C) denotes the number of integral ideals of norm ≤ x lying in the
class C.
Now consider
; G
2
0 < |uβ11 + vβ2 ||uβ1$ + vβ2$ | < 1,1
D= (u, v) ∈ R : 1 1 +vβ2 1
0 < log 1 |uβ1 +vβ2uβ
|1/2 |uβ " +vβ " |1/2 1
< log ε.
1 2
Solution. Clearly, *
N (x; K) = N (x, C),
C
N (x; K) * N (x, C)
lim = lim
x→∞ x x→∞ x
C
Exercise 10.5.11 Let K be a real quadratic field. Let N (x, K) denote the
number of integral ideals of norm ≤ x. Show that
N (x; K) 2h log ε
lim = 8 ,
x→∞ x |dK |
where h is the class number of K.
Thus,
∞ ! "
N (x, K) * dK 1
lim = .
x→∞ x n=1
n n
Comparing this limit with the previous two questions gives the desired
result.
*∞ ! "
D 1
= O(log d).
n=1
n n
which
√ was derived using the Pólya-Vinogradov inequality. Choosing u =
d, and noting that
* !D" 1
= O(log d),
√ n n
n≤ d
Estimating the infinite series as in the previous question, the result is now
immediate.
we differentiate to obtain
*∞
ζ$ Λ(n)
− (s) = ,
ζ n=1
ns
ψ(x) = x + O(x1/2+ε ),
Density Theorems
Solution. There are two conditions defining Bx . The second one involving
units is invariant under the homogenous change of variables. The first
inequality gets multiplied by tn .
Exercise 11.1.3 Show that N (x, C) = O(x). Deduce that N (x; K) = O(x).
333
334 CHAPTER 11. DENSITY THEOREMS
1
Exercise 11.1.6 Prove that ζ(s, C) extends to the region 5(s) > 1 − n
except
for a simple pole at s = 1 with residue
2r1 (2π)r2 RK
8 .
w |dK |
Deduce that ζK (s) extends to 5(s) > 1 − n1 except for a simple pole at s = 1
with residue
2r1 (2π)r2 hK RK
ρK := 8 ,
w |dK |
where hK denotes the class number of K.
*∞
am
f (s) = s
:= ζ(s, C) − αK ζ(s),
m=1
m
By Theorem 10.1.4, f (s) converges for <(s) > 1 − n1 . As ζ(s) has a simple
pole at s = 1 with residue 1, the latter assertions are immediate.
Exercise 11.1.7 Prove that there are infinitely many prime ideals in OK which
are of degree 1.
Solution. We have
* 1 * 1
log ζK (s) = + .
℘
N (℘) s nN (℘)ns
n≥2,℘
The second sum is easily seen to converge for <(s) > 1/2. The first sum
can be separated into two parts, one over primes of first degree and the
other over primes of degree ≥ 2. Again, the second sum converges for
<(s) > 1/2. If the first sum consisted of only finitely many terms, the right
hand side would tend to a finite limit as s → 1+ , which is not the case as
the Dedekind zeta function has a simple pole at s = 1.
Exercise 11.1.8 Prove that the number of prime ideals ℘ of degree ≥ 2 and
with norm ≤ x is O(x1/2 log x).
unless a = OK .
since an ideal has odd norm if and only if it has no prime ideal divisor
above 2. Interchanging summation and using Theorem 11.1.5, we obtain
the result.
We see that A(x) ∼ B(x) if and only if A(x) + B(x) ∼ 2B(x). That is, if
and only if ! "
1 6 1
= 1− .
2 N (℘)
℘|2
336 CHAPTER 11. DENSITY THEOREMS
Let ℘1 , ..., ℘t be the prime ideals above 2 with norms 2m1 , ..., 2mt respec-
tively. The above equation implies
t ! "
1 6 2mi − 1
= .
2 i=1 2mi
The right hand side is a product of odd numbers and so the only way the
left hand side can be odd is if m1 + · · · + mt = 1 which means that there is
only one prime ideal above 2 and it has norm 2. This can only happen if
K = Q or if K is a quadratic field in which 2 ramifies.
Let t = x1/n . Show that there is a δ > 0 (independent of x) such that for each
lattice point P contained in V(t−δ)n , all the points contained in the translate of
the standard unit cube by P belong to Vx .
where the summation is over all positive integers i1 , ..., in such that i1 +
· · · + in = n and the ai1 ,...,in ’s are rational integers. If P = (u1 , ..., un ),
then any point contained in the translate of the standard unit cube by P
is of the form (u1 + t1 , ..., un + tn ) with ti ’s bounded by 1. Thus, by the
solution of Exercise 11.1.1, we deduce that the norm of any such point is
* n−1
ai1 ,...,in ui11 · · · uinn + O(x n ).
i1 ,...,in
Exercise 11.2.1 Show that L(s, χ) converges absolutely for 5(s) > 1 and that
#$ χ(℘)
%−1
L(s, χ) = 1− ,
℘
N (℘)s
6! χ(℘)
"−1
L(s, χ) = 1−
℘
N (℘)s
and the product converges absolutely for <(s) > 1 if and only if
* 1
℘
N (℘)s
converges in this region, which is certainly the case as there are only a
bounded number of prime ideals above a given prime p. The non-vanishing
is also clear.
where we have used the fact that as C runs over elements of the ideal class
group, so does CC0 . The result is now immediate.
338 CHAPTER 11. DENSITY THEOREMS
Exercise 11.2.5 Let C be an ideal class of OK . For 5(s) > 1, show that
, , 1
χ(C) log L(s, χ) = hK
χ ℘m ∈C
mN (℘)ms
where the first summation is over the characters of the ideal class group and the
second summation is over all prime ideals ℘ of OK and natural numbers m such
that ℘m ∈ C.
Solution. In the left hand side, we insert the series for log L(s, χ). By
interchanging the summation and using the orthogonality relations estab-
lished in the Exercise 11.2.3, we obtain the desired result.
Solution. As noted earlier, the number of prime ideals above a fixed prime
p is at most the degree of the number field. Thus, the result is clear from
the fact that * 1
mpms
m≥2,p
3 + 4 cos θ + cos 2θ ≥ 0,
as follows. We write
χ(℘) = eiθp
so that for real σ > 1,
# $
< 3 log ζK (σ) + 4 log L(σ, χ) + log L(σ, χ2 )
* 1
= (3 + 4 cos θp + cos 2θp ) ≥ 0.
m,℘
mN (℘)σm
Hence,
|ζK (σ)3 L(σ, χ)4 L(σ, χ2 )| ≥ 1.
If L(1, χ) = 0, the left hand side of this inequality tends to zero as σ → 1+ ,
which is a contradiction.
Exercise 11.2.8 Let C be a fixed ideal class in OK . Show that the set of prime
ideals ℘ ∈ C has Dirichlet density 1/hK .
Exercise 11.2.9 Let m be a natural number and (a, m) = 1. Show that the set
of primes p ≡ a(mod m) has Dirichlet density 1/φ(m).
Exercise 11.2.10 Show that the set of primes p which can be written as a2 +5b2
has Dirichlet density 1/4.
√
Solution. We have already seen that the class number of Q( −5) is 2.√The
set of prime ideals lying in the principal class are of the form (a + b −5)
and have norm a2 + 5b2 . By Hecke’s theorem, the Dirichlet density of these
prime ideals is 1/2 and taking into account that there are two ideals of
norm p in the principal class gives us the final density of 1/4.
340 CHAPTER 11. DENSITY THEOREMS
Exercise 11.2.11 Show that if K = Q, the principal ray class group mod m is
isomorphic to (Z/mZ)∗ .
Solution. The elements of the principal ray class group are the ideals (α)
modulo m with a totally positive generator and (α, m) = 1. The result is
now clear.
Solution. Suppose not. Take a prime ideal ℘ which is not in the Galois
orbit of ℘i (say) lying above the prime ideal p. By the Chinese remainder
theorem (Theorem 5.3.13), we may find an element x ∈ ℘ and x − 1 ∈ σ(℘i )
for all σ in the Galois group. But then, NK/k (x) is an integer of Ok which
on one hand is divisible by ℘ and on the other coprime to p, a contradiction.
τa (ζm ) = ζm
a
.
Solution. By Theorem 7.4.2, we see that these are precisely the set of
primes which split completely in K and by Chebotarev, the density of such
primes is 1/2.
Solution. By Theorem 5.5.1 and Exercise 5.5.2, we see that the set of
primes p for which f (x) ≡ 0 (mod p) has a solution coincides with the set
of primes p which split completely in the field obtained by adjoining a root
of f . By our assumption, this is a Galois extension of degree n and to say
p splits completely is equivalent to saying that σp = 1. By Chebotarev, the
Dirichlet density of such primes is 1/n.
11.4. SUPPLEMENTARY PROBLEMS 341
Exercise 11.3.8 Let q be prime. Show that the set of primes p for which p ≡ 1
(mod q) and
p−1
2 q ≡ 1(mod p),
has Dirichlet density 1/q(q − 1).
Solution. If *
cχ Aχ = 0
χ
Solution. Since the row rank of a matrix is equal to the column rank, it
is clear that we can choose a set of such Hi ’s and ψi ’s. Thus,
*
IndG
Hi ψi = aHi (ψi , χ)χ.
χ
Moreover,
aH (ψ, χ) = (IndG
H ψ, χ)
are all non-negative integers. Thus, the inverse matrix consists of rational
entries.
Exercise 11.4.3 Deduce from the previous exercise that some positive integer
power of the Artin L-function L(s, χ; K/k) attached to an irreducible character
χ admits a meromorphic continuation to 5(s) = 1.
Solution. Since the right hand side represents the L-function attached to
the regular representation, which is IndG
1 1, we have that it is equal to
Exercise 11.4.5 Fix a complex number s0 ∈ C with 5(s0 ) ≥ 1 and any finite
Galois extension K/k with Galois group G. For each subgroup H of G define the
Heilbronn character θH by
,
θH (g) = n(H, χ)χ(g)
χ
where the summation is over all irreducible characters χ of H and n(H, χ) is the
order of L(s, χ; K/K H ) at s = s0 . (By Exercise 11.4.3, the order is a rational
number.) Show that θG |H = θH .
Solution. We have *
θG |H = n(G, χ)χ|H .
χ
But *
χ|H = (χ|H , ψ)ψ
ψ
By Frobenius reciprocity,
Thus, *
θG |H = n(H, ψ)ψ = θH .
ψ
Exercise 11.4.6 Show that θG (1) equals the order at s = s0 of the Dedekind
zeta function ζK (s).
344 CHAPTER 11. DENSITY THEOREMS
Solution. We have *
θG (1) = n(G, χ)χ(1)
χ
1 *
(θG , θG ) = |θG (g)|2 .
|G|
g∈G
Thus,
*
|θH (h)| ≤ n(H, ψ) = θH (1),
ψ
which by Exercise 11.4.6 is the order of the Dedekind zeta function ζK (s)
at s = s0 . The result is now immediate.
L(s, χ; K/k)
Exercise 11.4.11 Show that ζK (s)/ζk (s) is entire. (This is called the Brauer-
Aramata theorem.)
Exercise 11.4.12 (Stark) Let K/k be a finite Galois extension of algebraic num-
ber fields. If ζK (s) has a simple zero at s = s0 , then L(s, χ; K/k) is analytic at
s = s0 for every irreducible character χ of Gal(K/k).
for s0 != 1.
n(:g;, 1) − n(G, 1) ≥ 0.
Therefore,
*
|f (g)| ≤ n(:g;, 1) − n(G, 1) + n(:g;, ψ) = ords=s0 (ζK (s)/ζk (s)),
ψ*=1
[B] Bell, E.T. Men of Mathematics. Simon and Schuster, New York,
1937.
[FM] R. Foote and V. Kumar Murty, Zeros and Poles of Artin L-series,
Math. Proc. Camb. Phil. Soc., 105 (1989), 5-11.
[He] Hecke, E., Lectures on the Theory of Algebraic Numbers, GTM 77,
Springer-Verlag, 1981.
[HM] Harper, M., and Murty, R., Euclidean rings of algebraic integers,
Canadian Journal of Mathematics, 56(1), (2004), 71-76.
347
348 BIBLIOGRAPHY
349
350 INDEX
zeta function
Dedekind, 127, 313
Riemann, 127